You are on page 1of 176

L I NI U

Ngay t nm 1736, nh ton hc Euler gii quyt thnh cng bi ton t hp v by cy cu thnh ph Knigsberg, c (nay l Kaliningrad, Nga) nm trn sng Pregel, bao gm o hai hn o ln ni vi nhau v vi t lin bi by cy cu. Bi ton c t ra l C th i theo mt tuyn ng m i qua mi cy cu ng mt ln ri quay li im xut pht hay khng ?. V k t n nay, tri qua nhiu thng trm ca lch s, l thuyt t hp vn pht trin mnh m, ng gp nhiu cho s pht trin ca khoa hc v k thut hin i. Chng ta thng gp cc bi ton t hp trong cc m hnh sn xut nh Lp lch cho mt c quan, xut hin trong gii php an ton giao thng vi cc m hnh t cc trm xe bus ti u nht trong mt thnh ph, vo qun l con ngi vi m hnh Lp thi kho biu v phn vic,. . . , hoc c th ng dng gin tip trong cc thut ton gii cc bi ton ti u trong cc phn mm my tnh nh thut ton tm kim ca Google, Yahoo,. . . , hay cc phn mm ng dng m chng ta vn ang s dng hng ngy. Chnh v vy ton t hp lun dnh c s quan tm rt ln t cc nh ton hc, cc thy, c gio v cc bn hc sinh yu thch mn ton. Ton t hp l mt lp cc bi ton kh, thng xut hin trong cc k thi hc sinh gii cp tnh, thnh ph, cp quc gia, quc t. Do , gii quyt thnh tho v c vn kin thc chc chn, su rng v ton t hp l nim mong c ca nhiu gio vin v hc sinh. Mc d ton t hp quan trng nh vy nhng cc ti liu v ton t hp, ri rc dnh cho hc sinh gii Vit Nam vn cn rt t v hn ch. Xut pht t thc t trn v vi mc ch cung cp ti liu cht lng gm nhiu chuyn ton t hp nng cao gip cho vic hc tp ca hc sinh tt hn v cc thy, c gio c thm ti liu ging dy, nhm bin son bao gm cc gio vin, cc sinh vin h c nhn ti nng ton, cc hc sinh gii quc gia, quc t n t mi min ca T quc cng nhau vit nn cc chuyn , cc bi ging v ton t hp nng cao. Tuyn tp cc chuyn t hp ra i nh du cho thnh cng ln trong vic chia s tri thc cho cng ng cc bn yu thch mn ton, m nhng kinh nghim lm bi, nhng cch gii hay v sng to c c t s c kt trong thi gian hc tp ca nhiu thnh vin v ang l hc sinh gii quc gia, quc t hay y tnh s phm ca cc gio vin tch ly c trong qu trnh tham gia hc tp, ging dy. Tuyn tp c hon thnh v gi ti bn c trong dp Tt Nguyn n, hi vng n s l mt mn qu nm mi thc s hu ch vi bn c trn khp t nc. hon thnh cun sch, nhm bin tp xin gi li cm n su sc ti cc thy gio, cc bn hc sinh, sinh vin tham gia gi cc chuyn , cc bi ton trn din n MathScope. ng thi cng xin gi li cm n su sc ti ban qun tr din n MathScope v thy gio, TS. Trn Nam Dng - HKHTN - HQG TP. H Ch Minh c v, ng vin v cho nhiu nhn xt c gi tr cun sch va c gi tr chuyn mn cao m li min ph v ti chnh vi bn c. 3

4 Do thi gian gp rt v trnh c hn, d rt c gng nhng sai st l kh trnh khi. Mi kin ng gp cun sch hon thin hn xin gi v a ch hoangquan9@gmail.com hoc alephvn@gmail.com. H Ni, ngy 22 thng 1 nm 2012 (ngy Tt nin nm Nhm Thn) i din nhm bin son Ch bin Hong Minh Qun Phan c Minh

M CL C
Li ni u . . . . . . . . . . . . . . . . . . . . . . . . . . . . . . . . . . . . . . . . . . . . . . . . . . . . . . . . . . . . . . . . . . . . . . . . . . . . . . . . 3 S dng php m chng minh cc ng thc t hp Nguyn Tt Thu . . . . . . . . . . . . . . . . . . . . . . . . . . . . . . . . . . . . . . . . . . . . . . . . . . . . . . . . . . . . . . . . . . . . . . . . . . . 7 Phng php m bng hai cch Phan c Minh . . . . . . . . . . . . . . . . . . . . . . . . . . . . . . . . . . . . . . . . . . . . . . . . . . . . . . . . . . . . . . . . . . . . . . . . . . 17 Phng php xy dng m hnh trong gii ton t hp L Phc L . . . . . . . . . . . . . . . . . . . . . . . . . . . . . . . . . . . . . . . . . . . . . . . . . . . . . . . . . . . . . . . . . . . . . . . . . . . . . . 33 Phng php hm sinh Hong Minh Qun . . . . . . . . . . . . . . . . . . . . . . . . . . . . . . . . . . . . . . . . . . . . . . . . . . . . . . . . . . . . . . . . . . . . . . . 53 Phng php hm sinh L Hu Phc, Trn Nguyn Quc Cng . . . . . . . . . . . . . . . . . . . . . . . . . . . . . . . . . . . . . . . . . . . . . . . . 69 Gii ton t hp bng i lng bt bin Trn Gia Huy . . . . . . . . . . . . . . . . . . . . . . . . . . . . . . . . . . . . . . . . . . . . . . . . . . . . . . . . . . . . . . . . . . . . . . . . . . . 101 Mt s bi ton t mu L Tun Linh . . . . . . . . . . . . . . . . . . . . . . . . . . . . . . . . . . . . . . . . . . . . . . . . . . . . . . . . . . . . . . . . . . . . . . . . . . . 119 Cc tr v bt ng thc ri rc Nguyn Hin Trang . . . . . . . . . . . . . . . . . . . . . . . . . . . . . . . . . . . . . . . . . . . . . . . . . . . . . . . . . . . . . . . . . . . . . 141 Mt s bi ton t hp in hnh v bn c Nguyn Vit Dng . . . . . . . . . . . . . . . . . . . . . . . . . . . . . . . . . . . . . . . . . . . . . . . . . . . . . . . . . . . . . . . . . . . . . . 165 S Stirling loi hai Hong Minh Qun . . . . . . . . . . . . . . . . . . . . . . . . . . . . . . . . . . . . . . . . . . . . . . . . . . . . . . . . . . . . . . . . . . . . . . 173

S D NG PHP M CH NG MINH CC NG TH C T
Nguyn Tt Thu1

H P

Nh chng ta bit cc khi nim hon v, chnh hp, t hp c hnh thnh t cc bi ton m. Cc khi nim ny ra i gip chng ta trnh by bi ton m n gin hn. Tuy nhin k khi gp cc bi chng minh cc ng thc lin quan n Pn , Cn th chng ta thng s dng cc bin i i s hoc khai trin nh thc Newton chng minh. Do vic chng minh k cc bi ton ng thc lin quan n Pn , Cn v cc khi nim ca n khng c mi quan h no. iu ny t nhiu lm mt i v p ca cc khi nim ton hc ni chung v cc khi k nim Pn , Cn ni ring. Trong chuyn ny chng ti gii thiu vi cc bn cch chng minh k mt s ng thc lin quan n Pn , Cn bng phng php m. Ni dung ca phng php ny nh sau : k Gi s ta cn chng minh mt ng thc lin quan n Pn , Cn c dng A = B. Ta s i m s cch thc hin mt cng vic X no theo hai cch: Cch 1 ta c kt qu s cch thc hin cng vic X l A. Cch 2 cho ta kt qu s cch thc hin cng vic X l B. T ta c c A = B. lm tt phng php ny chng ta cn hiu ngha ca cc i lng xut hin trong hai v ca ng thc. Chng hn: 2m : l s tp con ca tp X gm m phn t v cng l s cch chn m phn t t m cp v mi cp chn mt phn t. 2m 1: l s tp con khc rng ca tp X gm m phn t.
k Cn : s tp con gm k phn t ca tp X gm n phn t.

Chng ta s bt u bng cc v d sau y:


k nk V d 1. Chng minh rng Cn = Cn vi mi k, n N; n

1; 0

n.

Li gii. Xt tp X = {x1 , x2 , . . . , xn }. Ta thy v tri l s tp con A gm k phn t ca tp X. lp A ta lm theo hai cch nh sau: 1. Mi cch ly k phn t trong X, ta c mt tp con A gm k phn t ca tp X, nn s k tp con A l Cn .
1

Gio vin trng THPT L Hng Phong, ng Nai.

8 2. thit lp A ta c th lm nh sau: Mi cch ly n k phn t ca tp X v loi n k phn t ny i, ta c c c k phn t cn li l mt tp con A gm k phn t ca X. nk Nn s tp con A l: Cn
k nk T ta c c Cn = Cn v bi ton c chng minh.

V d 2. Cho n

2, k l cc s t nhin tha 1

n. Chng minh rng

k k k1 Cn = Cn1 + Cn1

Li gii. V v tri ca ng thc l s tp con gm k phn t ca tp gm n phn t nn ta i m s tp con A gm k phn t ca tp X = {x1 , x2 , . . . , xn }. k Cch 1. S tp A c Cn tp. Cch 2. S tp A gm hai loi, ta s i m s tp thuc hai loi ny. Loi 1. Gm nhng tp con cha phn t xn . Mi tp A thuc loi ny cho ta mt tp A = A \ {xn } l tp con gm k 1 phn t ca tp X \ {xn }. V ngc li mi tp A cho ta mt tp A nn suy ra s tp A thuc loi ny chnh bng s k1 tp A v bng Cn1 . Loi 2. Gm nhng tp con khng cha phn t xn . Nh vy cc phn t ca tp A c ly k t tp X \ {xn } gm n 1 phn t nn s tp A thuc loi ny l Cn1 . k1 k Do theo cch 2 th s tp A l Cn1 + Cn1 . k1 k k Vy ta c Cn = Cn1 + Cn1 . V d 3. Cho n 1 l s t nhin. Chng minh ng thc sau:
0 Cn 2 1 + Cn 2 n n + + (Cn )2 = C2n

Li gii. Ta thy VP ca ng thc chnh l s tp con A gm n phn t ca tp X gm 2n phn t nn ta xt bi ton sau: Hy tnh s tp con A gm n phn t ca tp X = {x1 , x2 , . . . , x2n }. n Cch 1. Ta c s tp con A l C2n . Cch 2. Chia tp X thnh hai tp X1 = {x1 , x2 , . . . , xn } v X2 = {xn+1 , . . . , x2n }. lp tp con A ta lm nh sau: Ly k phn t (k = 0, n) thuc tp X1 , ri ly n k phn t cn li thuc tp X2 v ta c k nk k 2 Cn Cn = Cn cch chn A ng vi mi k. Cho k chy t 0 n n ri ly tng ta c c kt qu chnh l s tp A cn tm, hay
0 Cn 2 1 + Cn 2 n n + + (Cn )2 = C2n

V d 4. Chng minh ng thc


n
nk n k [ 2 ] 2k Cn Cnk = C2n+1

k=0

9 Li gii. Ta thy v phi l s cch chn n phn t t tp X gm 2n + 1 phn t nn ta xt bi ton sau: Tnh s cch chn n phn t t tp X gm 2n + 1 phn t. n Cch 1. S cch chn chnh bng C2n+1 . Cch 2. Ta chia X thnh n cp v phn t x. chn n phn t t X ta thc hin cc bc sau: k Bc 1. Ta chn k cp (k = 0, n) t n cp chia ta c Cn cch, sau mi cp ta chn k mt phn t nh vy ta c 2k Cn cch chn. Bc 2. Chn nk cp trong n k cp cn li. 2 V nk = nk nu n k chn v nk = nk1 nu n k l. 2 2 2 2 Do ta s chn x nu n k l v khng chn x nu n k chn. [ nk ] 2 S cch chn bc ny l Cnk . nk k [ 2 ] Suy ra c 2k Cn Cnk cch trong mi ln chn. Cho k chy t 0 n n v ly tng ta c s cch chn l:
n
nk n k [ 2 ] 2k Cn Cnk = C2n+1

k=0

V d 5. Chng minh rng vi mi s nguyn dng n ta c:


m k Cn+k 2mk + k=0 k=0 n k Cm+k 2nk = 2m+n+1

Do c

Li gii. Xt tp X = {1, 2, . . . , m + n + 1}. Ta s i m s tp con ca X. Cch 1. Ta c 2m+n+1 tp con. Cch 2. S tp con ca X gm hai loi: Loi 1. Gm nhng tp con c dng A = {x1 , x2 , . . . , xn+i } vi 1 i m + 1 v x1 < x2 < < xn+i v xn+1 = n + k + 1 vi 0 k m . lp tp con loi ny ta lm nh sau: n Bc 1. Chn n phn t t n + k phn t (vi 0 k m) ta c Cn+k cch. Bc 2. B sung mt tp con ca tp {n + k + 1, n + k + 2, . . . , n + m + 1} ta c 2mk cch.
m k=0 k Cn+k 2mk tp con A ca X c nhiu hn n phn t. n k=0 k Cm+k 2nk tp con ca X c nhiu hn m phn t.

Tng t c

M mi tp con ca X c hn m phn t ng vi mt tp con ca X c khng qu n phn t, suy ra s tp con ca X c khng qu n phn t l Do vy


m k=0 k Cn+k 2mk n k=0 k Cm+k 2nk .

n k=0 m

k Cm+k 2nk

chnh l s tp con ca X.
n

Vy ta c

k Cn+k 2mk k=0

+
k=0

k Cm+k 2nk = 2m+n+1

V d 6. Chng minh ng thc sau vi n 1 l s t nhin

1 2 3 n Cn + 2Cn + 3Cn + + nCn = n2n1

10 Li gii. Ta thy n chnh l s cch ly mt phn t t mt tp gm n phn t, cn 2n1 chnh l s tp con ca tp gm n1 phn t. Do ta xt bi ton sau: Cho tp X = {x1 , x2 , . . . , xn }. Hy m s cp (a, A) trong a X v A l mt tp con ca tp X = X \ {a}. Cch 1. Ta c n cch chn a, vi mi cch chn a ta c 2n1 cch chn A. Theo quy tc nhn ta c n2n1 cp (a, A). k nk Cch 2. Ta chn A l mt tp con gm k phn t ca (k = 0, n 1), nn c Cn = Cn cch chn A. Mi cch chn tp A ta s chn a X \A nn c nk cch chn a. Khi cho k chy t 0
n1 k=0 nk 1 2 n (nk)Cn = Cn +2Cn + +nCn

n n1 v ly tng th ta c c s cp (a, A) hay l c cp (a, A). So snh kt qu hai cch m ta c

1 2 3 n Cn + 2Cn + 3Cn + + nCn = n2n1

V d 7. Chng minh ng thc sau:


k 1 k1 k 0 0 k Cn Cn + Cn Cn1 + + Cn Cnk = 2k Cn k Li gii. Thy c 2k l s tp con ca mt tp gm k phn t, cn Cn l s tp con gm k phn t ca tp gm n phn t nn ta xt bi ton sau: Cho tp X = {x1 , x2 , . . . , xn }. Hy m s cp (A, M ) trong A l mt tp con gm k phn t ca X v M l mt tp con ca A. k Cch 1. Ta c Cn cch chn tp A, vi mi cch chn A ta c 2k cch chn M nn c tt c k 2k Cn cp (A, M ). i Cch 2. Ta c Cn cch chn M (0 i k) . Sau khi chn M ta chn k i phn t t n i ki i phn t cn li ri gp vi M ta c c tp A, nn vi mi i ta c Cn Cni cch chn cp

(A, M ). Cho i chy t 0 n k v ly tng ta c s cp (A, M ) l Vy ta c y l ng thc cn chng minh. V d 8. Chng minh rng vi mi s t nhin n
1 Cn 2 2 + 2 Cn 2

k i=0

ki i Cn Cni .

0 k 1 k1 k 0 k Cn Cn + Cn Cn1 + + Cn Cnk = 2k Cn

1, ta lun c:

n n1 + + n (Cn )2 = nC2n1

n1 Li gii. Ta thy nC2n1 chnh l s cc cp (a, A), trong a l mt phn t thuc tp X1 = {x1 , x2 , . . . , xn } cn A l mt tp con gm n1 phn t ca tp X = {x1 , . . . , xn , xn+1 , . . . , x2n }\ {a}. Nn ta xt bi ton sau: Cho hai tp ri nhau X1 = {x1 , x2 , . . . , xn } v X2 = {a1 , a2 , . . . , an }. Hy m s cp (a, A), trong a X1 cn A l mt tp con bt k gm n 1 phn t ca tp X = X1 X2 \ {a}. n1 Cch 1. chn a ta c n cch, vi mi cch chn a ta c C2n1 cch chn A nn c tt c n1 nC2n1 cch chn cp (a, A). k Cch 2. Ly k phn t thuc X1 (1 k n) ta c Cn cch, ri ta chn a t k phn t va

11 chn ta c k cch. Sau khi chn a ta ch cn li k 1 phn t. Tip tc chn n k phn t thuc X2 v kt hp vi k 1 phn t cn li trn ta c mt tp con A gm n 1 phn k nk k 2 t ca X. Nn mi trng hp ny ta c k Cn Cn = k Cn cch chn. Cho k chy t 1 n n v ly tng ta c s cch chn cc cp (a; A) l
1 Cn 2 2 + 2 Cn 2 n + + n (Cn )2

Do ta c iu cn chng minh.

V d 9. Cho s t nhin n 1. Mt hon v ca tp A = {1, 2, 3, . . . , n} c gi l hon v bo tn a A nu nh phn t a nguyn v tr c ca n trong hon v mi. K hiu Pn (k) l s hon v bo tn ng k phn t ca A. Chng minh rng: (a) kPn (k) = nPn1 (k 1); (b)
n

kPn (k) = n!.


k=0

Li gii. (a) Vi mi k = 1, 2, . . . , n ta i m s cp (i; f ) trong f bo tn ng k v tr v f (i) = i. Cch 1. Ta c s cch chn i l k v s cch chn f l Pn (k) nn s cp (i; f ) l kPn (k). Cch 2. Ta xt i l mt phn t c nh (tc l f (i) = i).Khi ta c mt hon v bo tn k 1 phn t ca tp A = A \ {i} v vi mi hon v bo tn k 1 phn t ca tp A ta b sung thm i vo ta s c mt hon v bo tn k phn t ca tp A. V c n cch chn i v c Pn1 (k 1) hon v bo tn k 1 phn t ca tp A nn s cp (i; f ) l nPn1 (k 1). T ta suy ra kPn (k) = nPn1 (k 1). (b) Theo kt qu trn ta c
n n

kPn (k) = n
k=1 k=1

Pn1 (k 1)

M Pn1 (0) + Pn1 (1) + + Pn1 (n 1) chnh l s hon v ca tp B gm n 1 phn t m trong hon v bo tn 0, 1, 2, . . . , n 1 phn t, do tng ny chnh bng s hon v ca tp B v bng (n 1)!. Vy
n

k=1

kPn (k) = n(n 1)! = n!

Nhn xt. Trong mt s trng hp ta c th dng m theo hai cch chng minh cc bt ng thc. Chng hn nu ta chng minh c A = B v D < A, B < C th ta c D < C. V d 10. Trong mt k thi c a th sinh v s l b 3 gim kho. Mi gim kho nh gi tng th sinh v cho kt lun th sinh hay trt. Gi s k l s tha mn: vi hai gim kho bt k th s th sinh m h cho kt lun ging nhau nhiu nht l k. Chng minh rng k a b1 2b

12 Li gii. Ta i m s b (A, B, x) trong x l mt th sinh no cn A, B l hai gim kho cho cng mt kt qu khi nh gi x. Gi N l s b nh vy, ta s m N theo hai cch. Cch 1. C tt c b(b1) b i gim kho v mi b i gim kho cho khng qu k th sinh 2 cng mt kt qu nn ta c kb(b 1) N (1) 2 Cch 2. Ta xt mt th sinh X c nh v c m gim kho cho th sinh X ny , suy ra c x(x1) cp gim kho cho X cng mt kt qu u v c (bx)(bx1) cp gim kho nh gi 2 2 th sinh ny trt. Do tng s cp gim kho nh gi th sinh ny cng mt kt qu l x(x 1) + (b x)(b x 1) 2x2 2xb + b2 b = 2 2 Nn suy ra N= T (1) v (2) ta suy ra c: kb(b 1) 2 Do k a 2x2 2bx + b2 b = Do b l nn
(b1)2 2

a(2x2 2xb + b2 b) 2

(2)

a(2x2 2xb + b2 b) 2 2x2 2xb + b2 b b(b 1) (3) (b 1)2 1 2 2

Mt khc:

(2x b)2 + (b 1)2 1 2

Z, suy ra 2x2 2bx + b2 b (b 1)2 2 (4)

T (3) v (4) ta c

k a

b1 . 2b

Qua cc v d trn ta thy, vn dng tt phng php ny chng ta cn hiu r ngha ca cc i tng c trong ng thc. Vi vic xt mt bi ton m v m theo nhiu cch s cho chng ta cc kt qu khc nhau v mt hnh thc v t c c cc ng thc t hp.

Cc bi tp ngh
Bi tp 1. Chng minh ng thc 1 1 k Cn = C k+1 k+1 n + 1 n+1 Bi tp 2. Chng minh ng thc
k k2 k(k 1)Cn = n(n 1)Cn2

13 Bi tp 3. Chng minh ng thc


k+1 k i i1 iCn Ck = nCn+k1 i=1

Bi tp 4. Chng minh ng thc


0 k 1 k1 k 0 k Cm Cn + Cm Cn + + Cm Cn = Cm+n

Bi tp 5. Chng minh rng


k1 k1 k1 k Ck1 + Ck + + Cn1 = Cn

Bi tp 6. Chng minh rng

Trong b (k1 , k2 , . . . , kn ) tha k1 + k2 + + kn = k.

k! = nk k1 !k2 ! kn !

Bi tp 7. Cho trc mt s nguyn dng l n ln hn 1 v cc s nguyn k1 , k2 , . . . , kn . K hiu a = (a1 , a2 , . . . , an ) l mt hon v trong n! hon v ca A = {1, 2, . . . , n}. Chng minh rng tn ti hai hon v b, c v s nguyn m sao cho sao cho
n

i=1

ki (bi ci ) = m n!

Bi tp 8. Tnh tng T = 1 n1 !n2 !...n2011 ! (n2 + 2n3 + + 2011n2011 )!

y ly tng theo tt c cc b th t cc s t nhin (n1 , n2 , . . . , n2011 ) tha iu kin n1 + 2n2 + + 2011n2011 = 2011.

Hng dn v gi
k+1 k Bi tp 1. Ta cn chng minh (n + 1)Cn = (k + 1)Cn+1 . Ta m s cp (a, A) trong a l mt phn t ca tp X = {1, 2, . . . , n + 1} cn A l mt tp con gm k phn t ca X \ {a} theo hai cch. k k Cch 1. C n + 1 cch chn a, khi chn a th c Cn cch chn A. Do c tt c (n + 1)Cn cp (a, A). Cch 2. Ly mt tp con A ca X gm k + 1 phn t ri t A ly mt phn t a v k+1 A = A \ {a} nn ta c (k + 1)Cn+1 cp (a, A). T ta c iu cn chng minh.

Bi tp 2. Ta m s cc b (a, b, A) trong a, b X = {x1 , x2 , . . . , xn } cn A l mt tp con gm k 2 phn t ca X \ {a, b} theo hai cch. k2 Cch 1. Chn a, b t X ta c n(n 1), khi s c Cn2 cch chn A.

14
k2 Nn c tt c n(n 1)Cn2 b (a, b, A). k Cch 2. Trc ht ta ly t X ra k phn t, c Cn cch ly ri t k phn t ta ly a, b ta k c k(k 1) cch ly. Tp cn li c k 2 phn t chnh l A nn ta c k(k 1)Cn s b (a, b, A). T ta c iu cn chng minh.

Bi tp 3. Cho tp A = {x1 , x2 , . . . , xn } v B = {a1 , a2 , . . . , ak }. Ta m s b (x, X) trong x mt phn t thuc A cn X l mt tp con gm k phn t ca tp A B \ {x} theo hai cch. k k Cch 1. Ta c n cch chn x v Cn+k1 cch chn X nn c tt c nCn+k1 cp. Cch 2. Ly t A B ra k + 1 phn t . Trong k + 1 phn t ny c i(i = 1, . . . , k + 1) phn t thuc A v k + 1 i phn t thuc B nn mi trng hp ta c i cch chn a v k phn t cn li lp thnh tp A. Do s cp l:
k+1 i=1 i k+1i iCn Ck . T ta c iu cn chng minh.

Bi tp 4. Hy m s cch ly k phn t t tp gm m + n phn t.


k1 Bi tp 5. Ta c Ci1 ( i = k, n) chnh l s tp con gm k phn t ca tp X = {1, 2, 3, . . . , n} cha i v khng cha phn t no ln hn i. k1 k1 k1 Do Ck1 + Ck + + Cn1 chnh l s tp con gm k phn t ca X m ta bit s k tp con ny chnh bng Cn nn ta c ng thc cn chng minh.

Bi tp 6. Cho tp X = {1, 2, 3, . . . , n}. m s dy gm k phn t thuc X. Cch 1. Mi v tr c n cch chn nn c nk s cc dy cn lp. Cch 2. Xt mi cch xp dy c k phn t, trong mi phn t i xut hin ki ln (ki Ta c s cch k1 !k2k! n ! . !k Do
k i=1 k! k1 !k2 !kn !

0).

l s cch xp dy gm k phn t.

T ta c iu cn chng minh. Bi tp 7. K hiu S(a) =


n i=1

ki ai , ta cn chng minh tn ti hai hon v b, c sao cho S(b)S(c)

chia ht cho n!. Ta tnh S(a) theo hai cch. Cch 1. Trong tng S(a) (theo ng d mod n!), mi ki c tnh lp trong mi giai tha vi h s ng (n 1)! ln ng vi mi s i A nhn n lm h s. Do tng h s ki trong S(a) l (n + 1)! (n 1)! (1 + 2 + + n) = 2 Nn S(a) =
(n+1)! 2 n i=1

ki .

Cch 2. Gi s khng tn ti hai hon v b, c sao cho S(b) S(c) chia ht cho n!. Khi s d m S(a) chia cho n! c n! s d khc nhau 0, 1, 2, . . . , n! 1. Do ta c S(a) (n! 1)n! 2 (mod n!)

15 T ta suy ra c
(n+1)! 2 n i=1

ki

(n!1)n! 2

(mod n!) ()

Ta cho n l th v tri ca () chia ht cho n!, trong khi v phi ca () khng chia ht cho n! nn dn ti iu v l. Bi tp 8. Gi A l tp cc b c dng (a1 , a2 , . . . , a2011 , . . . , a2010+2011 ), trong ai {0, 1} vi mi i = 1, 4021; Trong mi b c ng 2011 ch s 1. K hiu A(n1 ,n2 ,...,n2011 ) l tp cc b c th t (a1 , . . . , a4021 ) A sao cho trong mi b c ng ni nhm gm i ch s 1 ng lin tip nhau trong b (tc l nhm c dng 0 11 . . . 1 0, 1 . . . 1 0, 11 . . . 1 0) Khi ta c: A = tha
2011 i=1

A(n1 ,...,n2011 ) (hp ly theo cc b c th t cc s t nhin (n1 , n2 , . . . , n4021 )

k s

k s

k s

Suy ra

i ni = 2011) A(n1 ,...,n2011 ) = 2011! n1 !n2 ! n2011 ! (2011 n1 n2011 )! 1 = n1 !n2 ! n2011 ! (n2 + 2n3 + + 2011n2011 )!

|A| =

A(n1 ,...,n2011 ) =

2010 Mt khc, ta c |A| = C4021 . Do

1 n1 !n2 ! n2011 ! (n2 + 2n3 + + 2011n2011 )!

1 2010 = C4021 n1 !n2 ! n2011 ! (n2 + 2n3 + + 2011n2011 )!

Ti liu tham kho


[1] V nh Ha, L thuyt t hp v cc bi ton ng dng, NXB Gio dc, 2003. [2] Ng Thc Lanh, Tm hiu i s t hp ph thng, NXB Gio dc, 1998. [3] Cc ti liu t internet. [4] Cc din n v ton nh Din n Math.vn http://math.vn/index.php Din n MathScope http://forum.mathscope.org/index.php

16

PHNG PHP M B NG HAI CCH


Phan c Minh1

1. C s l thuyt
Nguyn l 1 (Nguyn l m bng hai cch). Nu cng mt s lng c m theo hai cch th cc kt qu thu c phi bng nhau. Khi p dng phng php m bng hai cch, ta cn ch n cc biu thc c ngha trong t hp : n l s tp con c k phn t ca mt tp c n phn t; n! l s cc hon v ca n k phn t; n l s cc bi s ca k trong n s nguyn dng u tin; nk l s chnh hp lp k chp k ca n phn t. . . Nm vng c bn cht ca cc biu thc trn, ta c th chng minh mt s ng thc v bt ng thc t hp bng phng php m bng hai cch.

2. Cc bi ton p dng phng php m bng hai cch


2.1. m s tp con, s cp v s hon v
Bi ton 1. Cho k, n l cc s t nhin vi 0 n k = k n nk n. Chng minh rng

Li gii. y l v d c bn nht cho phng php m bng hai cch. Ta quan st thy v tri ca ng thc cn chng minh l s cc k-tp con ca [n] = {1, 2, . . . , n}2 , trong khi v phi l s cc (n k)-tp con ca [n]. Nh vy ng thc s c chng minh nu ta ch ra c s cc k-tp con bng s cc (n k)-tp con. M iu ny l hin nhin v mi k-tp con S ca [n] tng ng duy nht vi (n k)-tp con [n] \ S. Vy s cc k-tp con bng s cc (n k)-tp con. Ta c iu cn chng minh. Bi ton 2. Chng minh rng vi n, k l cc s nguyn v 1 k n k =n n1 k1 = (n k + 1) k n, ta lun c ng thc n k1

Li gii. Tch ca k v n gi cho chng ta s dng quy tc nhn. n l s cc k-tp con k k ca [n] v k l s cch chn ra mt phn t t mt tp hp c k phn t. Do k n s cho k chng ta s N cc cp (e, S), trong e S [n] v |S| = k. Ta s m s cp trn theo cc
1 2

SV C nhn Khoa hc ti nng Ton hc K15, HKHTN - HQGHN. T y v sau, nu khng c gii thch g thm, ta s s dng k hiu nh trn. Vi n = 0 th [0] = .

17

18 cch khc : Vi mi phn t e ca [n], ta c n1 tp con c k phn t ca [n] cha e. Do N = n n1 . k1 k1 Mt khc, nu ta chn trc k 1 phn t ca S, sau chn e t n k + 1 phn t cn li n b sung vo S. Khi N = (n k + 1) k1 . Theo nguyn l m bng hai cch, ta c iu cn chng minh. Bi ton 3 (ng thc Pascal). Chng minh rng vi k, n l cc s nguyn v 1 lun c ng thc n+1 n n = + k k1 k k n, ta

Li gii. V tri ca ng thc cn chng minh l s tp con S c k phn t ca [n + 1]. V n phi l tng ca hai biu thc gi cho chng ta s dng quy tc cng. n v k1 tng ng k l s k-tp con v (k 1) tp con ca [n], trong khi tp hp ban u ca chng ta l [n + 1]. T ta xt hai trng hp sau vi mt phn t e bt k ca [n + 1] : 1. e S : S cc tp S tha mn s l 2. e S : S cc tp S tha mn l /
n k n k1

.
n k1

V vy s tp con c k phn t ca [n + 1] l

n k

. Ta c iu cn chng minh.

Bi ton 4. Chng minh rng vi mi s t nhin n, ta c


n

i=0

n i

= 2n
n 0

Li gii. t Sn l tp hp tt c cc tp con ca [n]. V trong Sn c n tp c 1 phn t, . . . , n tp c n phn t. Do ta c n 1


n

tp c 0 phn t,

|Sn | =

i=0

n i

Mt khc, vi mi phn t e v tp con S ca [n], ch c hai kh nng xy ra l e S v e S. / n Suy ra s cc tp con ca [n] l 2 . So snh vi ng thc trn, ta c iu cn chng minh. Ch : T kt qu s tp con ca [n] bng 2n , ta cn k hiu 2[n] l tp hp tt c cc tp con ca [n]. Tng qut hn, ta k hiu 2S l tp hp tt c cc tp con ca mt tp hp S bt k. Bi ton 5. Chng minh rng vi mi s t nhin n, ta c
n

(1)i
i=0

n i

=0

Li gii. Trc ht, ta s m s tp con c chn phn t ca [n]. S cc tp con ny l n 2k

19 c mt tp con c chn phn t ca [n], trc ht ta chn ra n 1 phn t c nh v chn tip mt tp con bt k ca n 1 phn t ny. Nu tp con chn ra c mt s chn phn t th ta c mt tp con c chn phn t ca [n], nu tp con chn ra c mt s l phn t th ta s b sung phn t cn li ca [n] vo tp con ny. Khi ta cng c mt tp con c chn phn t ca [n]. V vy s tp con c mt s chn phn t ca [n] cng bng s tp con ca [n 1] v bng 2n1 . V v s tp con ca [n] bng 2n nn s tp con c s chn phn t ca [n] bng s tp con c s l phn t ca [n]. T suy ra ng thc cn chng minh. Nhn xt : S xut hin (1)i trong s hng tng qut ca tng khin chng ta ngh n vic tch thnh hai tng nh : mt tng vi k chn v mt tng vi k l. Nu nh ng thc ta cn chng minh l ng (v hin nhin rng n ng!) th ta s suy ra rng s cc tp con c mt s chn phn t bng mt na s cc tp con ca [n] v bng 2n1 . Con s ny li chnh bng s tp con ca [n 1], t dn n cch chng minh trn. Nu suy ngh theo hng chng minh trc tip s tp con c mt s chn phn t bng s tp con c mt s l phn t th s tng i kh khn thc hin iu ny. Bi ton 6 (ng thc Vandermonde). Cho m, n, r l cc s t nhin vi r Chng minh rng r m+n m n = r i ri i=0 min{m, n}.

Li gii. Ta s phn hoch tp [m + n] thnh hai tp con A, B, trong |A| = m, |B| = n v m s tp con S c r phn t ca [m + n] bng cch xt |S A| v |S B| : Nu |S A| = i th |S B| = r i (v A B = ). Cho i chy t 0 n r v ly tng, ta s thu c s tp con S c r phn t ca [m + n]. ng thc c chng minh. Tng qut : Cho n, x, y, ki (i = 1, y) l cc s t nhin. Khi ta c ng thc
x

k1 ,...,ky =0

n k1

n n ky k2

n x

kj
j=1

(y + 1)n x

Bi ton 7. Chng minh rng vi mi s nguyn dng n, ta lun c


n

k
k=1

n k

=n

2n 1 n1

Li gii. Phn hoch [2n] thnh hai tp X, Y vi |X| = |Y | = n. Ta s m s N cc cp (e, A, B). Trong e A; A X, B Y ; |A| + |B| = n. 2 n t |A| = k, |B| = n k, k = 1, n. Ta c n nk = n cch chn A, B. T , vi mi k c k k 2 k n cch chn (e, A, B). V vy k n 2 n N = k k k=1 Mt khc, vi e bt k thuc [2n], mi t hp chp n 1 ca 2n 1 phn t ca [2n] \ {e} s

20 tng ng vi mt cp (e, A, B) tha mn bi. Do N =n Vy ta c ng thc cn chng minh. Bi ton 8. Chng minh rng vi mi s nguyn dng m, n v m
n

2n 1 n1 n, ta c

i=m

n i

i m

= 2nm

n m

Li gii. Xt cc cp (A, B) vi |A| = m, |B| = i v A B [n]. S cc cp ny l


n

i=m

n i

i m

Vi mi tp con A c m phn t ca [n], ta chn thm mt tp con bt k trong s n m phn n t ca [n] \ A b sung vo A to thnh B. Do s cc cp (A, B) l 2nm m . Bi ton 9. Chng minh rng nu k, n l cc s nguyn dng th
k

i=0

n+i i

n+k+1 k

Li gii. Ta vit li ng thc cn chng minh nh sau :


k

i=0

n+i n

n+k+1 n+1

Ta m s cc tp con c n + 1 phn t ca [n + k + 1] bng cch chia trng hp : C n tp n con c phn t ln nht l n + 1. C n+1 tp con c phn t ln nht l n + 2. . . C n+k n n tp con c phn t ln nht l n + k + 1. T cc trng hp trn, ta suy ra rng s tp con c n + 1 phn t ca [n + k + 1] bng k n+i n i=0 Mt khc, s tp con ny bng chng minh.
n+k+1 n+1

. Theo nguyn l m bng hai cch, ta c iu cn

Bi ton 10. Chng minh rng vi mi s nguyn dng n, ta c


n/2 i=0

2n2i

n i

ni i

2n n

Li gii. V phi ca ng thc cn chng minh l s tp con c n phn t ca [2n]. Ta s m s tp con ny bng cch phn hoch [2n] thnh hai tp con A, B vi |A| = |B| = n. Gi cc phn t ca A, B ln lt l a1 , a2 , . . . , an ; b1 , b2 , . . . , bn v ghp cp (aj , bj ), j = 1, n. Xt mt tp con S ca [2n] vi |S| = n. Ta gi mt phn t ca [2n] l tt nu n thuc S.

21 D thy rng vi mi tp con S c n phn t ca [2n], s cc cp (a, b) m a, b cng tt bng n s cp (a, b) m a, b cng khng tt. t s cp ny l i. R rng 0 i . 2 n ni cch chn ra i cp tt v i cp khng tt. Cn li n 2i cp, ta s chn t Ta c i i mi cp mt phn t v nh du phn t l tt. Nh vy ta chn c n phn t tt. Mt khc, d thy rng mi cch chn trn xc nh duy nht mt tp con c n phn t ca [2n]. V vy ta c
n/2 i=0

2n2i

n i

ni i

2n n

y chnh l ng thc cn chng minh.

Nhn xt : im c bit trong li gii trn l ghp cp cc phn t ca [2n] v s dng biu thc 2n2i chn ra ng n 2i phn t thay v mt nhm bt k trong s n 2i nh cng thc v s tp con quen thuc. Bng phng php tng t, chng ta c th gii quyt bi ton sau : Cho n l mt s nguyn dng, chng minh rng
n

2k
k=0

n k

nk
nk 2

2n + 1 n

Bi ton 11. Cho s nguyn n

1. Chng minh rng


n1

k=1

k k! = n! 1

Li gii. Xt bi ton sau : m s cc hon v ca [n] sao cho c t nht mt s i, (i = 1, n) m (i) = i. D thy rng s cc hon v nh vy l n! 1. Ta s m theo phn t i ca [n] nh nht m (i) = i. Vi mi k = 1, n 1, mt hon v nhn n k l phn t i nh nht m (i) = i c xy dng nh sau : n k 1 s u tin ca vn l 1, 2, . . . , n k 1. Tip theo , (n k) c th nhn k gi tr thuc tp {n k + 1, n k + 2, . . . , n}. k s cn li c th sp xp theo k! cch. V vy c k k! hon v tha mn iu kin trn vi mi k = 1, n 1. Cho k chy t 1 n n 1 v ly tng, ta s thu c s cc hon v cn tm l
n1

k=1

k k!

Vy ta c ng thc cn chng minh.

Bi ton 12 (IMO 1987). Gi pn (k) l s cc hon v ca [n] c ng k im c nh. Chng minh rng
n

k=0

k pn (k) = n!

22 Li gii. Ta s m tt c cc cp (x, s) vi s l mt hon v ca [n] v x l mt im c nh ca s. Vi mi phn t x, ta c (n 1)! hon v nhn x l im c nh. Suy ra s cc cp (x, s) l S=
x[n]

(n 1)! = n (n 1)! = n!

(1)

Mt khc, vi mi hon v s c ng k im c nh th ta c k cp (x, s). Do ta c


n

S=
k=0

k pn (k)

(2)

T hai ng thc (1) v (2), ta c iu cn chng minh.

Ch : Ta cng c th gii quyt bi ton trn bng cch chng minh ng thc k pn (k) = npn1 (k 1). Bi ton 13. S chia lp : H cc tp con khc rng A = {Ai | i I} ca tp X c gi Ai = X. l mt s chia lp tp X nu n tha mn iu kin Ai Aj = vi mi i = j v
iI

S Bell Bn l s tt c cc cch chia lp [n] (gi s B0 = 1). Chng minh rng


n

Bn+1 =
i=0

n Bi i

Li gii. Xt mt phn t e bt k ca [n + 1]. Ta s m s cc cch chia lp [n + 1] theo lc lng ca tp con ca [n + 1] cha e. Gi s e A vi |A| = i + 1 (i = 0, n). Ta thc hin chia lp [n + 1] theo cc bc sau : 1. B sung thm i phn t t n phn t cn li ca [n + 1] \ {e} : C 2. Chia lp n i phn t cn li : C Bni cch. V vy s cc cch chia lp [n + 1] bng
n n n n i

cch;

i=0

n Bni = i

i=0

n Bni = ni

i=0

n Bi i

Vy ng thc c chng minh.

Bi ton 14. Cho s nguyn dng n, K hiu n l s c nguyn dng ca n. Chng minh rng n n n + + + 1 + 2 + + n = 1 2 n Li gii. Xt trong mt phng ta Oxy. Ta gi mt im l im nguyn nu ta ca n l cc s nguyn. i Xt mt nhnh hyperbol (Hi ) c phng trnh y = x , x > 0. V phi ca ng thc cn chng minh l s cc im nguyn nm trong min R gii hn bi gc phn t th nht v (Hn ) (c th nm trn (Hn )).

23 Vi mi i = 1, n. Ta c i l s im nguyn nm trn (Hi ). Mt khc, d thy rng mi im nguyn nm trn (Hi ) u thuc R. Do cc nhnh hyperbol (Hi ), i = 1, n i qua tt c cc im nguyn thuc R. V vy s cc im nguyn thuc R chnh bng T cc nhn xt trn, ta c ng thc cn chng minh.
n

i .

i=1

Bi ton 15. Mt h F cc tp con ca [n] c gi l mt phn chui (antichain, cn gi l i xch hoc h Sperner ) nu khng c mt tp no ca F cha mt tp khc trong h F. t ak l s cc tp c k phn t trong F, k = 0, n. Cc kt qu v phn chui thng rt p v ng mt vai tr quan trng trong ti u t hp. Di y l mt s kt qu trong s : (a) (Bt ng thc Lubell Yamamoto Meshalkin)
n ak
n k=0 ( k )

1; tp.

(b) (nh l Sperner) Phn chui ln nht ca [n] cha

n n/2

Li gii. (a) Xt chui tp con = S0 S1 S2 Sn = [n], trong |Si | = i vi i = 0, n. D thy rng mi chui nh trn tng ng vi mt hon v ca [n]. V vy c ng n! chui nh vy. Mt khc, vi mi A F v |A| = k, c ng k!(n k)! chui cha A (ti sao?). Ch rng khng mt chui no cha hai tp con trong F. Suy ra s cc chui cha cc tp trong F l
n

k=0

ak k!(n k)!

Li c s cc chui nh trn khng vt qu tng s tt c cc chui ca [n]. Do


n

k=0

ak k!(n k)!

n!

Chia c hai v ca bt ng thc trn cho n!, ta c iu cn chng minh. (b) T cu (a), kt hp vi bt ng thc quen thuc 1
n n/2 n n n k n n/2

, k = 0, n. Ta suy ra

ak
k=0 k=0

ak
n k

Do |F| =

ak
k=0

n n/2
n n/2

Mt khc, xt F l h cc n/2-tp con ca [n] th F l mt phn chui v |F | = Ta c iu cn chng minh.

Bi ton 16 (nh l Erds Ko Rado). Mt h F cc tp con ca [n] c gi l mt o k-h giao nhau nu tt c cc tp trong F u c k phn t v hai tp bt k trong F u c phn t chung. Chng minh rng k-h giao nhau ln nht ca [n] cha n1 tp vi n 2k. k1 Li gii. Xt mt a gic u n cnh P . Mt cung di k gm k + 1 nh lin tip ca P v k cnh nm gia chng. Ta c b sau :

24 B . Gi s rng n 2k v ta c t cung phn bit A1 , A2 , . . . , At c di k. Bit rng hai cung bt k c mt cnh chung. Khi t k. Ch rng mi nh ca P l u mt ca ti a mt cung. Tht vy, nu hai cung Ai , Aj c chung mt u mt. Khi hai cung Ai , Aj nm v hai na a gic. V v n 2k nn Ai , Aj khng th c cnh chung, v l. Xt cung A1 . V mi cung Ai (i 2) u c cnh chung vi A1 nn mt trong hai u mt ca Ai nm trong A1 . M A1 c k 1 im trong v cc u mt ny phn bit nh ta va ch ra. Do c ti a k 1 cung khc. Suy ra s cung ti a l k

Quay tr li vi bi ton ca chng ta. Ta s m s N cc cp (A, ). Trong A F, l mt hon v vng quanh = (a1 , a2 , . . . , an ) ca [n] v cc phn t ca A l k s lin tip ca . Ta vit cc s a1 , a2 , . . . , an ln lt theo chiu kim ng h ln cc cnh ca P . T b trn, vi mi hon v vng quanh , c ti a k tp trong F c cc phn t l k s lin tip ca . Li c s hon v vng quanh bng (n 1)!. V vy ta c N k(n 1)! (1)

Vi mi A F, ta c k!(n k)! hon v vng quanh ca [n]. Do s N cc cp (A, ) bng |F| k!(n k)! T (1) v (2) ta suy ra |F| k(n 1)! = k!(n k)! n1 k1 (2)

Xt e l mt phn t bt k ca [n]. D thy rng c ng n1 tp con c k phn t ca [n] k1 cha e v cc tp con ny to thnh mt k-h giao nhau ca [n]. Vy ta c iu cn chng minh.

2.2. Phng php m bng hai cch v th hu hn


Chng ta s xt cc bi ton ca l thuyt th c gii quyt bng phng php m bng hai cch : Bi ton 17. Gi s rng th G = (V, E) c |V | = n v khng cha mt chu trnh di 4 (k hiu C4 ). Tm s cnh ln nht c th ca G. Li gii. t d(u) l bc ca nh u; S l tp cc cp (u, {v, w}), trong u k vi v v w, v = w. Vi mi nh u ca G, s cc cp nh trn l d(u) . V vy ta c 2 |S| = d(u) 2

uV

Mt khc, vi mi cp {v, w} ch tn ti nhiu nht mt nh u V sao cho (u, {v, w}) S n . Do (v G khng cha C4 ). Suy ra |S| 2
uV

d(u) 2

n 2

25 Tng ng vi d(u)2
uV

n(n 1) +

d(u)
uV

p dng bt ng thc Cauchy-Schwarz, ta c


2

n (n 1) + n Vi ch rng
uV

d(u)
uV

n
uV

d(u)

2 uV

d(u)

d(u) = 2|E|, ta c n2 (n 1) + 2n|E| 4|E|2

hay |E|2 n n2 (n 1) |E| 2 4 n 1 + 4n 3 4 Bi ton 18 (Cng thc Cayley). Chng minh rng c ng nn2 cy c th to thnh t n nh phn bit. Li gii. Ta s m s N cc dy cc cnh c hng c th thm vo n nh trn to thnh mt cy c gc. Cc cnh c hng c thm vo c hng sao cho vi mi nh V th cc cnh thuc ng ni gc R ca cy vi V c hng t R n V . Gi Tn l s cc cy biu din khc nhau ca th y Kn . Vi mi cy trong s Tn cy cha c gc, ta chn ra mt trong s n nh lm gc v chn mt trong s (n 1)! hon v ca n 1 cnh ca cy to thnh mt dy cnh c hng (ch rng hng ca mi cnh c xc nh duy nht v gia hai nh bt k ca mt cy ch c ng mt ng ni duy nht). Suy ra N = Tn n (n 1)! = n! Tn Ta s xy dng mt cy c gc nh trn bng cch thm tng cnh mt vo n nh cho. Gi s rng ta thm n k cnh (k = 2, n) vo K n . Khi ta thu c mt bi c gc gm k cy. C n(k 1) cch thm vo mt cnh : nh u ca n l mt trong s n nh v nh cui ca n l mt trong s k 1 gc ca k 1 cy khng cha nh u. V vy ta c
n

Gii bt phng trnh bc 2 trn, ta suy ra |E|

N =

k=2

n(k 1) = nn1 (n 1)! = nn2 n!

T hai ng thc trn ta suy ra n! Tn = nn2 n! hay Tn = nn2 .

Mt s bi ton c th c gii quyt bng cch s dng l thuyt th kt hp vi phng php m bng hai cch. Ta xt cc v d sau :

26 Bi ton 19. Cho cc s nguyn 0 n 2 = k n. Chng minh rng

k nk + k(n k) + 2 2

Li gii. Xt th y Kn . Ta chn ra k nh ty ca Kn v xp cc cnh ca Kn vo 3 nhm :


k 2

cnh ca th con y to t k nh chn; cnh ca th con y to t n k nh cn li;

nk 2

k(n k) cnh ni gia cc nh ca 2 th con trn. R rng rng tt c cc cnh ca th ban u phi thuc vo mt v ch mt trong 3 trng hp trn. V vy tng s cc cnh ca 3 nhm trn bng s cnh ca th ban u hay ta c n 2 = k nk + k(n k) + 2 2

y chnh l ng thc cn chng minh.

Bi ton 20 (B Sperner). Cho mt tam gic V1 V2 V3 c phn hoch thnh cc tam gic nh sao cho khng c mt nh no ca cc tam gic nh nm trn cnh mt tam gic nh khc (c th nm trn cnh ca tam gic ban u). Gi s rng cc nh c nh s 1, 2, 3 sao cho Vi c nh s i v ch hai s i, j c nh s cho cc nh nm trn cnh Vi Vj (i, j = 1, 3, i = j). Chng minh rng tn ti mt tam gic nh m 3 nh ca n c nh s bi c 3 s 1, 2, 3. Li gii. Gi tam gic nh m 3 nh ca n c nh s bi c 3 s 1, 2, 3 l tam gic tt. Ta s chng minh mt kt qu mnh hn : S cc tam gic tt l mt s l.
3 1 3 3 1 1 3 1 2 3 2 2

Xt th i ngu3 G ca cch phn hoch tam gic V1 V2 V3 nhng khng ly tt c cc cnh ca n. Hai nh ca G c ni vi nhau bi mt cnh khi v ch khi hai min mt phng
th i ngu ca mt th phng G l mt th G trong mi nh tng ng vi mt min mt phng ca th G v cc cnh c ni gia hai nh tng ng vi hai min k nhau.
3

27 tng ng vi hai nh c cnh chung m hai u mt ca n c nh s bi 1 v 2. D thy rng vi mi tam gic tt th bc ca nh tng ng vi n bng 1, bc bng 2 vi cc tam gic m 3 nh ca n c nh s bi mt trong hai s 1, 2 v bc bng 0 vi cc tam gic khng c ng thi hai nh c nh s 1, 2. Khi ch cc tam gic tt mi tng ng vi mt nh bc l. Mt khc, trn cnh V1 V2 , ta c mt s l cc cnh ca cc tam gic nh c dng 1 2. V v cc cnh c dng 1 2 ch xut hin trn cnh V1 V2 nn ta suy ra bc ca nh tng ng vi phn mt phng nm ngoi tam gic ban u c bc l. T kt qu quen thuc : s cc nh bc l ca mt th hu hn l mt s chn, ta suy ra rng s cc tam gic tt l mt s l.

2.3. Phng php ma trn lin thuc


Trc ht chng ta s n vi mt cch pht biu khc ca nguyn l m bng hai cch : Nguyn l 2 (Nguyn l Fubini). Cho hai tp hu hn R, C v S R C. Nu (p, q) S, ta ni rng p v q lin thuc vi nhau. t rp l s cc phn t lin thuc vi p R v cq l s cc phn t lin thuc vi q C. Khi
pR

rp = |S| =

cq
qC

minh ha cho tp S, ta s dng n ma trn lin thuc ca S : nh ngha 1. Ma trn M = (apq ) l ma trn m cc ct v cc hng ca n tng ng c nh s theo cc phn t ca R v C vi 1 nu (p, q) S apq = 0 nu (p, q) S /

Khi rp l tng ca cc phn t ct p, cq l tng ca cc phn t ct q v |S| l tng tt c cc phn t ca ma trn M . Vi mt s bi ton, phng php ma trn lin thuc s gip chng ta d hnh dung hn v cu trc ca bi ton, t a ra li gii cho bi ton. Ta xt cc v d sau : Bi ton 21 (IMO 1998). Trong mt cuc thi c a th sinh v b gim kho, trong b 3 v l s nguyn l. Mi gim kho nh gi t hoc trt. Gi s rng vi hai gim kho bt k, h nh gi ging nhau vi ti a k th sinh. Chng minh rng k a b1 2b

Li gii. Xt ma trn lin thuc b a vi cc hng c nh s theo cc gim kho v cc ct c nh s theo cc th sinh. Phn t tng ng ca ma trn nhn gi tr bng 1 nu gim kho nh gi th sinh l t v nhn gi tr bng 0 nu ngc li.

28 t T l tp hp cc cp cc s 0 hoc 1 trong cng mt ct. V hai gim kho nh gi ging nhau nhiu nht l k th sinh nn vi hai hng bt k, c nhiu nht k cp thuc T . Do |T | k b 2 = kb(b 1) 2
p 2

Vi mi ct trong ma trn, gi s c p s 0 v q s 1. Khi s c ng M p + q = b l nn ta c bt ng thc sau (bn c t chng minh) p q + 2 2 V v c a ct nn ta c |T |


a(b1)2 . 4

q 2

cp thuc T .

(b 1)2 4

Vy ta c |T | kb(b 1) 2

a(b 1)2 4 Suy ra


a(b1) 2

kb hay

k a

b1 . 2b

Nhn xt : Nh ni trn, phng php ma trn lin thuc kt hp vi nguyn l Fubini cho chng ta mt hnh dung d dng hn v cu trc bi ton. Ta vn c th gii chng m khng cn s dng n ma trn lin thuc. Chng hn nh v d trn, chng ta c th gii quyt bng cch xt cc b (A, B, x), trong x l mt th sinh v A, B l hai gim kho nh gi th sinh x ging nhau. Bi ton 22 (IMC 2002). C 200 th sinh tham gia trong mt cuc thi. H c ngh gii 6 bi ton. Bit rng mi bi ton c gii ng bi t nht 120 th sinh. Chng minh rng phi c 2 th sinh m vi mi bi ton, c t nht mt trong hai th sinh ny gii c bi ton . Li gii. Xt ma trn lin thuc 6 200, trong mi hng i din cho mt bi ton v mi ct i din cho mt th sinh tham gia cuc thi. Mi phn t ca ma trn nhn gi tr 1 nu th sinh tng ng vi ct khng gii c bi ton tng ng vi hng v 0 nu ngc li. t N l s cc cp cc s 1 cng hng. Gi s rng vi hai th sinh bt k, tn ti mt bi ton m c 2 u khng gii c. Khi , vi hai ct bt k, c t nht mt cp s 1 trong hai ct ny nm cng mt hng. M ta c ng 200 cp ct. Do 2 N 200 2 = 19 900
80 2

(1) cp s

Mt khc, theo gi thit th c ti a 80 s 1 mi hng. V v vy c nhiu nht 1 cng hng. V v c 6 hng nn suy ra N T (1) v (2), ta suy ra 19 900 T l sai v bi ton c chng minh. 6 80 2 = 18 960

(2)

18 960, v l. V vy iu gi s ban u ca chng ta

29 Bi ton 23. S Turan T (n, k, l) (l k n) l s nh nht cc tp con c l phn t ca [n] sao cho vi mi tp con c k phn t ca [n] u cha t nht mt tp con trn. Chng minh rng vi mi s nguyn dng l k n, ta c T (n, k, l)
n l k l

Li gii. t F l h nh nht cc tp con l phn t ca [n] sao cho vi mi tp con k phn t ca [n] u cha t nht mt phn t ca F. Xt ma trn lin thuc M = (mA,B ) vi cc c nh s theo cc tp con A ca F, cc ct c nh s theo cc tp con k phn t B ca [n] v mA,B = 1 khi v ch khi A B, trong cc trng hp khc mA,B = 0. t rA l s cc s 1 trong hng A v cB l s cc s 1 trong ct B. Theo gi thit th cB 1 vi mi B. Mt khc, rA l s cc tp con k phn t cha tp con l phn t A. Suy ra rA = nl kl vi mi A F. V vy |F| Do T (n, k, l) = |F| nl kl =
AF

rA =
B[n]

cB

n k
n l k l

n k nl kl

ng thc cui c gi c th t chng minh, tng t v d 8.

Bi ton 24. Cho s nguyn dng n, K hiu n l s c nguyn dng ca n v n l tng cc c nguyn dng ca n. Chng minh rng n n n + + + ; 1 2 n n n n +2 + + n . (b) 1 + 2 + + n = 1 2 n (a) 1 + 2 + + n = Li gii. chng minh ng thc (a), ngoi phng php s dng li im nguyn nh bi 12, ta cn c th s dng ma trn lin thuc nh di y : Xt ma trn M = (mij )nn vi mij = 1 khi v ch khi j chia ht cho i, trong cc trng hp khc, mij = 0. D thy rng s cc s 1 ct j bng j v s cc s 1 hng i bng n . T theo nguyn i l Fubini, ng thc (a) c chng minh. gii quyt cu (b), ta s thay i cch nh s cc phn t ca M nh sau : mij = i khi v . ch khi j . i. Khi tng cc s ct j s bng . d = j
d|j

V tng cc s hng i bng i

n i

. T ng thc (b) c chng minh.

Ch : T ng thc (a), ta c th suy ra bt ng thc sau |n ln n| < 1

30 Trong n l trung bnh cng ca cc s 1 , 2 , . . . , n , tc l n = 1 + 2 + + n n x vi mi s thc x, ta suy ra

Chng minh. T bt ng thc x 1 < x

Hn 1 < n < Hn Trong Hn l s iu ha th n : Hn =


n i=1 1 . i

Ta s tm cch nh gi Hn vi ln n.

n
1 ,x x

Da vo hnh v trn, so snh din tch ca hnh gii hn bi cc ng y = 0, y = 1, x = n v tng din tch ca cc hnh ch nht, ta thy rng
n

Hn 1 = v 1 Hn = n Do ln n + V vy ln n 1 < Hn 1

i=2

1 < i

n 1

1 dx = ln n x

n1

i=1

1 > i

n 1

1 dx = ln n x

1 < Hn < ln n + 1 n

1 < Hn 1 < n < Hn < ln n + 1 n

T suy ra bt ng thc cn chng minh.

3. Cc bi tp ngh
Bi tp 1. Trong mt phng ta trc chun Oxy, xt tp M nhng im c ta (x, y) vi x, y nguyn, x, y = 1, 12. Mi im ca M c t bi mt trong 3 mu xanh, , vng. Chng minh rng tn ti mt hnh ch nht c cc cnh song song vi cc trc ta , cc nh thuc M v c t bi cng mt mu. Bi tp 2 (Bulgaria MO 2006). Mt quc gia c 16 thnh ph v c 36 tuyn bay ni gia chng. Chng minh rng ta c th t chc mt chuyn bay vng quanh gia cc thnh ph.

31 Bi tp 3. Gi s rng cc cnh ca th K6 c t bi hai mu. Khi tn ti hai tam gic m c 3 cnh ca mi tam gic c t bi cng mt mu. Bi tp 4. Gi s k, n l hai s nguyn dng v S l tp hp n im trn mt phng tha mn tnh cht 3 im bt k ca S u khng thng hng, v vi mi im P S c t nht k im phn bit ca S cch u P . Chng minh rng k< 1 + 2n 2

Bi tp 5 (IMO Shortlist 1986). Cho 5 s c 100 ch s c to thnh t cc ch s 1, 2. Ta xp cc s thng nhau theo cc hng n v, chc, trm,. . . Bit rng hai s bt k trong 5 s u c t nht r hng ging nhau v mi hng sau khi xp u c hai ch s 1, 2. Chng minh rng 40 r 60. Bi tp 6. Vi cc s nguyn n k 0, t n l s cc hon v ca n phn t c ng k k xch. Ni cch khc, n l s cch xp n ngi phn bit vo k chic bn trn v khng c bn k no trng. n c gi l s Stirling loi mt. Chng minh rng k (a)
n k=1 n k

= n!; k 2, 1, 1,
n n k

(b) Vi n (c) Vi n (d) Vi n

=
n k

n1 k1

+ (n 1)

n1 k

(1)k

= 0;

k=1 n+1 2

= n!Hn . Trong Hn l s iu ha th n (xem thm v d 23)

Bi tp 7. Cho A1 , A2 , . . . , An l cc tp hp c r phn t v X l giao ca chng. Nu |Ai Aj | k vi mi i = j, khi |X| r2 n r + k(n 1)

Gi : Vi mi x X, t d(x) l s cc tp Ai (i = 1, n) m x Ai . Hy chng minh cc kt qu sau: (a)


xAi n

d(x) =
j=1 n

|Ai Aj |; d(x)2 .
xX

(b)

d(x) =
i=1 xAi

T tm cc nh gi thch hp suy ra kt qu ca bi ton. Bi tp 8. Cho X l mt tp hp c n phn t v A1 , A2 , . . . , An l cc tp con ca X sao n cho s phn t trung bnh ca n tp con t nht l w . Gi s rng n 2w2 . Chng minh rng tn ti i = j sao cho n |Ai Aj | 2w2

32

Ti liu tham kho


[1] Trn Nam Dng, K thut m bng hai cch v ng dng trong gii ton, K yu Hi ngh Khoa hc, Cc chuyn chuyn Ton Bi dng hc sinh gii THPT, Nam nh, thng 11/2010. [2] Stasys Jukna, Extremal Combinatorics: With Applications in Computer Science, Second Edition, Springer, 2001. [3] Martin Aigner, Gnter M. Ziegler, Proofs from THE BOOK, Fourth Edition, Springer, u 2010. [4] Arthur T. Benjamin, Jennifer J. Quinn, Proofs that Really Count: The Art of Combinatorial Proof, The Mathematical Association of America, 2003. [5] Titu Andreescu, Zuming Feng, 102 Combinatorial Problems, Birkhuser, 2003. a [6] Yufei Zhao, Counting in Two Ways, MOP 2007 Black Group. http://web.mit.edu/yufeiz/www/olympiad/doublecounting_mop.pdf [7] Carl G. Wagner, Basic Combinatorics, 2005. http://www.math.utk.edu/wagner/papers/comb.pdf [8] Cc din n Ton hc : Din n Art of Problem Solving (Mathlinks) http://www.artofproblemsolving.com/Forum/index.php Din n MathScope http://forum.mathscope.org/index.php

PHNG PHP XY D NG M HNH TRONG GI I TON T H P


L Phc L1

Song nh l mt cng c mnh gii nhiu bi ton chng minh v bi ton m trong t hp. tng chnh ca phng php ny chnh l thay i cch tip cn trong bi bng mt con ng, mt cch nhn khc c cc c im tng ng vi gi thit ban u m vi n, ta c th d dng x l hn. Hai trong cc t tng dng song nh thng gp chnh l vic xy dng bng v xy dng th trong mt phng ta . Vi cch dng bng v dng th gii Ton t hp, ta c th tip cn vn trc quan hn v cng thm mt s phng php m khc, ta hon ton c th x l c nhiu bi ton kh, thm ch khng gii c bng cch thng thng. Di y, chng ta s tm hiu mt s bi ton v tng xy dng mt bng hoc mt th trong mt phng ta tng ng gii quyt chng (nn phn bit vi mt s bi ton m cu hi t ra trc tip vi mt bng c sn no ).

1. Mt s vn v xy dng m hnh trong gii ton t hp


Vic xy dng m hnh y gip chng ta hnh dung vn mt cch tng minh hn v vic m, tnh ton y c th kim nghim trc quan hn. Chng hn, ta xt bi ton sau y: Bi ton 1. C bao nhiu cch chia tp hp S c n phn t thnh hai tp con (c tnh tp rng) sao cho hp ca chng bng S. Ta thy rng vn t ra y kh tng qut v cc tng m bng truy hi, chng minh bng quy np xut hin u tin trong trng hp ny. Tuy nhin, nu chng ta th v mt m hnh ra hnh dung th vn c th sng t hn:

x Ta c th biu din hai tp A, B nh trn bi cc hnh trn v cng vic cn lm l ng vi mi phn t x S, xp n vo trong cc hnh trn . R rng ta c 4 cch xp:
1

Sinh vin i hc FPT thnh ph H Ch Minh.

33

34 x thuc A nhng khng thuc B; x thuc B nhng khng thuc A; x thuc c A v B (nm trong phn giao); x khng thuc c A v B. Nhng v yu cu ca bi ton l hp ca hai tp con l S nn khng th tnh trng hp th 4 trn c. Do , ng vi mi phn t, ta c ng 3 cch xp vo hai tp hp. n 1 n +1 T y, ta c th tnh ra p s ca bi ton l 3 2 + 1 = 3 2 . Gii thch thm v kt qu ny, ta thy rng c mt vn cn gii quyt khi m l cc trng hp b trng nhau. Nu ban u cho sn hai tp con A v B ri th kt qu s l 3n r rng. Tuy nhin, yu cu y l chia tp S ra thnh hai tp con, nh th th vic chia y c tnh i xng gia A v B (tc l cch m x thuc A nhng khng thuc B, x thuc B nhng khng thuc A trn l ging nhau). Ch thm c mt trng hp c bit l khi A = B th buc phi c A = B = S nn ch c mt cch chia. B trng hp ra, chia i s trng hp ri li cng n vo th s c s cch chia cn tm. Cng thc y hon ton c th kim tra vi cc gi tr n nh. Da vo phn tch trn, cc bn hy th gii quyt bi ton sau: Bi ton 2. C bao nhiu cch phn hoch tp hp S gm n phn t thnh hai tp con? (tp hp S phn hoch thnh hai tp hp A v B khi A B = , A B = S). C mt bi ton kh th v v vic chia tp hp ny l: Bi ton 3. Cho tp hp S = {1, 2, . . . , n} l tp hp n s nguyn dng u tin. a) Hy tm s cch chia S thnh 3 tp con A, B, C sau cho AB = , B C = , C A = v A B C = . b) Hy tm s cc b ba cc tp con A, B, C tha mn A B C = S v B C = . c) Hy tm s cc b bn cc tp con A, B, C, D tha mn A B C D v B C D = . Chng ta li xem xt tip bi ton sau: Bi ton 4. C bao nhiu tp con ca tp hp gm 2n s nguyn dng u tin sao cho trong khng tn ti hai s a, b m a + b = 2n + 1? Bi ton ny c th s dng cng thc truy hi x l nhng lp lun hi rc ri. Ta c th chuyn v m hnh sau hnh dung:

1 2n

2 2n 1

3 2n 2

n n+1

35 n y th vn qu r rng, cng tng t nh bi trn v khng cn gii thch g thm, cc bn c th t gii y c nu bit kt qu l 3n . Vy nu thay iu kin trn thnh |a b| = n th sao? Xin mi cc bn t xy dng m hnh gii quyt. Cc p s c kim chng bng nhng trng hp nh. Nhc n bi ny, chng ta s nh n bi 5 trong k thi VMO 2009. thc s l mt bi ton kh v cch xy dng bng m s trng hp tha mn bi qu l con ng duy nht c th p dng c trong tnh hung ny. Ta hy xem li bi v li gii chi tit cho bi ton . Bi ton 5. Cho s nguyn dng n. K hiu T l tp hp 2n s nguyn dng u tin. Hi c bao nhiu tp con S ca T tha mn tnh cht: trong S khng tn ti hai s a, b m |a b| {1; n}? Bi ton pht biu rt n gin v d hiu nhng ng sau l c mt vn . Bi ton ny khc vi bi m chng ta va pht biu ch ngi ta loi thm mt trng hp |a b| = 1 (nu ch cho ring yu cu ny th ch cn m bng truy hi ra cng thc tng t dy Fibonacci quen thuc), nh th th vic sp xp theo kiu nh trn s a n vn l khng c hai no cnh no c chn (cng ct hoc lin k). Tuy nhin, vn cn trng hp a = n + 1, b = n cha x l c nu nh hiu theo kiu trn. Li gii. ng vi mt s nguyn dng n, t 2n s nguyn dng {1, 2, 3, . . . , 2n 1, 2n} vo mt bng 2 n nh sau: 1 n 2 n+1 3 n+3 n1 2n 1 n 2n

Ta s xc nh tng s cc cch chn mt s vung t bng ny (c th l khng chn s no) tha mn hai iu kin di y: (i) Hai vung k nhau (tc l hai cha hai s lin nhau hoc hai s cch nhau n n v) s khng c chn ng thi. (ii) Hai cha s n v n + 1 cng s khng c chn ng thi. R rng s cch chn ny chnh l s tp hp con S ca tp hp T cn tm, t s cch chn ny l S(n). Ta s xt thm s cch chn mt s vung A(n), B(n), C(n) t cc bng (A), (B), (C) cng tha iu kin (i) nhng c mt s thay i nh sau: A(n) : bng (A) ny cha cc s t 1 n 2n v hai vung cha s n, n + 1 vn c chn ng thi. B(n) : bng (B) ny khng c cha vung gc ca bng, chng hn l vung cha s n + 1. C(n) : bng (C) ny khng tnh hai vung gc ca bng cha hai s lin tip l n, n + 1.

36 Ta s chng minh cc quan h sau: (1) A(n) = S(n) + C(n 2); (2) A(n) = A(n 1) + 2B(n 1); (3) B(n) = A(n 1) + B(n 1); (4) B(n) = C(n) + C(n 1); (5) C(n) = B(n 1) + B(n 2) + C(n 2). Tht vy: (1) tnh s cch chn A(n) cc vung t bng (A), ta chia lm 2 trng hp: Cc cha n v n + 1 khng c chn ng thi. S cch chn ny chnh l S(n). Cc cha n v n + 1 c chn ng thi. Khi , cc cha s 1, n + 2, 2n, n 1 khng c chn, bng (A) c dng nh sau: 2 3 n+3 2n 1

Bng ny cha 2n 4 nn c th xem n tng ng vi bng: 1 2 n+2 3 n+3 n3 2n 3 2n 2

R rng, s cch chn cc vung t bng ny v tha iu kin (i) chnh l C(n2). Do cc trng hp trn l ri nhau nn A(n) = S(n) + C(n 2). (2) Cng tnh s cch chn A(n) cc vung t bng (A), ta chia lm 3 trng hp: Cc vung cha 1 v n + 1 khng c chn. bng (A) c dng: 2 n+2 3 n+3 n1 2n 1 n 2n

Ta thy bng ny ch cha 2n 2 nn c th xem tng ng vi bng sau

37 1 n 2 n+1 3 n+2 n2 2n 1 n1 2n

Chn vung cha s 1. Khi vung cha s n + 1 v 2 khng c chn, bng c dng 3 n+2 n+3 n1 2n 1 n 2n

S cch chn cc vung t bng ny v tha iu kin (i) chnh l A(n 1).

Ta thy bng ny cng c th xem tng ng nh bng sau: 2 n n+1 3 n+2 n2 2n 2 n1 2n 1

Chn vung cha s n + 1 Tng t trn, s cch chn trong trng hp ny l B(n 1). Cc trng hp trn l ri nhau nn A(n) = A(n 1) + 2B(n 1). Cc kt qu (3), (4) v (5) trn c chng minh tng t. T , ta s chng minh cng thc truy hi ca S(n) : S(n + 3) = S(n + 2) + 3S(n + 1) + S(n). 1 Tht vy, t (2) ta suy ra B(n1) = 2 [A(n) A(n 1)], thay vo (3) ta c 1 [A(n + 1) A(n)] = 2 1 A(n 1) + 2 [A(n) A(n 1)] hay A(n + 1) = 2A(n) + A(n 1). T suy ra A(n + 3) = 2A(n + 2) + A(n + 1) = A(n + 2) + 2A(n + 1) + A(n) + A(n + 1) hay tng ng vi A(n + 3) = A(n + 2) + 3A(n + 1) + A(n) Thay (4) vo (5), ta c C(n) = [C(n 1) + C(n 2)] + [C(n 2) + C(n 3)] + C(n 2) = C(n 1) + 3C(n 2) + C(n 3), n 3 T (1), (6) v (7), ta c c cng thc truy hi ca S(n) : S(n + 3) = S(n + 2) + 3S(n + 1) + S(n), n 0 (7) (6)

S cch chn cc vung t bng ny v tha iu kin (i) chnh l B(n 1).

Mt khc, d thy rng S(0) = 1, S(1) = 3, S(2) = 6. T ta s xc nh cng thc tng qut ca S(n). Dy {S(n)} c phng trnh c trng 3 = 2 + 3 + 1 = 1, = 1 2. T ta suy ra s tp con cn tm l n n 3 + 2 1 + 2 + 3 2 1 2 2(1)n , n 0 4

38 Li gii tuy di v rc ri nhng t , chng ta vn c th hc c nhiu iu cn thit. Trn thc t, nhiu bi ton c th c s chuyn i qua li gia vic pht biu dng tp hp nh trn v vic xy dng m hnh. Chng hn, ta bt u bng bi ton quen thuc sau: Bi ton 6. Chng minh rng khng th lt mt nn nh 10 10 bng cc vin gch 1 4. Cch gii ca bi ny l t mu cc c dng (l, l) trong bng, nh th th t c 25 . Tuy nhin, mi vin gch 1 4 nh trn phi chim 2 c t mu, tc l s c t mu nm trong phi chn. iu mu thun ny cho thy ta khng th lt gch c. Bi ton ny c th thay s 10 (s chn nhng khng chia ht cho 4) thnh cc s tng t nh 50, 2010, 2014,. . . th vn cho cu tr li tng t vi cch gii tng t. Ta th thay i bi ton theo kiu ni cc im trong mt phng thu c bi ton sau: Bi ton 7. Trong mt phng, cho tp hp A gm 20102 im phn bit c nh s t 1 n 20102 sao cho ba im bt k no trong chng cng khng thng hng. Mt t gic (li hoc lm) c gi l p nu cc nh ca n thuc A v c nh s bng 4 s tha mn mt trong hai iu kin sau: (i) l 4 s t nhin cch nhau 2010 n v. (ii) l 4 s t nhin lin tip v nu trong c cha s chia ht cho 2010 th s phi l ln nht. Ni tt c cc im thuc tp hp A li vi nhau sao cho im no thuc A cng thuc ng mt t gic. Tm s ln nht t gic p c to thnh. R rng cch ni cc im nh trn c th c m phng bng 4 s nm trn mt vin gch 1 4 nh trn. Ta c th gii chi tit nh sau: Li gii. Xt mt bng vung gm 2010 2010 vung con c in cc s theo th t t trn xung v tri sang phi. Trc ht, ta s chng minh rng khng th chia tt c 20102 im cho thnh cc t gic p c. R rng 4 s trn cc nh ca cc t gic p tng ng vi 4 s b che i trn bng vung khi t mt mnh ba hnh ch nht kch thc 1 4 vo . Ta s chng minh rng khng th che ht ton b bng vung ny bng cc hnh ch nht 1 4. Tht vy, ta t mu cc vung nm ct chn v hng chn. Do bng c 20102 vung nn 2 s vung b t mu bng 2010 = 1010025 l s l. 4 Gi s ngc li rng ta c th lp kn c c bng vung bng cc mnh ba. Khi , mi mnh ba s che i hoc hai vung hoc khng c vung no ca bng vung, tc l lun c mt s chn vung b che i. Do , s vung b che i trn bng l mt s chn. T ta thy c mu thun. Vy khng th che ht bng vung ny bng cc hnh ch nht 1 4 c. Gi k l s t gic p ln nht cn tm th k < 1010025 k 1010024. Ta s chng minh k = 1010024 bng cch ch ra cch dng cc mnh ba che kn bng vung. Tht vy, chia bng vung thnh hai phn:

39 Phn 1 gm 2008 ct u, ta xp cc mnh ba theo cc hng, mi hng c ng 502 mnh ba. Khi , ta s c th che kn ht phn 1 bi cc mnh ba. Phn 2 gm 2 ct cui, ta xp ni tip cc mnh ba t trn xung di th cui cng s cn li mt vung 2 2 gc di cng ca bng. Nh vy, ta dng 1010024 mnh ba che c ti a 20102 4 vung ca bng. T , ta thy, s t gic p ln nht cn tm l k = 1010024. Cc bn th tm hiu bi ton sau: Bi ton 8. C bao nhiu cch lt mt hnh ch nht kch thc 2 n bi cc vin gch: hnh ch I (hnh ch nht kch thc 1 2) v hnh ch L (hnh vung 2 2 b i mt )? Nu thay hnh ch nht 2 n bng hnh ch nht 3 n th s cch lt thu c s l bao nhiu? Qua , ta thy rng vic chuyn i cc m hnh, t cc im, cc hnh trong mt phng n cc s trong mt tp hp v ngc li gip cho bi ton c mt dng v mi kh th v. Chng ta th nhc n mt bi ton trong k thi IMO 1983: Bi ton 9. Ngi ta t tt c cc im nm trn cnh ca tam gic u ABC bi hai mu xanh v . Hi vi mi cch t mu nh th, c lun tn ti mt tam gic vung c ba nh c t cng mu hay khng? Li gii. Bi ton ny c th gii quyt khng qu kh khn bng cch xt cc im chia cc cnh ca tam gic ABC theo t s 2 : 1, c kh nhiu tam gic vung c to thnh t 6 im ny v nh vy m ta c th p dng nguyn l Dirichlet gii quyt bi ton. Tuy nhin, nu chng ta thay i cch pht biu thnh ta trong mt phng th bi ton s th v hn nhiu. Trong mt phng ta Oxy, mt tam gic u ABC c ta nh l A 0; 3 , B(1; 0), C(1; 0) th phng trnh ng thng cha cc cnh AB, AC c th vit l 3x + y = 3, ng thng cha cnh BC trng vi trc Ox. Hn na, r rng di ca BC c th thay bng mt s bt k no khc nn ta c bi ton sau: Bi ton 10. Trong mt phng ta Oxy, xt phn mt phng gii hn bi th ca y = 3 (a |x|) (a > 0) v trc honh. Gi S l tp hp cc im nm trn phn bao li ca min . Chng minh rng vi mi cch chia S thnh hai tp hp con ri nhau th lun tn ti ba im l nh ca mt tam gic vung. Cch gii bi ny hon ton tng t bi trn v vn mu cht l nhn nhn c bao li trn thc cht l mt tam gic u v cch chia tp con cng ging nh vic t cc im bi hai mu. Tip theo, chng ta s tm hiu mt m hnh kh thng dng na gii cc bi Ton m l xy dng mt bng thch hp v m theo hai chiu ca bng . Bi ton tip theo kh in hnh cho phng php ny xut hin trong k thi chn i tuyn Vit Nam d thi IMO 2001 (s 2001 xut hin trong bi ton c th thay bng mt s nguyn dng bt k no khc).

40 Bi ton 11. Cho dy s {an } tha mn 0 < an+1 an < 2001. Chng minh rng tn ti v s cp s nguyn dng (p, q) tha mn nu p < q th ap | aq . Li gii. T cch xc nh dy, ta thy rng s hng tip theo s khng ln hn s hng lin trc n cng thm 2001 n v. Nh th, ch cn xt 2002 s nguyn dng lin tip th s c t nht mt s hng thuc dy, t s l a. Ta xy dng bng nh sau: a(1,1) = a a(2,1) = a(1,1)
2002

a(1,2) = a + 1 a(2,2) = a(1,2)


2002

a(1,2002) = a + 2001 a(2,2002) = a(1,2001)


2002

+
i=1

a(1,i)

+
i=1

a(1,i)

+
i=1

a(1,i)

a(2002,1) = a(2001,1)
2002

a(2002,2) = a(2001,2)
2002

a(2002,2002) = a(2001,2002)
2002

+
i=1

a(2001,i)

+
i=1

a(2001,i)

a(2001,i)
i=1

Bng ny c tt c 2002 ct v 2003 hng. Trong , mi s hng hng th 2 tr i bng tch ca tt c s hng hng lin trc n cng vi s hng cng ct vi n. R rng, trong mi hng, c t nht mt s hng thuc dy cho. T theo nguyn l Dirichlet, ta c iu cn chng minh. Bi ton 12. Cho n s thc a1 , a2 , . . . , an bt k. Chng minh rng tn ti s thc x sao cho cc s a1 + x, a2 + x, . . . , an + x u l cc s v t. Li gii. Gi s t l mt s v t bt k. Ta s chng minh rng trong cc s t, 2t, 3t, . . . , (n+1)t c t nht mt s tha mn bi. Gi s khng c s no trong n + 1 s trn tha mn bi. Tc l vi mi i = 1, n + 1, trong cc s it + a1 , it + a2 , . . . , it + an lun tn ti mt s hu t. Xt bng n + 1 n nh sau: t + a1 2t + a1 (n + 1)t + a1 t + a2 2t + a2 (n + 1)t + a2 t + an 2t + an (n + 1)t + an

Theo nhn xt trn th trong mi hng tn ti t nht mt s hu t. Tc l trong ton b bng, ta c t nht n + 1 s hu t. V bng c n ct nn theo nguyn l Dirichlet, tn ti hai s hu t thuc cng mt ct, gi s l xt + ai v yt + ai . Khi (x y)t = (xt + ai ) (yt + ai ) cng phi l s hu t. iu ny v l v x, y nguyn v t v t. Mu thun trn chng t iu gi s ca chng ta l sai v bi ton c chng minh.

41

2. Xy dng th
2.1. th mt chiu
V phng php qu o ny, chng ta thng gp qu o xt trong mt phng ta hai chiu v cc kt qu xy dng trn . Trong phn ny, chng ta s th i phn tch thm mt s dng m rng na tm cc vn mi m hn. Trc ht, ta xt bi ton sau: Bi ton 13. C mt ngi i t gc ta O n im A c ta l n (n N) trn trc ta . Ngi ny mi ln c th bc mt hoc hai bc. Hi c bao nhiu cch bc ngi ny i c n A? Nu chng ta tip cn bi ton ny theo suy lun thng thng th c th gii nh sau: Gi x, y ln lt l s ln bc 1 bc v s ln bc 2 bc. Nh th ngi ny bc tng cng x + y bc v hai s ny phi tha mn x + 2y = n. ng vi mt trong s x + y ln bc, s cch bc tng ng l s cch chn x ln bc 1 bc, tc l x+y v s cch cn tnh l x x+y . x

0 x,y n,x+2y=n

R rng tng ny khng d dng rt gn c! Ta xt mt trng hp n gin, chng hn vi n = 4. Ta cn xt phng trnh nghim nguyn x + 2y = 4. D thy phng trnh ny c cc nghim l (x; y) = (0; 2), (2; 1), (4; 0) nn s cch bc trong trng hp ny l 2 + 3 + 4 = 1 + 3 + 1 = 5. 2 2 4 Nhng nu thay gi tr n = 4 bi n = 2011 th sao? Vic xt tt c cc b nghim ca phng trnh x + 2y = 2011 r rng khng n gin. Nu ta vit di dng cng thc tng qut th s c b nghim l x = 1 + 2t, y = 1005 t v s nghim cn xt l 1006. Chng ta th i cch tip cn sang mt phng php quen thuc l m bng truy hi. Gi S(n) l s cch bc ng vi trng hp ta ca A l n. Ta tnh c S(1) = 1, S(2) = 2 v nu n 3 th ta c hai trng hp: nu bc cui cng, ngi ny bc 1 bc th khi , s cch bc n 1 bc trc l S(n 1); nu bc cui cng, ngi ny bc 2 bc th khi , s cch bc n 2 bc trc l S(n 2). Hai trng hp ny l ri nhau v c tng l S(n) nn suy ra S(n) = S(n 1) + S(n 2), n 3. Vy dy s cc bc cn tnh chnh l dy Fibonacci 1, 2, 3, 5, 8, 13, 21, . . . v S(n) = Fn+1 (ta xt dy Fibonacci c hai s hng u l 1, 1). Nh th, ta c th rt ra mt kt qu kh th v x+y = Fn+1 bng mt s tht hin nhin l nu mt s lng m m bng l x
0 x,y n,x+2y=n

hai cch th phi cho cng kt qu. Chng ta s khng dng li y m th m rng bi ton ny hn na. Chng ta c th thay hai loi bc l 1 bc v 2 bc bi nhiu bc hn, phng trnh nghim nguyn khi cn xt cng s phc tp hn v dy s cng s dng truy hi nhiu cp hn. Chng hn, nu thay gi thit bi ton trn bi bc t 1 n 3 bc chn th cng thc phng trnh nghim x+y+z x+y+z v dy s cn xt s l nguyn s l y x
0 x,y,z n,x+2y+3z=n

S(1) = 1, S(2) = 2, S(3) = 4, S(n) = S(n 1) + S(n 2) + S(n 3), n

42 Cc tng lp lun v m hnh gii vn tng t. Nu ta thay i gi thit thm cht na th ta s c bi ton sau: Bi ton 14. C mt ngi i t gc ta O n im A c ta n trn trc ta . Ngi ny b say ru nn ch bc mi ln mt bc v c th bc ti hoc quay lui li. Ban u, ngay ti O ngi vn c th quay lui c nhng nu ngi i n A th khng th bc ti na. Hi ngi c bao nhiu cch bc nu s bc i thc hin l m? (R rng s bc c th l v hn nn ta ch xt trng hp c th vi m bc no ). Trc ht, chng ta s th m trong trng hp n gin hn l n = 5. R rng nu ch i ti th ngi cn 5 bc nhng y c cc bc li li nn nu gi x, y ln lt l s bc tin v bc li th x + y = m v x y = 5. Tuy nhin trong sut qu trnh thc hin cho n khi m bc th i lng x y khng c bng 5 m phi l x y 4 theo quy c l n A th khng th bc tip na. n y, cc bn th gii tip xy dng cc cng thc v cc kt qu tng t nh bi ton u tin trn. Ni v bi ton ny, chng ta li nh n mt bi ton ni ting khc trong k thi IMO 1979 v con ch nhy trn hnh bt gic u, k thi m ln u tin Vit Nam c c mt gii c bit ca TS. L B Khnh Trnh. Bi ton 15. Mt con ch nhy t nh A n nh E ca mt hnh bt gic u ABCDEF GH. Ti bt c nh no tr E, con ch c th nhy ti hai nh lin k vi nh . Gi an l s ng i phn bit t A n E c ng n bc nhy. Chng minh rng 1 a2n1 = 0, a2n = 2 H 2+ A B 2
n1

n1

O F E D

Li gii. D dng thy rng s cch bc phi l chn nn a2n1 = 0. Gi bn l s ng i phn bit ca ch t C (hoc G) n E. R rng sau hai bc nhy u tin, ch c th n C, hoc G hoc quay tr v A. Ta c cng thc sau a2n = 2a2n2 + 2b2n2 . T im C (hoc im G), cng bng hai bc nhy, ch c th tr v ch c hoc n A (nu n E th ch dng li), ta c b2n = 2b2n2 + a2n2 . T hai quan h ny, ta suy ra a2n = 4a2n2 2a2n4 , n 3. Ta tnh c a1 = a2 = a3 = 0, a4 = 2 nn d dng suy ra iu cn chng minh. Bi ton ny nu ct hnh bt gic ra v xp thnh ng thng th ta c th pht biu thnh:

43 Bi ton 16. C mt ngi i t gc ta O n im A hoc B (xem nh l i v mt trong hai ngi nh) c ta ln lt l 4, 4 trn trc ta . Ngi ny b say ru nn ch bc mi ln mt bc v c th bc ti hoc quay lui li. Ban u, ngay ti O ngi vn c th quay lui hoc tin ti nhng nu ngi i n A hoc B th phi dng li. Hi ngi c bao nhiu cch bc v nh nu s bc i thc hin l m? Kt qu ca bi ny hon ton ging vi bi ton trong k thi IMO trn. Nu ta th lm kh vn hn mt cht th c bi ton sau: Bi ton 17. C mt ngi i t gc ta O n im A c ta n trn trc ta . Ngi ny b say ru nn ch bc mi ln mt bc v c th bc ti hoc quay lui li. Ban u, ngay ti O ngi khng th quay lui c v nu ngi i n A th khng th bc ti na. Hi ngi c bao nhiu cch bc nu s bc i thc hin l m? R rng y c mt vn kh rc rc l nu ngi bc mt bc ri li quay v O ri c th lp li nhiu ln th cng thc truy hi s khng xy dng c (do t O th khng bc li c nhng t v tr c ta l 1 th c th c nn hai trng hp khng tng ng). Ni chung, vn cn nhiu iu kh th v trong trc ta mt chiu th ny. Trc khi kt thc, ta th xt mt bi ton t hp ni ting xut hin trong k thi IMO nm 2009: Bi ton 18. Cho n l mt s nguyn dng v a1 , a2 , . . . , an l cc s nguyn dng phn bit. Gi s T = {s1 , s2 , . . . , sn1 } l mt tp hp gm n 1 s nguyn dng nhng khng cha s = a1 + a2 + + an . Trn trc s thc c mt con chu chu lc u ng im 0. Con chu chu nhy ng n bc v hng dng ca trc s. Cc bc nhy ca n c di l a1 , a2 , . . . , an theo mt th t no . Trn trc s, ngi ta t sn n 1 ci by cc v tr s1 , s2 , . . . , sn1 . Chng minh rng con chu chu c th sp xp th t cc bc nhy ca mnh khng bao gi nhy vo by. y l bi ton c nh gi l kh th nh trong 50 nm thi IMO (ch sau bi 3, IMO 2007 t chc ti Vit Nam). Bi ton s dng quy np theo n l tng chnh. Li gii. Trng hp n = 1, theo gi thit th trn trc khng c by nn con chu chu c th nhy c v nh th th khng nh ng. Gi s con chu chu c th nhy c trong mi trng hp t 1 n n 1. Xt n bc nhy a1 , a2 , . . . , an v n 1 ci by s1 , s2 , . . . , sn1 . Khng mt tnh tng qut, ta sp th t a1 < a2 < < an v s1 < s2 < < sn1 . Ta s ch ra mt cch nhy tha mn iu kin. u tin con chu chu nhy bc c di an . S dng gi thit quy np, ta c th cho con chu chu nhy cc bc c di a1 , a2 , . . . , an1 sao cho n khng ri vo bt c by no trong s cc by s2 , s3 , . . . , sn1 . Nu trong qu trnh nhy ni trn con chu chu khng ri vo by s1 th hin nhin ta c mt cch nhy tha mn iu kin. Cn nu con chu chu ri vo by s1 , ta s ch ra mt

44 cch i ch cc bc nhy n khng nhy vo mt by no. Tht vy, gi s cc bc a1 , a2 , . . . , an1 c con chu chu nhy theo th t af (1) , af (2) , . . . , af (n1) . C hai kh nng xy ra: Con chu chu ri vo by ngay t bc nhy u tin, tc l s1 = an . Ta ch cn i ch hai bc nhy u tin ca con chu chu. C th n s nhy theo th t af (1) , an , af (2) , af (3) , . . . , af (n1) Do ta sp th t nn af (1) < an = s1 v hin nhin con chu chu khng vng vo by s1 na. T bc th hai tr i, con chu chu nhy nh c nn khng vng thm by no. Con chu chu ri vo by trong bc nhy af (t) , t 1. Ta xt 3 bc nhy af (t1) , af (t) , an . Ta i ch bc nhy u tin v bc nhy af (t+1) . Tc l cho con chu chu nhy theo th t af (t+1) , af (1) , af (2) , . . . , af (t1) , af (t) , an , af (t+2) , . . . , af (n1) Ch rng s1 l phn t nh nht trong tp n 1 ci by nn trc s1 , con chu chu s khng ri vo ci by no c. Cng do s i ch af (t+1) v an nn con chu chu s khng ri vo by s1 na. Tm li, t u n bc an , con chu chu khng ri vo by no. V v cc bc nhy sau c gi nguyn nn con chu chu cng khng nhy vo by. T hai trng hp trn, ta suy ra bi ton cng ng vi n. Theo nguyn l quy np ton hc, ta c iu cn chng minh. Bi ton kh nht theo nh gi l bi c ni dung nh sau (tt nhin l trong Shortlist vn cn nhiu bi kh hn na): Bi ton 19. Trong mt k thi Ton, mt s th sinh c quen bit nhau v tnh quen bit l hai chiu. Mt nhm nhng th sinh gi l nhm bn b nu nh tt c nhng ngi trong u c quen bit vi nhau. Nu nhm c t hn 2 ngi th cng c xem l nhm bn b. S lng th sinh trong mt nhm bn b c gi l c ca nhm . Gi s rng trong k thi ny, c ln nht ca mt nhm bn b tm c l s chn. Chng minh rng c th sp xp nhng th sinh ny vo hai phng sao cho c ln nht ca nhm bn b trong hai phng l bng nhau.

2.2. th hai chiu


Bi ton kh in hnh cho phng php ny xut hin trong k thi Olympic ca Vit Nam l bi 1, chn i tuyn tham d IMO nm 2003 vi ni dung nh sau: Bi ton 20. Trong mt phng ta , cho bn im phn bit A(0, 0), B(p, 0), C(m, q), D(m, n) vi m, n, p, q l bn s nguyn dng tha mn p < m v n < q. Xt mt ng i f t A n D v mt ng i g t B n C tha mn iu kin: cc ng ny ch i theo chiu dng

45 ca cc trc ta v ch i hng (t hng dng ca trc ta ny sang hng dng ca trc ta kia) ti cc im c ta nguyn. Gi S l s cc cp ng i (f, g) sao cho chng khng c im chung. Chng minh rng S= m+n n m+qp m+q q q m+np n

Cng nh cc m hnh phn tch trn th s dng qu o gii cc bi ton T hp gip cho li gii v lp lun r rng hn da trn cc nhn xt trc quan. Tuy nhin, chng ta cng phi cng nhn rng phng php ny ch c p dng trn mt s bi ton c trng nht nh no . Mt s bi ton m thun ty cng c th gii bng cch dng qu o th ny. Trc ht, chng ta s xy dng mt s khi nim c s cho phng php ny: nh ngha 1. Trn mt phng ta Oxy, mi im c cc ta u nguyn gi l mt im nguyn v tp hp cc im nguyn gi l li im nguyn (hay li Gauss). nh ngha 2. Trong h ta Oxy, ngi ta gi mt ng i t M ti N l mt ng gp khc ni M v N , cn ng i ngn nht l ng gp khc to bi cc on thng n v ngang v dc sao cho s on thng l t nht. Phng php chng minh mt cng thc t hp bi s ng i ngn nht gi l phng php qu o. Ta s chng minh mt s kt qu c s ca phng php ny : nh l 1. S ng i ngn nht t gc ta O n im A(m, n) vi m, n N l
m+n m

Chng minh. T O n A, ta cn thc hin m bc dch chuyn theo chiu ngang v n bc dch chuyn theo chiu dc, tc l tng cng c m + n bc. Cc ng i nh th ch khc nhau cch chn th t dc v ngang. Do , s ng i cn tm chnh l s cch chn m on ngang trong m + n on dc - ngang v c tng cng m+n ng i. m T y, ta c mt s kt qu sau: S cch chn on ngang v s cch chn cc on dc cng ging nhau nn m+n . n S ng i ngn nht t im A(m, n) n im B(p, q) vi 0 p+qmn . pm m < p, 0
m+n m

n < q l

S ng i ngn nht t im O n im B(p, q) v i qua im A(m, n) chnh l tch m+n p+qmn v s ng i t O n B khng i qua A l p+q m+n p+qmn . pm m p pm m T ta suy ra p+q > m+n p+qmn vi 0 m < p, 0 n < q. pm m p Tip theo, ta th dng phng php ny chng minh mt s bi ton: Bi ton 21 (ng thc Pascal). Vi k, n l cc s nguyn v 1 n k = n1 n1 + k1 k k n 1, ta c

46 Li gii. Ta thy rng c n ng i ngn nht t O(0, 0) n A(k, n k). k Xt hai im B(k 1, n k) v C(k, n k 1). R rng mi ng i t O n A nht thit phi qua B hoc C. S ng i t O n B l n1 v s ng i ngn nht t O ti C l n1 . k k1 Do , ta c n = n1 + n1 . k k1 k Bi ton 22. Chng minh rng vi m, n, k Z+ , m > k th
k k 0 k1 1 0 k Cm+n+1 = Cm Cn + Cm1 Cn+1 + + Cmk Cn+k

Li gii. C m+n+1 ng i ni O(0, 0) vi M (m + n k + 1, k). R rng c mi n+i i ki k 1 ng ct ng thng x = n + 2 ti im c tung i (i = 0, 1, . . . , k) gm: n+i ng ni O i vi im (n, i), mt ng ni im (n, i) vi im (n + 1, i) v mi ng ni im (n + 1, i) ki vi M . T ta c iu cn chng minh. Bi ton 23. C m + n ngi i mua v (m n), trong c m ngi mang tin loi 2 ng, n ngi mang tin loi 1 ng v mi v gi 1 ng. Bit rng ban u ngi bn v khng mang theo tin, hi c bao nhiu cch xp m + n ngi vo mua v ngi no cng c thi tin (nu cn) ngay lp tc? Li gii. t ai = x1 + x2 + + ai , trong xj bng 1 nu ngi th j mang tin loi 1 ng v bng 1 nu ngc li. y Am+n am+n . . . A2 a2 a1 O y = 1 Bi ton quy v vic m s ng i qua cc im Ai (i, ai ) m khng nm di trc honh trong mt phng to Oxy. Mun vy ta s m s ng i ct ng thng (d) : y = 1. Xy dng mt song nh t mi ng Q nh vy n mt ng Q l ng nhn c t Q khi cho i xng phn ca Q k t im u tin gp (d). Nu Q c x on hng ln v y on hng xung th x + y = m + n, y x = n m + 2 hay y = n + 1. Vy s ng Q l m+n , nn p s cn tm l m+n m+n . n+1 n n+1 A1 1 2 A3 3 m+n x Ai

47 thy r hn hiu qu ca phng php ny, ta s dng mt cch khc gii bi ton ny trong trng hp c bit hn l khi m = n. C th nh sau: Gi s c 2n ngi khch, n ngi trong c 1 ng v n ngi cn li c 2 ng. Ta cng cn tnh s cch sp xp 2n ngi ny tha mn iu kin bi, tc l ngi bn v c th bn c ngay v cho nhng ngi khch m khng c ai phi ng ch. Ta xt bi ton di mt hnh thc khc cho d lp lun. t 2n s, bao gm n s 1 v n s 2 ln mt hng v nh s chng t 1 n 2n theo chiu t tri sang phi. Vi mi 1 i 2n, ta gi ai , bi ln lt l s cc s 1 v s cc s 2 tnh t v tr i tr v trc, d thy rng i = ai + bi , i = 1, 2n. Chng ta cn tnh s trng hp tha ai bi i = 1, 2n. Gi S l tp hp cc hon v (t1 , t2 , t3 , . . . , t2n ) ca 2n phn t trn tha bi, T l tp hp cn li. n Suy ra |T | = C2n |S| v ta s tnh |T | trc. Do cch xc nh T nn vi mi phn t t = (t1 , t2 , . . . , tn ) ca T , ta thy tn ti t nht mt s i tha ai < bi . Gi f (t) l s th t u tin tha mn ai < bi , tc l trc , ai vn khng b hn bi v ngay ti thi im th bi hn ai ng mt phn t, tc l f (t) phi l s l v f (t) = ai + bi . Khi , ta tnh c af (t) = f (t)1 , b(t) = f (t)+1 , tf (t) = 2. Ti thi im ny, s 2 2 cc s 2 hn s cc s 1 ng mt n v nn khi ta i gi tr ca cc v tr pha sau tf (t) , t 1 sang 2, t 2 sang 1 th ta s c mt hon v g(t) ca n + 1 s 2 v n 1 s 1. Xt v d minh ha trong trng hp n = 6 nh sau t = {1, 2, 2, 1, 2, 2, 2, 1, 2, 1, 1, 1} f (t) = 3, a2 = 1, b2 = 1, t3 = 10

ai = {1, 1, 1, 2, 2, 2, 2, 3, 3, 4, 5, 6}, bi = {0, 1, 2, 2, 3, 4, 5, 5, 6, 6, 6, 6}

g(t) = {1, 2, 2, 2, 1, 1, 1, 2, 1, 2, 2, 2} Khi , g l mt nh x t T sang U , vi U l mt hon v ca n + 1 s 2 v n 1 s 1. Ta s chng minh g l song nh. Tht vy: Gi u l mt phn t ca U . Ta s m s cc s 1 v s cc s 2 t tri sang phi. Cng gi i l v tr nh nht sao cho s cc s 2 ln hn s cc s 1, do cch xc nh U nn v tr lun tn ti duy nht. Tip tc chuyn i 1 sang 2, 2 sang 1 cc v tr pha sau i Ban u, s cc s 2 hn s cc s 1 ng mt n v ti i v tr u v iu ny cng ng ti 2n i v tr sau. Sau khi thay i s cc s hn s cc s 2 ng mt n v ti 2n i v tr sau; ngha l s cc s 1 v s cc s 2 l bng nhau; tc l n tng ng vi 1 phn t ca T . Do , g l mt ton nh. Tip theo, ta s chng minh g l n nh. Gi t v t l hai phn t khc nhau ca T . Gi s i l v tr u tin m cc s t v t khc nhau. Khng mt tnh tng qut, gi s ti = 1, ti = 2. Khi d thy f (t) = i. Ta c hai trng hp. Nu f (t) < i th f (t ) < f (t) v ti i 1 v tr u, cc s l ging nhau. M ti v tr th i, ti = 1, ti = 2 nn g(t) = g(t ). Nu f (t) > i. Khi v tr th i ca g(t) l ti = 5. Ti i 1 v tr u, c hai bn

48 u ging nhau nn f (t ) g(t) = g(t ). 1. Do , v tr th i ca g(t ) l ti = 2. Tc l ta cng c

Do , lun c g(t) = g(t ), vi mi t = t nn g l n nh. Kt hp hai iu trn li, ta thy g l song nh. 2n 2n Ta c |T | = |U | = n1 , suy ra |S| = 2n n1 . n Kt qu ny tng ng vi iu ta thu c trong bi ton trn v r rng nhn c iu ny th cc bc lp lun rc ri hn kh nhiu. Quay li bi ton ban u trong k thi TST 2003 ca Vit Nam, ta c mt li gii y nh sau : Li gii. Ta gi mt ng i tha mn bi l ng i tt. Trc ht, ta thy s ng i tt t im A(0, 0) n im D(m, n) l m+n v t B(p, 0) n C(m, q) l m+qp . Do q n , s cp ng i tt (f, g) ty l m+n m+qp . q n Ta s tnh s cp ng i tt (f , g ) trong f i t A n C v g i t B n D, r rng hai ng i ny phi c t nht mt im chung K(i, j), p i m, 0 j q. Tng t trn, s cp ng i tt (f , g ) l m+q m+np . n q Ta li thy rng, s ng i tt t A n C i qua K l i+j (qj)+(mi) , s ng i tt qj j t B n D i qua K l i+jp (nj)+(mi) . nj j Do m+q m+np = i+j (qj)+(mi) i+jp (nj)+(mi) . nj j qj j n q Ta li thy K l im chung ca cc cp ng i (f, g) m chng c t nht mt im chung nn ta tnh c s cp ng i ny l i+j (qj)+(mi) i+jp (nj)+(mi) . nj j qj j iu ny c ngha l s cp ng i tt (f , g ) cng chnh bng s cp ng i (f, g) m chng c t nht mt im chung. Vy s cp ng i (f, g) tha mn bi chnh l m+n m+qp m+q m+np . Ta c iu n q q n cn chng minh. Tip theo ta xt mt s khi nim m u v phng php qu o nh ngha 3. Cho x > 0 v y l cc s nguyn. Qu o t gc to n im(x, y) l ng gp khc ni cc im (0, 0), (1, s1 ), (2, s2 ), . . . , (x, sx ) trong |si si+1 | = 1, sx = y. Gi Nx,y l s cc qu o ni im 0, 0 vi im (x, y). Ta c cc nh l sau nh l 2. Nx,y = khc tnh chn l.
p+q p

vi p =

x+y ,q 2

xy 2

nu x, y cng tnh chn l v Nx,y = 0 nu x, y

Chng minh. Gi s qu o gm p on hng ln trn v q on hng xung di. Khi , ta c p + q = x, p q = y hay p = x+y , q = xy . 2 2 V p v q l cc s nguyn nn x, y cn phi c cng tnh chn l. Do qu o s hon ton c xc nh nu ta ch ra cc on c hng ln trn, do s cc qu o t im O n im (x, y) l Nx,y = p+q . Trng hp x, y khc tnh chn l c th thy do khng tn p ti cc s p, q nguyn c th thc hin c ng i ca qu o nn Nx,y = 0. nh l 3 (Nguyn l i xng gng). Gi s A(a, ) v B(, b) l cc im c to nguyn vi b > a 0, > 0, > 0 v A (a, ) l im i xng vi A qua trc Ox. Khi s cc qu o t A n B ct trc Ox hoc c im chung vi Ox bng s cc qu o t A n B.

49 Chng minh. Mi mt qu o T t A n B, ct trc Ox hoc c im chung vi Ox ta cho tng ng vi qu o T t A n B theo quy tc sau: xt on qu o T t A cho n im gp nhau u tin gia T v Ox v ly i xng on ny qua Ox, tip theo T v T trng nhau. Nh vy mi mt qu o T t A n B ct Ox tng ng vi mt qu o xc nh t A n B. Ngc li mi mt qu o t A n B tng ng vi mt v ch mt qu o t A n B ct Ox (ly on qu o t A n B n im gp u tin v ly i xng on ny qua Ox). Nh vy, ta thit lp c song nh t tp hp cc qu o t A n B ct Ox vo tp hp cc qu o t A n B. nh l c chng minh. Ta xt tip bi ton sau xut hin trong IMO Shortlist 41. Bi ton 24. Cho p, q Z+ , (p, q) = 1. Hi c bao nhiu tp S gm cc s t nhin sao cho 0 thuc S v nu x thuc S th x + p, x + q cng thuc S? Li gii. Mi s nguyn n biu din duy nht di dng px + qy, vi 0 th ng nht mi s nguyn n vi im (x, y) trong mt phng to . y O 1 2 3 . . . 1 2 q x x < q, nn ta c

A(q, p)

R rng S cha mi im n khng nm di trc honh, nn bi ton quy v vic tm s cch nh du cc im nm trong tam gic vung cnh huyn OA vi A(q, p), sao cho nu mt im c nh du th nhng im bn phi v nm trn n cng c nh du. Ta thy rng mi cch nh du nh vy tng ng vi mt chui gm q s 0 v p s 1 th hin mt con ng i t O ti A (0 l sang phi, cn 1 l i xung) m nm hon ton trn ng OA ( l tr hai u mt, on OA khng cha im nguyn no). Phn hoch tt c cc chui thnh cc lp, m hai chui bt k trong mt lp l hon v vng quanh ca nhau. D thy mi lp c p + q chui, hn na trong mi lp ch c ng mt chui (p+q) p tho mn. T ta i ti p s l p+q . Nu ta m rng bi ton ra trong khng gian 3 chiu th vn s tr nn th v hn rt nhiu. Cc nh l v cc kt qu ca nhng php m ni chung vn tng t nh trong trng hp

50 hai chiu nhng chng ta s c dp tng tng, hnh dung ra cc qu o, nhng con ng i xut hin trong v t y, ta hon ton c th xy dng c nhiu bi ton, biu thc phc tp hn. Ta xt tip mt s ng dng khc trong S hc. Bi ton 25. Cho p, q Z+ , (p, q) = 1. Chng minh rng (p 1)(q 1) 2p (q 1)p q 2q (p 1)q p + + + = + + + = q q q p p p 2

Li gii. Trong mt phng to Oxy, xt cc im nguyn nm trong hnh ch nht ng cho OA, vi A(p, q). Ta thy ngay l c (p 1)(q 1) im. Do (p, q) = 1 nn khng c im no nm trn ng cho, do OA chia chng thnh hai phn i xng nhau. y q A

M O i p x

Xt phn nm di OA, ta thy s im nm trn ng thng x = i, 0 < i < p l [M N ] = OM AB OA = iq p

Do s im nm di ng cho OA l
p1

i=1

(p 1)(q 1) iq = p 2

Do tnh i xng ca p v q, ta c iu cn chng minh.

Bi ton 26. Vi mi s dng t, ta gi d(t) l s cc phn s ti gin p m 0 < p, q t. q + Tnh tng sau vi m, n Z m m m S=d +d + + d 1 2 n Li gii. Trong mt phng to Oxy, ta ng nht mi phn s a (khng nht thit ti b gin) vi im M (a, b) trong a, b l cc s nguyn dng. Xt cc im nguyn khng nm ngoi hnh ch nht ng cho AB, vi A(1, 1), B(m, n). Gi s mt ng thng l no qua O v cha cc im nguyn (p, q), (2p, 2q), . . . , (kp, kq) vi (p, q) = 1. V kp m, kq n nn suy ra m m n n n m < < < ,q < < < p k k1 1 k k1 1 Suy ra phn s ti gin p c tnh k ln trong cc s d m , d m , . . . , d m . iu cho thy q 1 2 k S chnh l s tt c cc im khng nm ngoi hnh ch nht ng cho AB, nn S = mn.

51 Bi ton 27. (a) Chng minh rng vi mi s t nhin n, tn ti duy nht xn , yn N sao cho 2n = (xn + yn )2 + 3xn + yn (b) Vi (xn , yn ) c xc nh trn, chng minh rng x1 + x2 + + x2015 = y1 + y2 + + y2015 = 41664 Li gii. Trong mt phng to Oxy, gi Tn l im th n trong dy im ch ra trn hnh v di y. y

10 6 3 1 11 7 4 8 12 13

14

D chng minh nu ta ca Tn l (xn , yn ) th (xn + yn )2 + 3xn + yn = 2n. T cu (a) l hin nhin. By gi ta chng minh cu (b). 2015 2015 2015 yi . rng T2015 l im th 62 trn trc honh v V = xi , OTi = i=1 i=1 i=1 Gi l vector ng cho ca hnh vung n v, d thy tng cc vector OTi vi cc im v Ti nm trn cnh huyn ca tam gic cnh k vung cn ti O bng k(k+1) . Do v 2 v v (12+23+ +6263) = (12(30)+23(41)+ +6263(6461)) = 41664 v V = 2 6 Ta c iu cn chng minh.

Ti liu tham kho


[1] Nguyn Vn Mu, Chuyn T hp v ri rc, NXBGD, 2007.

52 [2] Trn Nam Dng, Ti liu tp hun i tuyn thi IMO 2010. [3] Hunh Tn Chu, ng dng im nguyn gii Ton s hc v t hp, 2011. [4] Nguyn Khc Minh, Nguyn Vit Hi, Cc bi thi Olympic Ton THPT Vit Nam, 2007. [5] Tp ch Ton hc v Tui tr. [6] Cc din n Ton hc : Din n Art of Problem Solving (Mathlinks) http://www.artofproblemsolving.com/Forum/index.php Din n MathScope http://forum.mathscope.org/index.php

PHNG PHP HM SINH


Hong Minh Qun1

1. Tm tt l thuyt
1.1. nh ngha
nh ngha 1. Hm sinh ca dy s thc a0 , a1 , a2 , . . . , an , . . . l hm s c xc nh bi G(x) = a0 + a1 x + a2 x2 + + an xn + nh ngha 2. Cho dy s thc a0 , a1 , a2 , . . . , an , . . . Hm s cho bi cng thc G(x) =
n=0

an

xn n!

c gi l hm sinh dng m ca dy a0 , a1 , a2 , . . . , an , . . .

1.2. Mt s ng thc lin quan n hm sinh


1 = 1 + x + x2 + x3 + 1x 1 2 3 2 = 1 + 2x + 3x + 4x + (1 x)

1 n(n + 1) 2 n(n + 1)(n + 2) 3 x + x + = n = 1 + nx + (1 x) 2! 3! 1 = 1 x + x2 x3 + 1+x 1 2 2 3 3 2 = 1 + 2ax + 3a x + 4a x + (1 ax) 1 = 1 + xr + x2r + x3r + r 1x Hai mnh thng c s dng : Mnh 1. Cho hm sinh G(x) = (1 + x + x2 + )n .
r (a) t ar l h s ca xr trong khai trin ca G(x) th ar = Cr+n1 ; 1 2 (b) (1 xm )n = 1 Cn xm + Cn x2m + (1)n xmn ;
1

i=0

i Ci+n1 xi

Gio vin trng THPT Ngc To, H Ni.

53

54 (c) (1 + x + x2 + + xm1 )n = (1 xm )n (1 + x + x2 + )n . Mnh 2 (Cng thc xc nh h s tch ca hai hm sinh). Cho hai hm sinh ca hai dy (an ), (bn ) ln lt l A(x) = a0 + a1 x + a2 x2 + B(x) = b0 + b1 x + b2 x2 + t G(x) = A(x)B(x) = (a0 + a1 x + a2 x2 + )(b0 + b1 x + b2 x2 + ) Khi h s ca xr trong khai trin ca G(x) l a0 br + a1 br1 + a2 br2 + + ar2 b2 + ar1 b1 + ar b0 ()

= a0 b0 + (a0 b1 + a1 b0 )x + (a0 b2 + a1 b1 + a2 b0 )x2 + (a0 b3 + a1 b2 + a2 b1 + a0 b3 )x3 +

Ch . Trong cc v d ng dng hm sinh gii bi ton m nng cao phn II chng ta rt hay s dng ng thc ().

2. ng dng hm sinh gii cc bi ton m in hnh


2.1. ng dng hm sinh gii bi ton chia ko Euler
tng chung ca phng php s dng hm sinh gii bi ton m l i tm h s ca xr trong khai trin ca hm sinh vi r l s phn t c chn ra trong n i tng vi nhng iu kin rng buc cho trc. By gi chng ta s vn dng nhng kin thc hm sinh trn vo vic gii quyt cc bi ton m t hp nng cao. Thng qua nhiu v d khc nhau di y chng ta s nh hnh v nm chc c cch s dng hm sinh trong vic gii bi ton m t hp nng cao. V d 1. Vo ngy ngh ch nht, c Hoa i chi v mua qu l 12 qu cam cho 3 a tr An, Bnh, Chi.Hi c Hoa c bao nhiu cch phn phi 12 qu cam sao cho An c t nht 4 qu, Bnh v Chi mi ngi u c t nht 2 qu, nhng Chi khng c nhiu hn 5 qu? Li gii. Hm sinh cho s cch chn qu cho An l A(x) = x4 + x5 + x6 + x7 + x8 = x4 (1 + x + x2 + x3 + x4 ) = x4 Hm sinh cho s cch chn qu cho Bnh l B(x) = x2 + x3 + x4 + x5 + x6 = x2 (1 + x + x2 + x3 + x4 ) = x2 Hm sinh cho s cch chn qu cho Chi l C(x) = x2 + x3 + x4 + x5 = x2 (1 + x + x2 + x3 ) = x2 1 x4 1x 1 x5 1x 1 x5 1x

55 Hm sinh cho s cch phn phi 12 qu cam tha mn iu kin bi l G(x) = A(x)B(x)C(x) = x4 1 x5 2 1 x5 2 1 x4 x8 (1 x5 ) (1 x4 ) x x = 1x 1x 1x (1 x)3 1 x1
3 3 2

= x8 (1 2x5 + x10 )(1 x4 )

= (x8 x12 2x13 + 2x17 + x18 x22 )

Do cn tm h s ca x12 trong khai trin ca G(x) nn ta ch quan tm ti h s ca U (x) = (x8 x12 2x13 + 2x17 + x18 x22 ) vi bc 12. Do U (x) ch c cc h s a8 , a12 l tha mn. 3 1 r V h s ca xr trong khai trin V (x) = x1 l br = Cr+2 . 4 0 Vy h s ca x12 trong khai trin ca G(x) l a8 b4 + a12 b0 = 1.C6 1.C2 = 14. Kt lun : C Hoa c 14 cch phn chia 12 qu cam cho 3 a tr tha mn yu cu An c t nht 4 qu, Bnh v Chi mi ngi u c t nht 2 qu, nhng Chi khng c nhiu hn 5 qu. Nhn xt. Thot nhn ban u chng ta thy cch gii bng lit k cho li gii ngn gn hn cch hm sinh nhng suy ngh su thm chng ta s thy i vi bi ton c d kin ln th cch lm lit k t ra km hiu qu thm ch kh lm ra c, chng hn bi ton trn chng ta thay i mt cht nh sau : Vo ngy ngh ch nht, c Hoa i chi v mua qu l 50 ci ko cho 3 a tr An, Bnh, Chi.Hi c Hoa c bao nhiu cch phn phi 50 ci ko sao cho An c t nht 4 ko, Bnh v Chi mi ngi u c t nht 2 ko, nhng Chi khng c nhiu hn 5 ko? R rng cch lm lit k i vi bi ton ny tr nn km hiu qu, kh khn v mt thi gian hn rt nhiu v chng ta phi xt qu nhiu trng hp. Khi gii php hm sinh trong bi ton ny em li cho chng ta hiu qu r rt v chng ta ch cn quan tm ti h s trong khai trin ca hm sinh tng ng bi. Trong thc tin th d liu rt a dng, vi nhng bi ton m c nhiu iu kin rng buc khc nhau vic s dng hm sinh s cho chng ta li gii hiu qu. V d 2. C bao nhiu cch xp mt gi gm n tri cy gm (to, chui, cam,o), sao cho s to phi l chn, s chui chia ht cho nm, ch c th nhiu nht 4 qu cam v nhiu nht 1 qu o. Li gii. Hm sinh cho s cch chn qu to (s chn) l A(x) = 1 + x2 + x4 + x6 + = Hm sinh cho s cch chn qu chui (s chia ht cho 5) l 1 1 x2 1 1 x5

1 x1

B(x) = 1 + x5 + x10 + x15 + = Hm sinh cho s cch chn qu cam (nhiu nht 4 qu) l C(x) = 1 + x + x2 + x3 + x4 =

1 x5 1x

56 Hm sinh cho s cch chn qu o (nhiu nht 1 qu) l D(x) = 1 + x = Hm sinh cho s cch chn c 4 loi qu l 1 x2 1x
i=0

1 1 x5 1 x2 1 1 = = G(x) = A(x)B(x)C(x)D(x) = 2 1 x5 1x 1x 1x (1 x)2 Vy s cch chn tri cy tha mn bi l n + 1 cch.

(i + 1)xi

V d 3. C bao nhiu cch chn ra 15 USD t 20 ngi nu 19 ngi u, mi ngi c th a ra nhiu nht 1 USD, ngi th 20 c th a ra 1 USD hoc 5 USD hoc khng USD no. Li gii. Hm sinh cho s cch chn nhiu nht 1 USD t 19 ngi l A(x) = (1 + x)19 Hm sinh cho s cch chn 1 USD hoc 5 USD hoc khng USD no ngi th 20 l B(x) = 1 + x + x5 Hm sinh cho s cch chn ra 15 USD l G(x) = A(x)B(x) = (1 + x)19 (1 + x + x5 ) Chng ta tm h s ca x15 trong khai trin ca G(x). Ta c:
19

(1 + x)

19

=
k=0

k C19 x19k

t ar l h s ca xr trong khai trin ca A(x), br l h s ca xr trong khai trin ca B(x). r Khi ta c ar = C19 v b0 = b1 = b5 = 1. Vy h s ca x15 trong khai trin ca G(x) l a15 b0 + a14 b1 + a13 b2 + + a0 b15 Ta c
15 14 10 a15 b0 + a14 b1 + a10 b5 = C19 + C19 + C19 = 107882

Vy c 107882 cch chn ra 15 USD tha mn iu kin bi. V d 4. Tm s nghim nguyn dng ca phng trnh u + v + w + z = 27 vi 3 u, v, w, z 8

Li gii. Hm sinh cho s nghim nguyn dng ca phng trnh l G(x) = (x3 + x4 + + x8 )4

= x12 (1 + x + x2 + + x5 )4

= [x3 (1 + x + x2 + + x5 )]4

57 S nghim nguyn dng ca phng trnh l h s ca x27 trong khai trin ca G(x) v l h s ca x15 trong khai trin ca H(x) = (1 + x + x2 + + x5 )4 . Ta c 4 4 1 1 x6 = (1 x6 )4 H(x) = (1 + x + x2 + + x5 )4 = 1x 1x t A(x) = (1 x6 )4 , B(x) =
4 1 . 1x

Ta c

1 2 3 A(x) = (1 x6 )4 = 1 C4 x6 + C4 x12 C4 x18 + x24

B(x) =

Do tm h s ca x15 trong khai trin ca H(x) nn ta ch quan tm ti h s ca A(x) vi bc 15.Do A(x) ch c cc h s a0 , a6 , a12 l tha mn. 4 1 r r V h s ca xr trong khai trin B(x) = x1 l br = Cr+41 = Cr+3 . Vy h s ca x15 trong khai trin ca H(x) l
15 1 9 2 3 a0 b15 + a6 b9 + a12 b3 = 1 C18 C4 C12 + C4 C6 15 1 9 2 3 Vy s nghim nguyn dng ca phng trnh l C18 C4 C12 + C4 C6 .

1 1x

1 2 3 = 1 + C 4 x + C 5 x2 + C 6 x3 +

V d 5. Phng trnh x1 + x2 + x3 + x4 + x5 = 30 c bao nhiu nghim nguyn dng tha mn 0 x1 , x2 10; 3 x3 , x4 , x5 5 v x1 , x2 chn

Hng dn. Hm sinh cho s nghim nguyn dng ca phng trnh l G(x) = 1 + x2 + x4 + + x10 Cng vic tm h s x30 xin dnh cho bn c.
2

x3 + x4 + x5

2.2. ng dng hm sinh chng minh ng thc t hp


V d 6. Chng minh ng thc sau vi n l s t nhin n 0
2

n 1

+ +

n n
2n n

2n n

Li gii. Xt a thc (1 + x)2n c h s ca xn l

. Mt khc ta c

(1 + x)2n = (1 + x)n (1 + x)n Xt hm sinh G(x) = H(x) = (1 + x)n c h s ar = br = a0 bn + a1 bn1 + + an b0 = V vy ta c n 0


2 n r

. H s xn ca G(x)H(x) l
2

n 0

n 1
2

+ + 2n n

n n

n + 1

n + + n

58 V d 7 (ng thc Vandermonde). Chng minh ng thc sau vi r


m

min{m, n}

k=0

m k

n rk

m+n r

Li gii. Da vo tng li gii v d ??? bn c c th t chng minh ng thc Vandermonde. V d 8 (China 1994). Chng minh ng thc
n

2nk
k=0

n k

k
k 2

2n + 1 n

Li gii. Ta c Khi
n

k k 2

l h s t do trong khai trin (1 + x)(x + x1 )k .


k n k

k=0

1 n (1 + x) x + k x

nk

= (1 + x)
k=0

n k

1 x+ x 1 x
n

2nk

= (1 + x) 2 + x + =

1 (1 + x)(2x + x2 + 1)n xn 1 = n (1 + x)2n+1 x k


k 2

So snh h s t do ta c

k=0

n nk 2 k

2n + 1 n

V d 9. Chng minh cc s Fibonacci c th c vit di dng Fn = n n1 n2 + + + 0 1 2

Li gii. Chng ta bit cng thc dng truy hi ca s Fibonacci F1 = F2 = 1 Fn = Fn1 + Fn2 ,n 3

t G(x) l hm sinh cho dy (Fn ) v gi s F0 = 0. ta c: G(x) = F0 + F1 x + F2 x2 + F3 x3 +

x2 G(x) = F0 x2 F1 x3 F2 x4 Cng v vi v ba ng thc trn, ta c (1 x x2 )G(x) = F0 + (F1 F0 )x + (F2 F1 F0 )x2 + = x

xG(x) = F0 x F1 x2 F2 x3

59 Do G(x) = Ta vit li G(x) nh sau:

x 1 x x2

1 1 = = 1 + x(1 + x) + x2 (1 + x)2 + + xn (1 + x)n 1 x x2 1 x(1 + x) H s ca xn trong khai trin cui l


n 0

n1 1

n2 2

+ .

2.3. ng dng hm sinh gii bi ton phn hoch


Mt phn hoch ca s t nhin r l mt cch vit r thnh tng ca cc s nguyn dng hay mt b s khng th t (ai ) tha mn ai = r. V d v phn hoch : 2=1+1 3=2+1=1+1+1 4=3+1=2+2=2+1+1=1+1+1+1 5=4+1=3+2=3+1+1=2+2+1=2+1+1+1=1+1+1+1+1 t ek l s nguyn dng thnh phn xut hin k ln. Ta c 1e1 + 2e2 + + kek + + rer = r Xy dng hm sinh cho phng trnh trn ta c G(x) = (1 + x + x2 + x3 + + xn + )

(1 + x2 + x4 + x6 + + x2n + )

(1 + x3 + x6 + x9 + + x3n + ) (1 + xk + x2k + x3k + + xkn + )

V d 10. Xy dng hm sinh m s nghim nguyn dng ca phng trnh 2x + 3y + 5z = r Li gii. Hm sinh cho phng trnh trn l (1 + x2 + x4 + + x2n )(1 + x3 + x6 + + x3n )(1 + x5 + x10 + + x5n ) V d 11. Hi c bao nhiu cch i t 500 nghn ng thnh cc t 1 nghn, 2 nghn, 5 nghn, 10 nghn, 20 nghn. vi x, y, z 0

60 Hng dn. Bi ton cho quy v m s nghim nguyn dng ca phng trnh x1 + 2x2 + 5x3 + 10x4 + 20x5 = 500 S nghim nguyn dng ca phng trnh chnh l h s ca x500 trong khai trin ca hm sinh G(x) = 1 + x + x2 + 1 + x2 + x4 + 1 + x5 + x10 +

1 + x10 + x20 + 1 + x20 + x40 + 1 = 2 ) (1 x5 ) (1 x10 ) (1 x20 ) (1 x) (1 x

V d 12 (China 1996). Cho n l s nguyn dng. Tm s a thc P (x) vi h s thuc {0; 1; 2; 3} tha mn P (2) = n. Li gii. t P (x) = a0 + a1 x + a2 x2 + + ak xk + Theo gi thit P (2) = n nn ta c a0 + 2a1 + 4a2 + + 2k ak + = n vi 0 Chng ta xy dng hm sinh: G(x) = (1 + x + x2 + x3 )(1 + x2 + x4 + x6 )(1 + x4 + x8 + x12 ) =

ak

3.

1 + x2 + x2(2 ) + x3(2
k+2

k)

1 1 x2 = = 2k (1 x) (1 x2 ) 1x k=0 = = 1 1 1 1 1 1 + 2 + 4 1 x 2 (1 x) 41+x

k=0

k=0

1 1 1 + (1)n + (n + 1) xn 4 4 2

Vy c

k=0

1 4

1 + 1 (1)n + 2 (n + 1) a thc tha mn bi. 4

3. Mt s bi ton tng hp
V d 13. Hun luyn vin bng c n cu th tp luyn hng ngy. u tin hun luyn vin chia cc cu th thnh 2 nhm v yu cu cc cu th mi nhm xp thnh hng. Nhm th nht c th chn o da cam, o trng hoc o xanh, nhm th hai c th chn o . Hi c bao nhiu cch thc hin cng vic chn o nh th. Hng dn. Gi s hun luyn vin chn k ngi t nhm th nht. t ak l s cch m k ngi ny chn cc o mu da cam, trng, xanh v nhm xp thng hng nn c ak = k!3k .

61 Do hm sinh ly tha cho ak l A(x) =


k 0

k!3k

1 xk = k! 1 3x

Tng t t bm l s cch chn m ngi t nhm th hai xp hng thng v chn o , ta c bm = m!. Hm sinh cho dy bm l B(x) =
m 0

m!

xm 1 = m! 1x

Vy hm sinh cho c 2 nhm chn o l G(x) = A(x)B(x) = n y bn c ch cn tm h s ca n!(3n+1 1) p s : . 2


xn n!

1 1 . 1 3x 1 x

trong khai trin G(x) l xong.

V d 14. Cho r vt. Hi c bao nhiu cch phn phi r d vt khc nhau vo trong n ci hp sao cho khng c hp no rng. Li gii. Hm sinh ly tha cho s cch phn phi r vt l G(x) = x2 x3 + + x+ 2! 3!
n n

= (ex 1)n =
i=0 n

n (ex )ni (1)i i n i


n r=0

= = Do ta c ar =
n i=0 i=0 r=0

(n i)r xr r! n (n i)r i

ni

(1)i xr r!

(1)i
i=0

(1)i

n i

(n i)r .

V d 15. Cho s t nhin r. m s phn hoch r gm cc thnh phn xut hin cc s 1, 2, 3, 5, 7. Hng dn. Xy dng hm sinh G(x) = 1 (1 x)(1 x2 )(1 x3 )(1 x5 )(1 x7 ) V d 16. C bao nhiu cch phn phi 25 qu bng ging ht nhau vo by hp ring bit sao cho hp u tin c khng qu 10 qu bng v s bng l ty mi hp trong su hp cn li.

62 Li gii. Hm sinh cho s cch phn phi r qu bng vo by hp ring bit sao cho hp u tin c khng qu 10 qu bng l: G(x) = (1 + x + x2 + x3 + + x10 )(1 + x + x2 + )6 = 1 x11 1x
7

1 1x

= (1 x11 )

1 1x

V B(x) =

1 t A(x) = 1 x11 , B(x) = 1x . Chng ta c h s khc 0 trong hm s A(x) l a0 = 1; a11 = 1. 7 1 1x

25 14 Vy h s ca x25 trong khai trin ca G(x) l a0 b25 + a11 b14 = C31 C20 = 697521. Vy c 697521 cch phn phi 25 qu bng ging ht nhau vo by hp ring bit sao cho hp u tin c khng qu 10 qu bng v s bng l ty mi hp trong su hp cn li.

r=0

r r Cr+6 nn h s ca xr l Cr+6 .

V d 17. C bao nhiu cch chn 25 chi t by loi chi khc nhau sao cho mi loi chi c t 2 n 6 chi c chn. Li gii. Hm sinh cho s cch chn r chi t by loi chi khc nhau sao cho mi loi c t 2 n 6 chi c chn l: G(x) = (x2 + x3 + x4 + x5 + x6 )7 = [x2 (1 + x + x2 + x3 + x4 )]7 = x14 (1 + x + x2 x3 + x4 )7 tm h s ca x25 trong khai trin ca G(x) th chng ta tm h s ca x11 trong khai trin H(x) = (1 + x + x2 x3 + x4 )7 = t A(x) = (1 x ) ; B(x) = B(x) =
11 5 7

1 x5 1x

= (1 x5 )7 1 1x
r Cr+6 xr 7

1 1x

1 1x

r=0

Do tm h s ca x trong khai trin ca H(x) nn ta ch quan tm ti h s ca A(x) vi bc 11.Do A(x) ch c cc h s a0 ; a5 ; a10 l tha mn. V h s ca xr trong khai trin B(x) r l br = Cr+6 . 11 1 6 2 1 Vy h s ca x11 trong khai trin ca H(x) l a0 b11 + a5 b6 + a10 b1 = C17 + (C7 )C12 + C7 C7 = 6055. V d 18. Tnh tng S = 1 2 + 2 2 + 3 2 + + n2 Li gii. Xut pht t hm sinh x 2 3 r 2 = 1x + 2x + 3x + + rx + (1 x)

63 Ta c x x d dx (1 x)2 = x(1 + x) = 12 x + 22 x2 + 32 x3 + + r 2 xr + (1 x)3

Nhn c 2 v ca ng thc cui vi 2 ta c 2x(1 + x) = 2(12 x + 22 x2 + 32 x3 + + r2 xr + ) 3 (1 x)

G(x) =

1 Vy h s ar = 2r2 . Ta c cn tnh 2 (a0 + a1 + a2 + + an ). Ta c

G (x) =

x(x + 1) G(x) = = x(1 x)4 + x2 (1 x)4 2(1 x) (1 x)4

H s ca xn trong khai trin x(1 x)4 l h s ca xn1 trong khai trin (1 x)4 . H s ca xn trong khai trin x2 (1 x)4 l h s ca xn2 trong khai trin (1 x)4 . Do S= (n 2) + 4 1 (n 1) + 4 1 + n2 n1 = n+1 n+2 + 3 3

V d 19. Trong mt ti sch ca Long c cha bao gm 10 chic nhn vng, 20 chic nhn bc v 30 vin kim cng.Hi Long c bao nhiu cch chn ra 30 vt em bn, bit rng mi loi trang sc c t nht 1 vt c ly ra.

Li gii. Hm sinh cho s cch chn t nht mt chic nhn vng c chn l M (x) = x + x2 + x3 + + x10 Hm sinh cho s cch chn t nht mt chic nhn bc c chn l N (x) = x + x2 + x3 + + x20 Hm sinh cho s cch chn t nht mt vin kim cng c chn l P (x) = x + x2 + x3 + + x30 Vy hm sinh cho s cch chn 30 vt em bn, bit rng mi loi trang sc c t nht

64 mt vt c ly ra l: G(x) = M (x)N (x)P (x) = (x + x2 + x3 + + x10 )(x + x2 + x3 + + x20 )(x + x2 + x3 + + x30 ) 1 x10 1 x20 1 x30 . . 1x 1x 1x x3 (1 x10 )(1 x20 )(1 x30 ) = (1 x)3 x3 (1 x20 x10 + x30 )(1 x30 ) = (1 x)3 x3 (1 x30 x20 + x50 x10 + x40 + x30 x60 ) = (1 x)3 x3 (1 x10 x20 + x40 + x50 x60 ) = (1 x)3 1 = (x3 x13 x23 + x43 + x53 x63 ) (1 x)3 x3 (1 x10 x20 + x40 + x50 x60 ) = (1 x)3 1 = (x3 x13 x23 + x43 + x53 x63 ) (1 x)3 = x3 t A(x) = x3 x13 x23 + x43 + x53 x63 ; B(x) =
1 (1x)3 1 . (1x)3

= x3 (1 + x + x2 + + x9 )(1 + x + x2 + + x19 )(1 + x + x2 + + x29 )

Trong khi B(x) =

H s khc khng v c bc nh hn 30 ca A(x) l a3 = 1; a13 = 1; a23 = 1. =


r=0 r Cr+31 xr = r=0

r r Cr+2 xr c h s ca xr l br = Cr+2 .

Vy h s ca x30 trong khai trin ca hm sinh G(x) l


27 17 7 a3 b27 + a13 b17 + a23 b7 = C29 C19 C9 = 199

Vy Long c 199 cch chn 30 vt em bn m mi loi trang sc c t nht mt vt c ly ra. V d 20. C bao nhiu cch phn hoch s t nhin n thnh cc thnh phn gm cc s nguyn dng l khc nhau? Hng dn. Bi ton tr thnh tm h s ca xn trong khai trin ca hm sinh 1 1 1 1 3 1 x5 1 x7 1x 1x V d 21. Phng trnh sau c bao nhiu nghim nguyn? u + v + w + z = 20 vi u 0, v 0, w = 2m, z = 2k + 1

65 Hng dn. Xt hm sinh G(x) = (1 + x + x2 + x3 + x4 + )2 (1 + x + x2 + x4 + )(1 + x3 + x5 + x7 + ) V d 22 (ng thc Pascal). Chng minh rng n k = n1 n1 + k k1

Li gii. Xt hm sinh Gn (x) = (1 + x)n . Ta vit li nh sau : Gn (x) = (1 + x)n = (1 + x)(1 + x)n1 = (1 + x)Gn1 x = Gn1 (x) + xGn1 (x) rng : H s ca xk trong khai trin Gn (x) = (1 + x)n l trin Gn1 (x) + xGn1 (x) l n1 + n1 . k1 k Vy n = n1 + n1 . k1 k k
n k

, cn h s ca xk trong khai

V d 23 (PTNK 2009). Tm s tt c cc s c n ch s lp t cc ch s 3, 4, 5, 6 v chia ht cho 3. Li gii. Mt s chia ht cho 3 nu tng cc ch s ca n chia ht cho 3. Mi ch s c th l 3, 4, 5, 6. Xt hm sinh G(x) = (x3 + x4 + x5 + x6 )n = g0 + g1 x + g2 x2 + + g6n x6n Gi s tt c cc s c n ch s lp t cc ch s 3, 4, 5, 6 v chia ht cho 3 l Sn th Sn chnh l tng cc h s ca cc s m chia ht cho 3. 2i Gi = e 3 l cn bc ba nguyn thy ca phng trnh x3 = 1, ta c 2 + + 1 = 0. Suy ra G(1) + G() + G(2 ) = 3g0 + (1 + + 2 )g1 + (1 + + 2 )g2 + 3g3 + = 3(g0 + g3 + g6 + ) = 3Sn Vy Sn = 1 G(1) + G() + G(2 ) 3 1 (1 + 1 + 1 + 1)n + 2 + 1 + + 1 = 3 1 n = (4 + 2) 3

+ + 1 + 2 + 1

V d 24. Cho s nguyn dng n. Gi n l s cch phn tch n thnh tng cc s t nhin l, n l s cch phn tch n thnh tng cc s t nhin i mt khc nhau. Hy chng t rng n = n . Li gii. Xt hm sinh F (x) =
iN,i l

(1 + xi + x2i + x3i + ).

H s ca xn trong khai trin ca F (x) chnh l n . Xt hm sinh G(x) = (1 + x) (1 + x2 ) (1 + x3 ) . H s ca xn trong khai trin ca G(x) chnh l n . 2 4 1 1 1 Ta c F (x) = 1x 1x3 1x5 v G(x) = 1x 1x4 1x 1x Do F (x) G(x) hay n = n .

1x6 1x3

1 1x

1 1x3

1 1x5

66 V d 25 (IMO 1998 Shortlist). Cho dy s a0 , a1 , a2 , . . . , an l dy s khng gim sao cho mi s t nhin u c th biu din duy nht di dng ai + 2aj + 4ak vi i, j, k l cc s t nhin khng nht thit khc nhau. Tm a1998 . Li gii. Xt hm sinh G(x) = xa0 + xa1 + xa2 + Ta c G(x2 ) = x2a0 + x2a1 + x2a2 + v G(x4 ) = x4a0 + x4a1 + x4a2 + Vy ta c hm sinh F (x) = G(x)G(x2 )G(x4 ) = xai +2aj +4ak
i,j,k

Theo gi thit: Mi s t nhin u c th biu din duy nht di dng ai + 2aj + 4ak vi i, j, k l cc s t nhin khng nht thit khc nhau nn ta c th biu din F (x) = 1 + x + x2 + x3 + = T ta c: F (x) = G(x)G(x2 )G(x4 ) = 1 1x

1 1x 1 1 1 2 2 4 8 = F (x ) = G(x )G(x )G(x ) = 2 1x 1x 1+x


G(x) V th G(x8 ) = 1 + x hay G(x) = (1 + x)G(x8 ) = (1 + x)(1 + x8 )G(x8 ) = 2 2 3 Vy G(x) = (1 + x)G(x8 ) = (1 + x)(1 + x8 )G(x8 ) = (1 + x)(1 + x8 )(1 + x8 )G(x8 ) Ta c ai l s t nhin nn vit theo c s 8 th c biu din bi cc s 0 v 1. Do ta biu din 1998 theo c s 2 , ri thay c s 2 bi c s 8. Ta c 1998 = 11111001110(2) . Do a1998 = 11111001110(8) .
2

V d 26. Tm s cc tp con ca tp {1, 2, 3, . . . , 2003} sao cho tng cc phn t ca chng chia ht cho 5. Hng dn. Xt hm sinh G(x) = (1 + x)(1 + x2 )(1 + x3 ) (1 + x2003 ). Gi s khai trin c G(x) = an xn . Ta cn tnh tng cc h s c s m chia ht cho 5, tc l tnh a0 + a5 + a10 + p s : 1 (22003 + 2401 ). 5

4. Cc bi tp t luyn
Bi tp 1. Tm s nghim nguyn dng ca phng trnh u + v + w + z = 20, 1 u, v, w, z 7

Bi tp 2. Tm s nghim nguyn dng ca phng trnh u + v + w + z = 20 vi 1 u 4; 3 v, w, z 8

Bi tp 3. C bao nhiu cch phn phi 10 qu bng ging nhau cho 2 cu b v 2 c b sao cho mi cu b c t nht 1 qu bng v mi c b c t nht 2 qu bng? Bi tp 4. C Trang c 25 bng hoa v 4 l hoa. Hi c Trang c bao nhiu cch phn phi 25 bng hoa v 4 l hoa sao cho mi l c t nht l 3 bng hoa v nhiu nht l 7 bng hoa.

67 Bi tp 5. Hi c bao nhiu cch chn 25 qu bng gm 3 loi bng, xanh, , trng sao cho s bng chn nhiu nht l 2, s bng xanh chn nhiu nht l 3 v s bng trng chn nhiu nht l 4. Bi tp 6. Mt cu b c cha tng 30 vin bi lm chi. Hi cu b c bao nhiu cch phn phi 30 vin bi vo 5 ci hp sao cho hai hp u c cha s chn vin bi v s bi trong mi hp khng vt qu 10 vin, v s bi trong mi hp cn li c t nht 3 vin v nhiu nht l 5 vin. Bi tp 7. C bao nhiu cch su tm 24 con tem t 4 bn nam v 6 bn n.Bit rng mi ngi c t nht mt con tem nhng mi bn nam c nhiu nht 4 con tem cn mi bn n c nhiu nht 7 con tem. Bi tp 8 (IMO 1995). Cho p l mt s nguyn t l. Tm s cc tp con A ca tp hp {1, 2, . . . , 2p} tha mn i) Tp A c ng p phn t. ii) Tng cc phn t ca tp A chia ht cho p. Bi tp 9. Chng minh rng s cch thm du ngoc vo vo tch gm n + 1 nhn t l s 1 n Catalan Cn = n+1 C2n . Bi tp 10 (Rookie Contest 1990). Cho n l s nguyn t v a1 , a2 , . . . , am l cc s nguyn dng. Gi f (k)l s cc b m s (c1 , c2 , . . . , cm ) tha mn iu kin 0 ci ai v c1 + c2 + + cm k (mod m). Chng minh rng f (0) = f (1) = = f (n 1) khi v ch khi n | aj vi j no thuc {1, 2, . . . , m}. Bi tp 11. Tm h s ca x18 trong khai trin (1 + x3 + x6 + x9 + )7 Bi tp 12 (AMM 2010). Chng minh rng vi mi s nguyn dng n ta c
n n 2 k

k=0

(2k + 1)

2n 2k

24n (n!)4 (2n)! (2n + 1)!

Ti liu tham kho


[1] Kim nh Sn, Hm sinh. [2] Nguyn Vn Mu (Ch bin), Bin phc, nh l v p dng, NXB Gio dc, 2009. [3] Titu Andreescu, Zuming Feng, A Path to Combinatorics for Undergraduates: Counting Strategies, Birkhuser, 2003. a [4] Titu Andreescu, Zuming Feng, 102 Combinatorial Problems, Birkhuser, 2003. a

68 [5] Daniel A.Marcus, Combinatorics aproblem oriented approach, The Mathematical Association of America, 1999. [6] Philippe Flajolet, Robert Sedgewick, Analytic combinatorics, Cambridge, 2009. [7] John Michael Harris, Jeffrey L. Hirst, Michael J. Mossinghoff, Combinatorics and graph theory, Springer, 2000. [8] Din n MathScope http://forum.mathscope.org/index.php

PHNG PHP HM SINH


L Hu Phc1 Trn Nguyn Quc Cng2

1. C s l thuyt
1.1. Hm sinh
nh ngha 1. Cho mt dy (an )n 0 , khi , hm s f xc nh nh sau: f (x) = c gi l hm sinh bi dy (an )n 0 . R rng cc h s ca f cho ta thng tin ca dy (an )n 0 . Cc php ton. Cho hm f sinh bi dy (an )n 0 v hm g sinh bi dy (bn )n 0 . Khi ta c cc php ton thng dng sau: Php cng: f (x) + g(x) = y l hm sinh ca dy (an + bn )n 0 . Php nhn: f (x) g(x) = Trong ci =
i k=0 i=0 i=0

ai x i

i=0

(ai + bi )xi

c i xi

ak bik .

T y ta cng suy ra c php nhn vi hng s. Php chia: Khi b0 = 0 ta c th nh ngha c php chia: f (x) = f (x) g(x) g(x) Trong g(x) g(x) = 1. iu kin b0 = 0 y l iu kin cn v tn ti g(x).
1 2

SV C nhn Khoa hc ti nng Ton hc K15, HKHTN - HQGHN. Lp 11A1, trng THPT chuyn L Qu n, nng.

69

70 o hm: f (x) =

i=1

iai xi1

Ngoi ra, ta cng thng s dng loi hm sinh khc ca dy (an )n F (x) = xa0 + xa1 + xa2 + Vi kiu hm sinh nh th ny, ta c php ton nhn nh sau: Cho hm F sinh bi (an )n 0 v G sinh bi (bn )n 0 , khi : F (x) G(x) = trong C = {ai + bj |i, j 1}. xc
cC

sau:

1.2. Mt s cng c b sung


nh l 1 (Khai trin ly tha ca mt hm). Cho hm f kh vi v hn ln trn R, khi : f (x) = f (x0 ) + f (x0 ) f (x0 ) (x x0 ) + (x x0 )2 + 1! 2!

nh l 2 (RUF (Root of Unity Filter)). Cho s nguyn dng n, nh ngha = cos 2 + n i sin 2 . Xt a thc n F (x) = a0 + a1 x + a2 x2 + (ak = 0 nu k > deg F ). Khi : a0 + an + a2n + = 1 (F (1) + F () + F (2 ) + + F (n1 ) n

nh l 3. Cho p l mt s nguyn t, t = cos 2 + i sin 2 . Khi , nu: p p a0 + a1 + a2 2 + + ap1 p1 = 0 th a0 = a1 = a2 = = ap1 Cc bn nn t chng minh cc vn trn (khng kh) v thc hin khai trin ly tha i vi mt s hm quen thuc nh ex , ln x, sin x, cos x, (x + 1) vi R, . . .

71

1.3. Khai trin ca mt s hm


1 = 1x
k=0

1 = (1 x)n+1 1 = 1 4x
k=0

k=0

n+k k x k

ln

1 = 1x

k=0

xk k

k=0

2k k x k

e =

k=0

xk k!

sin x =

(1)k

x2k+1 (2k + 1)!

(1 + x) =

k=0

k x k

sin x =

k=0

(1)k

x2k+1 (2k + 1)!

(1 + x) =

k=0

k x k

cos x =

k=0

(1)k

x2k (2k)!

2. Mt s k thut dng hm sinh


Thc ra s dng a thc trong gii ton khng qu xa l, ta c th ly ngay v d nh chng minh
n

i=0

n i

= 2n

ta c th xt F (x) = (1 + x)n v thay x = 1; chng minh:


n

i=0

n i

2n n

ta li xt G(x) = (1 + x)2n v so snh h s ca xn trong hai cch khai trin. Nhng phn sau s ni r hn mt phn ca vic dng a thc gii ton qua cc v d.

2.1. Tm cng thc tng qut ca dy s


V d 1. Tm cng thc tng qut ca dy Fibonacci xc nh bi i) f0 = 1, f1 = 1; ii) fn+2 = fn+1 + fn vi mi n N.

72 Li gii. Xt hm sinh ca dy Fibonacci: F (x) = =


i=0 i=0

f i xi (fi + fi+1 )xi+2 + f1 x + f0

= x2 F (x) + x(F (x) 1) + x + 1 Suy ra: F (x) = Ta phn tch 1 F (x) = 5x Trong 1 =
1+ 5 , 2 2

1 1 x x2

1 5 , 2

nn: 1+ 5 2

1 1 1 1 x 1 2 x
i+1 i=0

F (x) = Vy ta c

i=0

x
n+1

1 5 2

i+1

xi

1+ 5 1 fn = 2 5

1 5 2

n+1

V d 2. Tm cng thc tng qut ca s Catalan Cn xc nh bi cng thc: C0 = 1 v


n1

Cn =
k=0

Ck Cn1k

Li gii. Xt hm sinh: F (x) = Ta c: F (x) =


t=0

t=0

C t xt
t=1

C t xt = 1 +
t1

C t xt

=1+

Ck Ct1k xt

t=1 k=0

= 1 + xF 2 (x) Suy ra: F (x) = Ta khai trin ly tha ca 1 4x: 1 4x = 1 2 2m 2 m x m m1 m=1

1 4x 2x

73 Do F (x) c h s khng m nn ta c: F (x) = = 1 2m 2 m1 x m m1 m=1 2m m 1 x m+1 m m=0


1 Vy Cn = n+1 2n . S Catalan c nhiu ngha v mt t hp v cc cch chng minh khc, n cc bn c th tham kho trong nhiu ti liu.

V d 3. Tm cng thc tng qut ca s hon v derangement Dn .3 Li gii. Ta c cng thc sau: n! =
i=0 n

Dni

n i

Tng ng vi Dn =1 n!
n1 i=0

Tht vy, tp cc hon v ca {1, 2, . . . , n} c th c chia thnh cc tp con, m trong mi tp con , mi hon v c ng i im c nh. S phn t ca mi tp con s bng cch chn ra i im c nh v hon v derangement n i phn t cn li. t ai = Di . Xt hm sinh: i! F (x) =
t=0

Di i! (n i)!

at x t = 1 +
t=1

at x t ai (t i)! 1 x = 1x
t t1

t=1 t1

=1+

1
i=0

1 = 1x

Vy ta c

1 = (ex 1)F (x) 1x F (x) = =

1 ai xt = (t i)! 1x t=i+1

i=0

t=1 i=0

ai x

i=0

xti (t i)! t=i+1

ai xt (t i)!

ex 1x

xi

i=0

i=0

(x)i i!

Bng php nhn hai hm, d dng suy ra:


n

an =
i=0

(1)i i! (1)i i!
i n.

Tng ng Dn = n!

i=0

Hon v ca {1, 2, . . . , n} c gi l derangement nu (i) = i vi mi 1

74

2.2. Tnh cc tng t hp


V d 4. Cho s t nhin n. Tnh tng sau: A= Li gii. t = cos 2 + i sin 2 . Khi : 3 3 A= (1 + )n + (1 + 2 )n + (1 + 1)n 3 (1)n (2n + n ) + 2n = 3 A= Nu n khng chia ht cho 3: A= 2n + 2(1)n 3 2n + (1)n+1 3 V d 5. Cho s t nhin n. Tnh tng: n
k k=0

n 3k

Nu n chia ht cho 3:

k 2

xk

Li gii. Ta c:

k=0

n
k 2

x =

i=0

n 2i x + i
2 n

i=0

n 2i+1 x i
n

= (1 + x ) + x(1 + x2 ) = (x + 1)(1 + x2 )
n

V d 6. Cho hai s nguyn dng n v m tha mn n


n

m. Tnh tng:

k=m

n k

k m

Li gii. (i) Cch 1. Ta c ng thc sau: n k Nh vy:


n

k m

n m

nm km
n

k=m

n k

k m

n m

k=m

= 2nm

n m

nm km

75 (ii) Cch 2. t an =
k=m

n k

k m

Xt hm: f (x) = = =
n=0

an x =

n=0 k=m

n k

k n x m

k=m n=k k=m

n k

n k k k n xnk x x = nk m m n=k k=m


k=m

1 xm = m+1 x (1 x) 1 1x xm = (1 2x)m+1

xk k xm = m (1 x)k+1 (1 x)m+1
m+1

xkm k k m (1 x)km

Vy h s ca xn trong khai trin ca f l: an = n nm 2 m V d 7.


n

(1)k
k=m

n k

k m
(1+x)k . xm

Li gii. (i) Cch 1. Ta c

k m

l h s t do trong khai trin


n

S= th S l h s t do trong khai trin:


n

(1)k
k=m

n k

k m

(1)k
k=m

(1)n n (1 + x)k = xm xm k

k=0

n (1)nk (1 + x)k k

(1)n xn = xm Do : i) nu m = n th S = (1)n ; ii) nu m = n th S = 0. (ii) Cch 2. t: am =


k=m n

(1)k

n k

k m

76 Xt hm sinh: F (x) = = = = Suy ra F (x) = (x)n Ta thu c kt qu tng t li gii trc. V d 8. Tnh tng:
n

m=0

am x m
n

(1)k (1)
k

m=0 k=m m=0 m=0

n k
n

k m x m k m x m

n k

k=m

(1)k

n (1 + x)k k

n r n r 2n 2r n1 2n 2r k

(1)r
r=0

Li gii. (i) Cch 1. t: ak =

(1)r
r=0

Xt hm sinh: F (x) = = =

k=0

ak x k
n

(1)r (1)
r

k=0 r=0 n

n r
k=0

2n 2r k x k 2n 2r k x k

r=0 n

n r

=
r=0

(1)r

n (1 + x)2n2r r

= (x2 + 2x)n Suy ra F (x) = xn (x + 2)n Vy an1 = 0. (ii) Cch 2. Ta c


2n2r n1

l h s t do trong khai trin


n

(1+x)2n2r , xn1

do :

(1)r
r=0

n r

2n 2r n1

l h s t do trong khai trin: 1 xn1


n

(1)r
r=0

(x2 + 2x)n n (1 + x)2n2r = r xn1 = x(x + 2)n

77 M h s t do ca biu thc ny bng 0, ta thu c cng kt qu vi li gii trc.

Trong li gii 1, ta thay lng n 1 thnh bin mi l k, iu ny lm iu cn chng minh tr nn nhiu hn (tng qut hn) nhng li lm tng tnh c lp cho cc bin.

2.3. Cc bi ton m
V d 9. Cho s t nhin n v s nguyn dng m. Tnh s nghim t nhin (x1 , x2 , . . . , xm ) ca phng trnh x1 + x2 + + xm = n Li gii. Gi s nghim ca phng trnh trn l fm,n th n chnh l h s ca y n trong khai trin ca: m 1 i y = F (y) = (1 y)m i=0 S dng khai trin ly tha ta c: F (y) = Cho y0 = 0 ta c: F (y) = T suy ra: fm,n = m+n1 m1
i=0

F (i) (y0 ) (y y0 )i i!

i=0

yi

m+i1 m1

Trong nhiu trng hp, xi nhn gi tr thuc tp Ai N, khi ta xt hm sinh:


m

F (y) =
i=1 aAi

ya

V d 10. Andrian c cc ng xu C1 , C2 , . . . , Cn . Vi mi k, Ck c c sao cho khi tung 1 ln, xc sut n xp l 2k+1 . Tung tt c n ng xu, tnh xc sut s ng xp l l (biu din dng rt gn). Li gii. Xc sut s ng xp l l l tng h s bc l ca a thc:
n

P (x) =
k=1

2k x + 2k + 1 2k + 1

Tng h s ny bng n P (1) P (1) = 2 2n + 1

78 V d 11. Tm tt c n sao cho c th t mu xanh v cho tp S = {1, 2, . . . , n} S 3 cha ng 2007 b (x, y, z) m x, y, z cng mu v x + y + z chia ht cho n. Li gii. Gi cc s c t mu l a1 , a2 , . . . , au v cc s c t mu xanh l b1 , b2 , . . . , bv . Xt hai a thc: U (x) = xa1 + xa2 + + xau V (x) = xb1 + xb2 + + xbv Nh vy s b s tha mn bi chnh l h s ca cc n thc c s m chia ht cho n trong khai trin U 3 (x) + V 3 (x). Ta c: U 3 (x) + V 3 (x) = (U (x) + V (x))((U 2 (x) U (x)V (x) + V 2 (x)) rng: U (x) + V (x) = x + x2 + x3 + + xn Vi mi n thc trong khai trin ca U 2 (x) U (x)V (x) + V 2 (x) s tn ti duy nht mt n thc trong U (x) + V (x) sao cho tch ca chng l mt n thc c s m chia ht cho n. T suy ra tng cc b s tha mn bi l: U 2 (1) U (1)V (1) + V 2 (1) = u2 uv + v 2 V d 12. Vi mt tp hp A, nh ngha s(A) l tng tt c phn t ca A (nu A = th s(A) = 0). t S = {1, 2, . . . , 1999} Vi 0 r 6, nh ngha: Tr = {T |T S, s(T ) r Vi mi r, tnh s phn t ca Tr . Li gii. Xt hm sinh: F (x) =
i=1 1999

(mod 7)}

(xi + 1)

t = cos

2 7

+ i sin

2 , 7

ta c: F () = 2285 (1 + )(1 + 2 )(1 + 3 )(1 + 4 ) = 2285 (1 + 3 )

Mt khc, ta vit
6

F () =
r=0

|Tr |r

Nh vy T0 2285 = T1 = T2 = T3 2285 = T4 = T5 = T6

79 M T0 + T1 + T2 + T3 + T4 + T5 + T6 = 21999 Suy ra 21999 2286 7 21999 + 5 2285 T0 = T3 = 7

T1 = T2 = T4 = T5 = T6 =

y, ta c th tnh T0 mt cch n gin bng RUF nh sau: 1 T0 = 7 = 2


6

F (i )
i=0 1999

+ 5 2285 7

Cng bng cch ny, ch cn mt cht iu chnh n gin, ta c th tnh c cc Tr khc, cc bn nn t thc hin vic ny. V d 13. Cho p l mt s nguyn t l v t A = {1, 2, . . . , 2p}. Tnh s tp con ca A c p phn t v tng cc phn t ca n chia ht cho p. Li gii. t = cos 2 + i sin 2 . Xt a thc: p p
2p

F (x) =
i=1

(x i )

= x2p 2xp + 1 Ta vit: F (x) =


i=1 p1 2p

= (xp 1)2

(1)i ai xi Ai,t t
t=0

ai =

Khi ta s thy rng Ai,t chnh l s tp con c 2p i phn t v tng cc phn t ng d t theo modulo p. Ta c
p1

ap =
t=0

Ap,t t = 2

Do Ap,t u l s nguyn nn: Ap,0 2 = Ap,1 = Ap,2 = = Ap,p1 Li c Ap,0 + Ap,1 + Ap,2 + + Ap,p1 = 2p p

80 Suy ra Ap,0 =
2p p

+ 2(p 1) p

V d 14. Cho p l mt s nguyn t l v n l mt s nguyn dng khng chia ht cho p. Tnh s tt c cc b (x1 , x2 , . . . , xp1 ) {0, 1, . . . , n 1}p1 sao cho:
p1

ixi
i=1

chia ht cho p. Li gii. Xt hm sinh:


p1

F (x) =
i=1

(1 + xi + x2i + + x(n1)i )

Ta c: F (1) = np1 t = cos 2 + i sin 2 . Khi , vi mi 1 p p F ( ) =


i=1 j

j
p1

p 1: 1 nij 1 ij

=1 Theo nh l RUF ta c s b cn tnh l: 1 p


p1

F (j ) =
j=0

np1 + p 1 p

2.4. Cc bi ton chng minh


V d 15. Xt cc cp (ai , bi ) Z+ N vi 1 i k, (k 2). Bit rng vi mi s nguyn khng m n, tn ti duy nht mt ch s i sao cho n bi (mod ai ). Chng minh rng: (a) Tn ti i = j sao cho ai = aj ; (b) (c)
k i=1 k i=1 1 ai

= 1,
n1 . 2

bi ai

81 Li gii. Theo bi, ta c:


k

i=1

xbi (1 + xai + x2ai + ) = 1 + x + x2 +


k

Tng ng
i=1

Hay
k i=1

xb i 1 = 1 xai 1x

xb i =1 1 + x + x2 + + xai 1

(a) Gi s phn chng, cc ai l i mt phn bit. Vi mi i t 1 n k t: ri = 1 + x + x2 + + xai 1 v A=


i=1 k

ri

Ta c:

xb i
i=1

A =A ri

t ak = max{ai |1 i k}, = cos 2 + i sin 2 . Nh vy ta c l nghim ca rk v ak ak khng l nghim ca tt c cc ri cn li, iu ny cho ta mu thun vi biu thc trn. (b) Thay x = 1 trong biu thc:
k

i=1

xb i =1 1 + x + x2 + + xai 1
k

Ta c
i=1

1 =1 ai

(c) Ly o hm ca biu thc:


k

i=1

xb i =1 1 + x + x2 + + xai 1

ta c:
k

i=1

bi xbi 1 (1 + x + + xai 1 ) xbi (1 + 2x + + (ai 1)xai 2 ) =0 (1 + x + + xai 1 )2

Lc ny thay x = 1 ta thu c:
k
i bi ai ai (a21) =0 a2 i

i=1

82 Suy ra:
k i=1

k1 bi = ai 2

V d 16. Tm hm sinh ca dy s (pn )n dng n.

trong pn l s cch phn hoch s nguyn vi nh

Li gii. Trong mt phn hoch ca n, ta xc nh mt dy cc s t nhin (ai )i ngha trong phn hoch c ng ai phn c gi tr bng i. Nh vy d thy rng hm sinh ca dy (pn )n 1 s l: F (x) = (1 + x + x2 + )(1 + x2 + x4 + )(1 + x3 + x6 + ) = 1 (1 x)(1 x2 )(1 x3 ) . . .

V d 17. Chng minh rng s cch phn hoch n thnh cc phn i mt phn bit bng s cch phn hoch n thnh cc phn l. Li gii. S phn hoch n thnh cc phn i mt phn bit l h s ca xn trong khai trin P (x) = (1 + x)(1 + x2 )(1 + x3 ) . . . S cch phn hoch n thnh cch phn l l h s ca xn trong khai trin Q(x) = (1 + x + x2 + )(1 + x3 + x6 + )(1 + x5 + x10 + ) . . . Ta c: 1 x2 1 x4 1 x6 ... 1 x 1 x2 1 x3 1 1 ... = 1 x 1 x3 1 1 Q(x) = ... 1 x 1 x3 P (x) =

Vy P (x) Q(x), suy ra iu cn chng minh.

3. Bi tp
Bi tp 1. Cho s t nhin n. Tnh tng:
i=0

n i

n i n2i 2 i

Bi tp 2. Cho s t nhin n. Tnh tng:


n

i=0

n i

ni
ni 2

83 Bi tp 3. Cho m, n l hai s t nhin. Tnh tng:


k=0

n k

nk
mk 2

Bi tp 4. Cho s t nhin n. Tnh tng:


n

k=0

2n 2k

2k 2n2k 2 k

Bi tp 5. Cho m, n l hai s nguyn dng. Chng minh rng:


m

k=0

2m k k 2 = 4m m+n

j=1

2m + 1 m+j

Bi tp 6. Tm cng thc tng qut ca dy s (an,k ) xc nh nh sau: an,0 = 2n+1, a0,k = 1, vi n 0 v k 1, ta c: an+1,k = an,k + an,k1 + + an,0 Bi tp 7. Cho s t nhin n. Bit rng cc phng trnh ix + (i + 1)y = n + 1 i c ng Ri nghim (x, y) N2 vi mi 1 i n + 1. Chng minh rng:
n+1

Ri = n + 1
k=1

Bi tp 8. Cho dy (an ) xc nh bi a1 = 0, a2 = 1, v vi mi n an = Tnh tng: an + 2 n n a1 an1 + + n n1 1

3:

nan1 n(n 1)an2 (1)n1 n + + + (1)n 2 2 2

Bi tp 9. m s ng i vi k bc i i t (0, 0) n (m, n) sao cho mi bc i l i t (i, j) n ng mt trong bn v tr (i 1, j), (i + 1, j), (i, j 1), (i, j + 1). Bi tp 10. Cho 2n s thc phn bit a1 , a2 , . . . , an v b1 , b2 , . . . , bn . Bng n n c in s theo quy tc: hng i ct j th c in s ai + bj . Chng minh rng tch cc s mi ct l bng nhau th tch cc s mi hng cng bng nhau. Bi tp 11. Chng minh rng s phn hoch mt s nguyn dng n thnh cc phn m cc phn chn i mt phn bit bng s cch phn hoch n thnh cc phn m mi gi tr xut hin khng qu 3 ln. Bi tp 12. Gi an l s cch biu din n di dng tng c th t ca cc s 1 v s 2, bn l s cch biu din n di dng tng c th t ca cc s khng nh hn 2. Chng minh rng an = bn+2 .

84 Bi tp 13. Cho hai tp A = {ai |i = 1, n}, B = {bi |i = 1, n} phn bit. Xc nh hai tp A2 = {ai + aj |1 i < j n}, B2 = {bi + bj |1 i < j n}. Gi s ta c A2 = B2 . Chng minh rng n l mt ly tha ca 2. Bi tp 14. Cho tp E = {1, 2, . . . , 2008}, mi s thuc E c t bi ng 1 trong 3 mu , vng, xanh. Gi A l s b (x, y, z) E 3 m x, y, z cng mu v x + y + z chia ht cho 2008, B l s b (x, y, z) E 3 m x, y, z i mt khc mu v x + y + z chia ht cho 2008. Chng minh rng 2A > B. Bi tp 15. Xt mt phn hoch ca tp cc s nguyn khng m thnh hai tp A, B sao cho, vi mi s nguyn khng m n, s cch biu din n = a1 + a2 (a1 , a2 A, a1 = a2 ) bng s cch biu din n = b1 + b2 (b1 , b2 B, b1 = b2 ). Chng minh rng cch phn hoch ny tn ti v duy nht. Bi tp 16. Xt mt phn hoch ca s nguyn dng n. Gi () l s s 1 trong phn hoch , () l s phn phn bit trong phn hoch . Chng minh rng () = (tng ly theo tt c cc phn hoch ca n) Bi tp 17. Cho s nguyn dng n 3. K hiu [n] := {1, 2, . . . , n}, i() l s cp nghch th trong hon v ca [n], v ak l s hon v ca [n] sao cho i() = k. Chng minh rng: a3k+1 =
k 0 k 0

()

a3k+2

Bi tp 18. Cho s nguyn t l p. Chng minh rng trong 2p 1 s nguyn bt k, lun tn ti p s c tng chia ht cho p. Bi tp 19. Mt dy s thc a1 , a2 , . . . , an gi l p - cn bng nu ta c tng ak + ak+p + ak+2p + bng nhau vi mi 1 k p. Chng minh rng nu mt dy c 50 phn t l p cn bng vi p bng 3, 5, 7, 11, 13, 17 th dy gm ton cc s 0. Bi tp 20. Cho hai h tp hp (An )n 1 , (Bn )n i) A1 = , B1 = {0}; ii) An = {x + 1|x Bn1 }, Bn = An1 Bn1 An1 Bn1 . Tm tt c s nguyn dng n sao cho Bn = {0}.
1

tha mn:

4. Li gii
Bi tp 1. Cho s t nhin n. Tnh tng:
i=0

n i

n i n2i 2 i

85 Li gii. Ta c: an = l h s t do trong khai trin:


i=0

i=0

n i

n i n2i 2 i

n 2n2i (1 + x)ni = i xi

i=0

n (2 + 2x)ni (2x)i i
n

1 = 2x + 2 + 2x 2n (2x + 1) = (2x)n Do suy ra: an = 2n n

Bi tp 2. Cho s t nhin n. Tnh tng:


n

i=0

n i

ni
ni 2 1 ni . x

Li gii. Ta c ni l h s t do trong khai trin ca (x + 1) x + ni 2 Do : n nk k 2 nk k 2 k=0 l h s t do trong khai trin:


k=0

1 n (1 + x) x + k x

nk

2k = (1 + x) x + = (1 + x)2n+1 xn 2n + 1 n

1 +2 x

Suy ra:

k=0

n k

nk
nk 2

2k =

Bi tp 3. Cho m, n l hai s t nhin. Tnh tng:


k=0

n k

nk
mk 2

nk Li gii. Ch rng mk l h s t do trong khai trin ca: 2

y nm (1 + y) y +

1 y

nk

86 Do :

k=0

n k

nk k x mk 2
k=0 nm nk

l h s t do trong khai trin theo y ca:


k=0

1 n nm y (1 + y) y + y k

nk

x = (1 + y)y

nm

n k

1 y+ y
n

xk

1 x+y+ = (1 + y)y y 2 n (1 + y)(xy + y + 1) = ym T ta c c cch tnh tng cho. Bi tp 4. Cho s t nhin n. Tnh tng
n

k=0

2n 2k

2k 2n2k 2 k

Li gii. (i) Cch 1. Ta c: 2n 2k 2k k = (2n)! (2k)! (2k)!(2n 2k)! (k!)2 (2n)! (2n k)! = (2n k)!k! (2n 2k)!k! 2n 2n k = k k 2n 2k 2k 2n2k 2 k

Do :

k=0

l h s t do trong khai trin:


2n

k=0

2n 22n2k (x + 1)2nk 1 2n = (2x + 2)2n k 2 + 4x)k k k (4x x k=0 = (2x + 2) =


2n

2n

1 1+ 2 4x + 4x

2n

(2x + 1)4n (2x)2n 4n 2n Nh vy ta c:

Tc l:

k=0

2n 2k

2k 2n2k 2 = k

(ii) Cch 2. Ta c

2k k

l h s t do trong khai trin


n

(x+1)2k . xk

k=0

2n 2k

2k 2n2k 2 k

87 l h s t do trong khai trin ca


n

k=0

2n (1 + x)2k 2n2k 2 xk 2k

tc l h s t do trong khai trin: 2


2n1

1+x 1+ 2 x
4n 2n

2n

1+x + 1 2 x

2n

(1 +

x)4n + (1 2xn

x)4n

Ta c p s l

(iii) Cch 3. Xt hm sau: F (x) = = Ta c:


n=0 k=0 n

n k=0

2n 2k
n=k

2k 2n2k 2 k 2n (2 x)2n 2k 1

1 2k 22k k

1 2n (2 x)2n = (2 x)2k 2 2k

Nh vy: 1 F (x) = 2(1 2 x) 1 = 2(1 2 x) 1


k=0

(1 2 x)2k+1
k

(1 + 2 x)2k+1

2k k 1

x (1 2 x)2

1 + 2(1 + 2 x) 1 1

k=0

2k k

x (1 + 2 x)2

1 + 2 14 x 2 1+4 x 2n n 1 2n 1 x + ( x)n = 2 n=0 n 2 n=0 n =


n=0

4x (12 x)2

1 + 2(1 + 2 x)

4x (1+2 x)2

4n n x 2n

Ta thu c iu cn chng minh. Bi tp 5. Cho m, n l hai s nguyn dng. Chng minh rng:
m

k=0

2m k k 2 = 4m m+n
2m+1

j=1

2m + 1 m+j

Li gii. Ta c: 22m+1 =

i=0 2m+1

2m + 1 i 2m + 1 i

=2
i=m+1

88 Suy ra:
2m+1

4 =
i=m+1

2m + 1 i

Nh vy ta c:
n

4 Ta t:

j=1

2m + 1 m+j

2m+1

=
j=m+n+1

2m + 1 j

2m+1

at =
j=t+1

2m + 1 j
t=0

Xt hm sinh: F (x) = Vy: F (x) =


2m+1 t=0 j=t+1 2m+1 j1

at x t

2m + 1 t x j 2m + 1 t x j

=
j=1 t=0 2m+1

=
j=1

2m + 1 xj 1 j x1
2m+1

1 = x1 =

j=0

2m + 1 j x j

2m+1

j=0

2m + 1 j

1 (1 + x)2m+1 22m+1 x1 = (1 + x)2m + 2(1 + x)2m1 + + 22m

T ta c h s ca xt trong khai trin ca F l:


2m

at =
j=0

j 2mj 2 t

Suy ra ng thc cn chng minh.

Bi tp 6. Tm cng thc tng qut ca dy s (an,k ) xc nh nh sau: an,0 = 2n+1, a0,k = 1, vi n 0 v k 1, ta c: an+1,k = an,k + an,k1 + + an,0 Li gii. Ta c: an+1,k+1 = an,k+1 + an,k + + an,0 = an,k+1 + an+1,k

89 Xt hm sinh: F (x, y) = = = = Ngoi ra ta c:


an,k xn y k
n k=0 k=0 k=0

n=0 k=0 n=0 n=0 n=0

an,0 x + an,0 xn + an,0 x +


n

a0,k y 1 + a0,k y k 1 +
k

an,k xn y k (an1,k + an,k1 )xn y k an,k x y + x


n k

n=1 k=1

a0,k y 1 + y

n=1 k=1 n=1 k=0

an,k xn y k

n=0 k=1

n=0

an,0 x =

(2n + 1)xn (n + 1)xn


n=0

=2

n=0 n=0

xn

k=0

1 2 = 2 (1 x) 1x a0,k y = =
k

yk

k=0

1 1y

n=1 k=0

an,k xn y k = y F (x, y)

k=0

a0,k y k

y = yF (x, y) 1y an,k xn y k = x F (x, y)

an,0 xn

n=0 k=1

n=0

Nh vy ta suy ra: F (x, y) =

x2 + x = xF (x, y) (1 x)2 1+x (1 x)(1 x y)

1+x = 1x 1+x = 1x =
n

(x + y)t y
k t=k

t=0 k=0 k=0

t tk x k

1+x 1x

yk (1 x)k+1 tc l:

Vy an,k bng h s ca x trong khai trin ca an,k =

1+x (1x)k+2

n+k+1 n+k + k+1 k+1

90 Bi tp 7. Cho s t nhin n. Bit rng cc phng trnh ix + (i + 1)y = n + 1 i c ng Ri nghim (x, y) N2 vi mi 1 i n + 1. Chng minh rng:
n+1

Ri = n + 1
k=1

Li gii. Ta c Ri l h s t do trong khai trin ca: 1 xi )(1 xi+1 ) (1 xi+1 ) (1 xi ) = (1 x)xn+1 (1 xi )(1 xi+1 ) 1 1 1 = n+1 x (1 x) 1 xi 1 xi+1 xn+1i (1 Nh vy:
n+1

Ri
k=1

l h s t do trong khai trin: 1 xn+1 (1 x) = 1

1 1 1 x 1 xn+2 xk
k=0

xn+1

k=0

xk

xk(n+2)

k=0

tc l h s t do trong khai trin: 1 (1 + x + x2 + + xn )2 n x T ta c:


n+1

Ri = n + 1
k=1

Bi tp 8. Cho dy (an ) xc nh bi a1 = 0, a2 = 1, v vi mi n 3:

nan1 n(n 1)an2 (1)n1 n an = + + + (1)n 2 2 2 Tnh tng an + 2 Li gii. B sung thm a0 = 1. t bn = an . Khi : n! n n an1 + + n a1 1 n1

(1)n1 bn1 bn2 (1)n + + + bn = 2 2 n! 2(n 1)!

91 Khi , xt: F (x) = Ta c: x2 + bk xk F (x) = 1 + 2 k=3 =1+ =1+ Suy ra: F (x) = Ta tnh tng: an + 2
n

k=0

bk xk

x2 + 2 k=3

x2 x2 x x x2 x + (F (x) 1) + (F (x) 1) + ex 1 + x + (e 1 + x) 2 2 2 2 2 ex 1x

bk1 bk2 (1)k (1)k1 + + + 2 2 k! 2(k 1)!

xk

=n!
i=0

(i + 1)bni i!

n n n an1 + + n a1 + (n + 1) a0 1 n1 n

D thy:

i=0

(i + 1)bni i!

l h s ca xn trong khai trin ca:

bk xk

k=0

e (xex + ex ) = 1x 2x =1 + 1x =1 + 2x + 2x2 + Vy ta c kt qu: an + 2 n n a1 = 2n! n 1 an1 + + n n1 1 Bi tp 9. m s ng i vi k bc i i t (0, 0) n (m, n) sao cho mi bc i l i t (i, j) n ng mt trong bn v tr (i 1, j), (i + 1, j), (i, j 1), (i, j + 1). Li gii. iu kin tn ti ng i l k + m + n chn v k Xt hm: k 1 1 f (x, y) = x + y + + x y m + n.

k=0 x

k+1 k x k!

92 H s ca xm y n trong f chnh l s ng i cn tm. Ta c: f (x, y) = (x + y) = (x + y)


k

1 1+ xy
k

k t=0

k 1 t t (xy)

Nh vy ta cn tm h s ca xr y s trong (x + y)k vi r, s tha mn r s = m n, r + s = k. Tc l: k+mn r= 2 km+n s= 2 T suy ra h s ca xm y n trong f l: k


k+mn 2

k
kmn 2

Ta c kt qu cn tm.

Bi tp 10. Cho 2n s thc phn bit a1 , a2 , . . . , an v b1 , b2 , . . . , bn . Bng n n c in s theo quy tc: hng i ct j th c in s ai + bj . Chng minh rng tch cc s mi ct l bng nhau th tch cc s mi hng cng bng nhau. Li gii. Xt a thc sau: P (x) = (x + a1 )(x + a2 ) (x + an ) U Trong U = (a1 + b1 )(a2 + b1 ) (an + b1 ). Theo bi th P (x) c n nghim l b1 , b2 , . . . , bn m P (x) bc n nn ta phi c: P (x) = (x b1 )(x b2 ) (x bn ) Nh vy ta suy ra vi mi i = 1, n ta c: P (ai ) = (1)n (ai + b1 )(ai + b2 ) + + (ai + bn ) Mt khc, P (ai ) = U . T suy ra:
n n n

(a1 + bj ) =
j=1 j=1

(a2 + bj ) = . . . =
j=1

(an + bj )

Nh vy tch tt c cc s mi hng cng bng nhau.

Bi tp 11. Chng minh rng s phn hoch mt s nguyn dng n thnh cc phn m cc phn chn i mt phn bit bng s cch phn hoch n thnh cc phn m mi gi tr xut hin khng qu 3 ln.

93 Li gii. S cch phn hoch n thnh tng ca cc phn m cc phn chn i mt phn bit l h s ca xn trong khai trin: P (x) = (1 + x + x2 + )(1 + x2 )(1 + x3 + x6 + )(1 + x4 ) . . . S cch phn hoch n thnh tng cc phn m mi gi tr xut hin khng qu 3 ln l h s ca xn trong khai trin: Q(x) = (1 + x + x2 + x3 )(1 + x2 + x4 + x6 ) . . . Ta c: (1 x4 )(1 x8 ) . . . (1 x)(1 x2 )(1 x3 )(1 x4 ) . . . (1 x4 )(1 x8 ) . . . Q(x) = (1 x)(1 x2 ) . . . P (x) =

Vy P (x) Q(x), ta c iu cn chng minh.

Bi tp 12. Gi an l s cch biu din n di dng tng c th t ca cc s 1 v s 2, bn l s cch biu din n di dng tng c th t ca cc s khng nh hn 2. Chng minh rng an = bn+2 . Li gii. Xt hai hm sau: F (x) =
k=1

ak x =

k=1

(x + x2 )k

1 = 1 1 x x2 G(x) =

bk x =

k=2

k=1

(x2 + x3 + )k

x2 = 1 x x2 Suy ra: G(x) = x2 F (x) + x2 Ta c ngay pcm.

Bi tp 13. Cho hai tp A = {ai |i = 1, n}, B = {bi |i = 1, n} phn bit. Xc nh hai tp A2 = {ai + aj |1 i < j n}, B2 = {bi + bj |1 i < j n}. Gi s ta c A2 = B2 . Chng minh rng n l mt ly tha ca 2. Li gii. t: A(x) =
i=1 n n

xai xb i
i=1

B(x) =

94 Ta c: xa =
aA2

1 2 A (x) A(x2 ) 2 1 B 2 (x) B(x2 ) 2

xb =
bB2

Vy suy ra: A2 (x) A(x2 ) = B 2 (x) B(x2 ) tng ng: (A(x) B(x))(A(x) + B(x)) = A(x2 ) B(x2 ) Do A(1) = B(1) = n nn ta c th vit A(x) B(x) = (x 1)k P (x) trong P l mt a thc sao cho P (1) = 0 v k Z+ . Khi : P (x)(A(x) + B(x)) = (x + 1)k P (x2 ) Thay x = 1 ta c n = 2k1 . Yu cu bi ton c chng minh xong.

Ta c th chng minh c nu n l mt ly tha ca 2 th s tn ti hai tp A v B tha mn gi thit bng quy np, cc bn c th th t chng minh. Bi tp 14. Cho tp E = {1, 2, . . . , 2008}, mi s thuc E c t bi ng 1 trong 3 mu , vng, xanh. Gi A l s b (x, y, z) E 3 m x, y, z cng mu v x + y + z chia ht cho 2008, B l s b (x, y, z) E 3 m x, y, z i mt khc mu v x + y + z chia ht cho 2008. Chng minh rng 2A > B. Li gii. Gi cc s trong E c nh ln lt mu l a1 , a2 , . . . , am , mu vng l b1 , b2 , . . . , bn , mu xanh l c1 , c2 , . . . , cp . Xt cc a thc: A(x) = xa1 + xa2 + + xam B(x) = xb1 + xb2 + + xbn C(x) = xc1 + xc2 + + xcp

Khi s b (x, y, z) cng mu tha mn tng ca chng chia ht cho 2008 chnh l h s ca x2008 trong khai trin ca (A(x))3 + (B(x))3 + (C(x))3 . S nhng b khc mu l h s ca x2008 trong khai trin ca 6A(x)B(x)C(x). 2(A3 (x) + B 3 (x) + C 3 (x) 3A(x)B(x)C(x))

=(A(x) + B(x) + C(x))((A(x) B(x))2 + (B(x) C(x))2 + (C(x) A(x))2 ) =(x + x2 + + x2008 )((A(x) B(x))2 + (B(x) C(x))2 + (C(x) A(x))2 ) Ta cn chng minh tng h s ca cc n thc c s m chia ht cho 2008 trong biu thc trn l dng. Tht vy, vi mi n thc trong ((A(x) B(x))2 + (B(x) C(x))2 + (C(x) A(x))2 ), tn ti duy nht mt n thc trong (x + x2 + + x2008 ) sao cho tch ca chng l mt n thc c s m chia ht cho 2008, nh vy ta c h s ca cc n thc c s m chia ht cho 2008 trong

95 khai trin ban u bng tng h s ca ((A(x) B(x))2 + (B(x) C(x))2 + (C(x) A(x))2 ) tc l (A(1) B(1))2 + (B(1) C(1))2 + (C(1) A(1))2 M r rng lng ny khng m v ng thc xy ra khi v ch khi m = n = p nhng r rng iu ny v l do m + n + p = 2008. Vy ta c 2A > B. Bi tp 15. Xt mt phn hoch ca tp cc s nguyn khng m thnh hai tp A, B sao cho, vi mi s nguyn khng m n, s cch biu din n = a1 + a2 (a1 , a2 A, a1 = a2 ) bng s cch biu din n = b1 + b2 (b1 , b2 B, b1 = b2 ). Chng minh rng cch phn hoch ny tn ti v duy nht. Li gii. Gi s tn ti mt cch phn hoch tha mn bi. Khng mt tng qut, ta cho rng 0 A. Xt hai hm sau: F (x) = xa
aA

G(x) =
bB

xb

Theo ta c:

F 2 (x) F (x2 ) = G2 (x) G(x2 ) T suy ra:

F (x) + G(x) = 1 + x + x2 +

F (x) G(x) = (1 x)(F (x2 ) G(x2 )) t P (x) = F (x) G(x), h s t do ca P bng 1. Nh vy, lp lun tng t nh bi trc, ta c: P (x) = (1 x)(1 x2 )(1 x4 ) . . . T ta suy ra tp A gm cc s nguyn khng m m biu din nh phn ca s c mt s chn s 1, tp B gm cc s nguyn khng m m biu din nh phn ca s c mt s l s 1. D dng kim tra c cch phn hoch ny tha mn bi. Nh vy ta c iu phi chng minh. Bi tp 16. Xt mt phn hoch ca s nguyn dng n. Gi () l s cc s 1 trong phn hoch , () l s phn phn bit trong phn hoch . Chng minh rng () = (tng ly theo tt c cc phn hoch ca n) Li gii. Xt: P (y) = (1 + xy + x2 y 2 + )(1 + x2 + x4 + )(1 + x3 + x6 + ) ()

Q(y) = (1 + xy + x2 y + )(1 + x2 y + x4 y + )(1 + x3 y + x6 y + )

96 Ta thy rng tng s s 1 trong tt c phn hoch ca n l h s ca xn trong P (1) v tng s phn phn bit trong tt c phn hoch ca n l h s ca xn trong Q (1). Ta c: 1 P (y) = 1 xy Q(y) = = nh ngha: R= Khi : P (y) = Q (y) = Suy ra:
m=1

1 1 xm m=2 y 1 xm

1y+

1 xm + yxm 1 xm m=1 1 1 xm m=2

x R (1 xy)2

1 xm + yxm 1 xm m=1 x R (1 x)2

m=1

xm 1xm yxm + 1xm

P (1) =

Vy ta c iu cn chng minh.

1 Q (1) = 1 xm m=1 x R = (1 x)2

m=1

xm

Bi tp 17. Cho s nguyn dng n 3. K hiu [n] := {1, 2, . . . , n}, i() l s cp nghch th trong hon v ca [n], v ak l s hon v ca [n] sao cho i() = k. Chng minh rng: a3k+1 =
k 0 k 0

a3k+2

Li gii. Vi mi hon v ca [n] ta xc nh mt dy x1 , x2 , . . . , xn1 vi xi c nh ngha l s ch s 1 t i m (t) > (i + 1), t 0 xi i. Ta "d" thy rng c mt tng ng song nh gia tp cc dy (xi ) v tp cc hon v ca [n]. Nh vy: F (x) = xi()
Sn

t = cos 2 + sin 2 , ta c: 3 3

= (1 + x)(1 + x + x2 ) . . . (1 + x + x2 + + xn1 ) F (1) + 2 F () + F (2 ) 3 F (1) + F () + 2 F (2 ) 3

a3k+1 =
k 0

a3k+2 =
k 0

97 M F () = F (2 ) = 0 nn ta c ngay iu cn chng minh.

Bi tp 18. Cho s nguyn t l p. Chng minh rng trong 2p 1 s nguyn bt k, lun tn ti p s c tng chia ht cho p. Li gii. Gi 2p 1 s nguyn l a1 , a2 , . . . , a2p1 . Gi s phn chng rng: tng ca p s bt k trong ny u khng chia ht cho p. T , theo nh l nh Fermat: (ai1 + ai2 + + aip )p1 1 vi 1 i1 < i2 < . . . < ip Suy ra:
(i1 ,i2 ,...,ip )

(mod p)

2p 1. (ai1 + ai2 + + aip )p1 2p 1 p (mod p)

Ta c

2p1 p

khng chia ht cho p, by gi ta s chng minh: (ai1 + ai2 + + aip )p1

(i1 ,i2 ,...,ip )

chia ht cho p. V (ai1 + ai2 + + aip )p1 = nn (ai1 + ai2 + + aip )p1 = =
x1 +x2 ++xp =p1 x1 +x2 ++xp =p1

p1 x ax11 ax22 . . . aipp i i x1 , x2 , . . . , xp p1 x ax11 ax22 . . . aipp i i x1 , x2 , . . . , xp ax11 ax22 . . . aipp i i


(i1 ,i2 ,...,ip ) x

(i1 ,i2 ,...,ip )

(i1 ,i2 ,...,ip ) x1 +x2 ++xp =p1

p1 x1 , x2 , . . . , xp

(cc tng hng di ly theo cc tp khng th t) x Ta m s ln xut hin ca gi tr ax11 ax22 . . . aipp trong tng. i i Trong cc s x1 , x2 , . . . , xp c ng 1 m p 1 s c gi tr bng 0. Nh vy ta c p1+m m xp p1+m x1 x2 trong cch chn cc s aj ng vi cc gi tr bng 0 ny. Tc l gi tr ai1 ai2 . . . aip lp m tng. M p1+m chia ht cho p, suy ra tng chia ht cho p. m Ta c mu thun vi gi thit phn chng, v bi ton c chng minh. Bi tp 19. Mt dy s thc a1 , a2 , . . . , an gi l p - cn bng nu ta c tng ak + ak+p + ak+2p + bng nhau vi mi 1 k p. Chng minh rng nu mt dy c 50 phn t l p cn bng vi p bng 3, 5, 7, 11, 13, 17 th dy gm ton cc s 0. Li gii. Xt a thc: F (x) =
i=1 50

ai xi1

Theo gi thit ta c, tt c cc nghim ca cc phng trnh xp1 + xp2 + + 1 = 0 vi p {3, 5, 7, 11, 13, 17} u l nghim ca F . Tng s nghim ny l 2+4+6+10+12+16 = 50, nh vy F l mt a thc bc 49 nhng c n 50 nghim, suy ra tt c cc h s ca F u bng 0.

98 Bi tp 20. Cho hai h tp hp (An )n 1 , (Bn )n i) A1 = , B1 = {0}; Tm tt c s nguyn dng n sao cho Bn = {0}. Li gii. Ta xt hai dy a thc sau: an (x) =
aAn 1

tha mn:

ii) An = {x + 1|x Bn1 }, Bn = An1 Bn1 An1 Bn1 .

xa xb
bBn

bn (x) =

Ta thy rng nu xt h s ca hai dy a thc ny theo modulo 2 ta s c: bn (x) = an1 (x) + bn1 (x) v an (x) = xbn1 (x) suy ra: bn (x) = bn1 (x) + xbn2 (x) By gi ta s xt dy a thc Kn (x) tha mn: K1 (x) = K2 (x) = 1, Kn (x) = Kn1 (x) + xKn2 (x) Nu ly cc h s ca Kn (x) theo modulo 2 ta s c bn (x). Sau y ta i tm cng thc cho Kn (x), xt hm sinh: f (y) = Bin i t cng thc truy hi ta c: (1 y xy 2 )f (y) = 1 hay f (y) = T ta tnh c
n1 i=0

Ki+1 (x)y i

i=0

(y + xy 2 )i

Kn (x) =
k=0

xk

n1k k

Bng cch tnh th vi gi tr u ca bn (x) ta s c d on bn (x) = 1 khi n = 2m . Vy ta s chng minh iu ny. Xt n = 2m , ta chng minh: n1k chia ht cho 2 vi mi 1 k n 1. k v2 n1k k = =
t=1 m t=1 m

k 2m 1 2k 2m 1 k t 2t 2 2t k 2t k 2t + 1 2k 2t 1 + 2k 2t

=
t=1

+1

99 Ta chn t sao cho 2t1 k < 2t (do k 1 nn ta chn c nh vy), khi : = k 2t 1 + 2k = 1 2t 2t 1 k +1=1 2t

k 2t 1 + 2k 2t Suy ra: k 2t +1 1 + 2k 2t

Ta chng minh c nu n = 2m th nk1 chia ht cho 2 vi mi 1 k n 1. k By gi t n = 2m r, r l, r > 1. Ta s cmr tn ti n 1 k 1 sao cho n1k l s l: Ta k m1 chn k = 2 (r 1), khi : v2 n1k k = = = Nu t m th
t=1 t=1 t=1

2m1 (r 1) 2m r 1 2m (r 1) 2m r 1 2m1 (r 1) 2t 2t 2t 2m1 (r 1) 2t 2m1 (r 1) 2t 2m1 (r 1) 2t < 2t 2m1 (r 1) (mod 2t ) + + 2m r 1 2m (r 1) 2t 2m 1 2t 2m 1 2t

=0

Nu t

m + 1 th t 0

suy ra: = 2m s

2m (2tm 1). Khi : 2m1 (r 1) 2t + 2m 1 2t = =0 + 2m 1 2t

Vy nu n = 2m th s tn ti 1 k n 1 n1k l s l. k T ta c tt c gi tr cn tm ca n l cc phn t ca tp {2m |m N}.

Ti liu tham kho


[1] Titu Andreescu, Zuming Feng, A Path to Combinatorics for Undergraduates: Counting Strategies, Birkhuser, 2003. a [2] Milan Novakovic, Generating Functions, Olympiad Training Materials, The IMO Compendium Group, 2007.

100

GI I TON T H P B NG I L NG B T BI N
Trn Gia Huy1

1. Thut ton
nh ngha 1. Cho tp A = v ta gi l khng gian cc trng thi, mi phn t ca A gi l mt trng thi. Khi , mi nh x T : A A gi l mt thun ton (tmat).

2. Cc bi ton v thut ton


Bi ton 1 (Bi ton tm kim thut ton). Cho trng thi ban u 0 v trng thi kt thc n . Hi c hay khng thut ton T trn A sao cho khi thc hin T hu hn ln ta thu c n ? T T T T 0 1 2 n . Bi ton 2. Cho thut ton T trn A v trng thi ban u A. a) Gi s A. Hi c th nhn c t sau mt s hu hn bc thc hin T hay khng? b) Tm tp tt c cc trng thi c th nhn c t sau mt s hu hn bc thc hin thut ton T : = { A : = T n ()}.

3. Hm bt bin
Cho T l mt thut ton trn A, I l mt tp hp khc rng m ta gi l khng gian cc mu bt bin. Khi nh x H : A I gi l hm bt bin trn A nu: a, b A : b a H(a) = H(b). gii cc bi ton bng nguyn l bt bin th vic quan trng nht chnh l pht hin ra cc yu t bt bin, sau l vic s dng cc yu t vo trong bi ton mt cch thch hp. Trong mi bi ton sau khi gii xong chng ti u phn tch nhng bt bin nm trong bi ton. chnh l cha kha tm li gii cho bi ton.
1

Gio vin trng THPT chuyn Quang Trung, Bnh Phc.

101

102

4. Cc v d
V d 1. Trn bng, ngi ta vit cc s t nhin lin tip t 1 n 99 sau thc hin tr chi nh sau: mi ln xa hai s bt k v vit mt s mi bng tng hai s xa. Vic lm ny thc hin lin tc cho n khi cn mt s trn bng. Hi s cui cng cn li trn bng l bao nhiu? Ti sao? Li gii. V mi ln thc hin tr chi th thay hai s bng tng ca chng nn tng cc s trn bng khng thay i trong mi thi im. Tng cc s lc u l 1 + 2 + 3 + + 99 = Suy ra s cui cng l 4950. (1 + 99) 99 = 4950 2

Nhn xt. Bt bin trong bi ton trn l tng ca cc s trn bng khng thay i trong mi thi im. Vi mi cch thc hin tr chi th s cui cng cn li trn bng l 4950. V d 2. Trn bng, ngi ta vit cc s t nhin lin tip t 1 n 999 sau thc hin tr chi nh sau: mi ln xa hai s bt k v vit mt s mi bng hiu hai s xa(ly s ln tr s nh). Vic lm ny thc hin lin tip cho n khi cn mt s trn bng. Hi s cui cng cn li trn bng c th l 1 khng? Ti sao? Li gii. Ta thy rng nu xa i hai s a, b (a > b) v thay bng hiu a b th tng cc s trn bng gim i mt i lng l a + b (a b) = 2b l s chn. Nh vy tng cc s trn bng khng thay i tnh chn l ti mi thi im thc hin tr tri. Tng cc s lc u l: 1 + 2 + 3 + + 999 = (1 + 999) 999 = 499500 2

l s chn. Suy ra s cn li cui cng l s chn v do khng th l s 1.

Nhn xt : Bt bin ca bi ton l tng ca cc s trn bng khng thay i tnh chn l ti mi thi im da vo c im tng cc s gim i mt lng chn. V d 3. Trn bng, ngi ta vit 100 ch s 1 v 10 ch s 2 sau thc hin tr chi nh sau: mi ln xa hai s bt k v vit mt s mi bng tch hai s xa. Vic lm ny thc hin lin tc cho n khi cn mt s trn bng. Hi s cui cng trn bng cn li l bao nhiu? Ti sao? Li gii. V mi ln thc hin tr chi thay hai s bng tch ca chng nn tch cc s trn bng khng thay i trong mi thi im. Tch cc s lc u l 1100 210 = 1024 nn s cn li cui cng l 1024. Nhn xt. Bt bin ca bi ton trn l tch cc s trn bng khng thay i trong mi thi im. V d 4. Trn bng, ngi ta vit cc s t nhin lin tip t 1 n 100 sau thc hin tr chi nh sau: mi ln xa hai s bt k v vit mt s mi bng tng lp phng ca hai s cho. Vic lm ny thc hin lin tc cho n khi cn mt s trn bng. Hi s cui cng cn li trn bng c th l 9876543212008 hay khng? Ti sao?

103 Li gii. Ta thy rng nu xa hai s a, b (a > b) v thay bng tng lp phng a3 + b3 th tng cc s trn bng tng mt i lng l: a3 + b3 (a + b) = (a3 a) + (b3 b) l s chia ht cho 3 Tng cc s trn bng lc u v tng cc s trn bng ti mi thi im km nhau mt bi s ca 3. Tng cc s lc u l 1 + 2 + 3 + + 100 = (1 + 100) 100 = 5050 2

l s chia cho 3 d 1(v tng cc ch s ca 5050 bng 10 chia 3 d 1). Suy ra s cn li cui cng phi l s chia 3 d 1. S 987654321 chia ht cho 3 v tng cc ch s ca s ny l 45 chia ht cho 3. Suy ra 9876543212008 chia ht cho 3. Vy s cn li cui cng khng th l 9876543212008 . Nhn xt. Bt bin ca bi ton trn l tng cc s trn bng ti mi thi im hn km nhau mt bi s ca 3. V d 5. Cho s t nhin c 8 ch s l 12456789. T s ny ngi ta i v tr cc ch s ca n, hi c th to c s chnh phng hay khng? Li gii. Ti mi thi im thay i v tr cc s hng th s c to thnh c tng cc ch s l: 1 + 2 + 4 + 5 + 6 + 7 + 8 + 9 = 42 chia ht cho 3 nhng khng chia ht cho 9. Suy ra s c to thnh chia ht cho 3 nhng khng chia ht cho 9 nn khng th l s chnh phng. Nhn xt. Bt bin ca bi ton chnh l tng ca cc s c to thnh lun khng i bng 42. V d 6. Xt mt bng vung 4 4 . Ti mi ca bng vung c cha du + hoc du . Mi mt ln thc hin, cho php i du ca tt c cc trn cng mt hng hoc cng mt ct. Gi s bng vung ban u c mt du + v 15 du . Hi c th a bng ban u v bng c ton du cng c khng? Li gii. Cu tr li l khng. V li gii kh n gin. Thay du cng bng s 1 v du tr bng 1. Xt tch tt c cc s trn bng vung. Khi , qua mi php bin i, tch ny khng thay i (v s i du 4 s). V vy, cho d ta thc hin bao nhiu ln, t bng vung (1, 15) s ch a v cc bng vung c s l du , c ngha l khng th a v bng c ton du cng. Nhn xt. Vic thay du cng bng s 1 v du tr bng s 1 tm ra bt bin ca bi ton trn. Bt bin ca n l tch ca cc s trn bng khng thay i qua mi ln thc hin. V d 7. Trn mt ci bng, ngi ta vit 2008 du (+) v 2009 du (). Gi s mi ln, hai du bt k b xa i v vit thay bi mt du (+) nu chng ging nhau v thay bng mt du () nu chng khc nhau. Sau khi thc hin 4016 ln nh vy, du no s cn li trn bng?

104 Li gii. (i) Cch 1. Sau mi ln xa, s cc du () c gi nguyn hoc gim i 2. V th, tnh chn l ca s du () trn bng khng thay i. Ban u, s du tr l s l nn cui cng du cn li trn bng l du (). (ii) Cch 2. Thay mi du (+) bi s 1, thay mi du () bi s 1. Khi mi ln thc hin cch lm theo bi c th m t di dng nh sau: hai s bt k c xa i v thay bng tch ca chng. Nhu vy ti mi thi im thc hin th tch ca cc s trn bng khng thay i. Ban u tch cc s trn bng l 1 nn cui cng tch cc s trn bng cng l 1. Vy du cn li trn bng l du (). (iii) Cch 3. Thay mi du (+) bi s 0, thay mi du () bi s 1. Khi , tng hai s b xa i cng tnh chn l vi s c vit thay chng, nh vy tng cc s trn bng khng thay i tnh chn l. V tng cc s lc u bng 2009, l s l nn s cn li cui cng l s l. Do du cn li trn bng l du (). Nhn xt. Trong cch 1, i lng bt bin l tnh chn l ca s du () trn bng. Trong cch 2, i lng bt bin l tch tt c cc s trn bng. Trong cch 3, i lng bt bin l tnh chn l ca tng cc s trn bng. Ta c th lp mt tnh hung mi tng ng vi tnh hung c xc nh iu kin v yu cu trong bi ton d tm i lng bt bin. V d 8. Ngoi bin ng, trn mt hn o sinh sng ba ging thn ln c ba loi mu: mu xm c 133 con, mu nu c 155 con v mu c 177 con. Nu hai con thn ln khc mu gp nhau th chng ng thi i sang mu th ba (v d nu thn ln mu xm gp thn ln mu nu th c hai con u i sang mu ). Trong nhng trng hp hai con thn ln cng mu gp nhau th chng gi nguyn khng i mu. C xy ra tnh trng l trn o tt c thn ln cng mt mu c khng? Li gii. c im ca bi ton ny nm b s 133, 155, 177 v khi chia cho 3 ta c b s d 1, 2 v 0. Ta th xt nu mt con thn ln xm gp mt con thn ln nu, th chng ng thi i mu . Khi ta c 132 con xm, 154 con nu v 179 con . Nhng s d ca 132, 154 v 179 cho 3 tng ng l 0, 1 v 2, ngha l gp y cc s d c. Nu mt con thn ln xm gp con thn ln mu , th chng ng thi i mu nu. Khi ta c 132 thn ln xm, 157 thn ln nu v 176 thn ln . Ly nhng s trn chia cho 3 cho s d tng ng l 0, 1, 2 ngha l gp li c ba kh nng ca s d. Nu con thn ln nu v thn ln gp nhau, th chng cng i mu xm. Khi c 135 thn ln xm, 154 thng ln nu v 176 thn ln . S d ca nhng s thn ln trn chia cho 3 tng ng l 0, 1 v 2, vn c y cc s d khi chia cho 3. D thay i mu th no th s d ca cc s lng thn ln chia cho 3 u c y ba s l 0, 1, 2. S lng tt c thn ln trn o l 133 + 155 + 177 = 465 l mt s chia ht cho 3. Nu tt c thn ln cng mt mu th s d lng thn ln mu xm, mu nu, v chia cho 3 c d tng ng l 0, 0, 0. iu ny v l v cc s d phi c y cc s d khi chia cho 3. Nh vy cu tr li l khng c.

105 Nhn xt. Bt bin y l d thay i mu th no th s d ca cc s lng thn ln chia cho 3 u c y ba s l 0, 1, 2. V d 9. Nhng s 1, 2, 3, . . . , 1974 c vit trn mt bng. Ngi ta thay hai s bt k bng mt s hoc l tng hoc l hiu ca hai s . Chng minh rng sau 1973 ln thc hin thao tc trn, ch cn mt s cn li trn bng khng th l s 0. Li gii. Ta quan tm n tnh chn l ca cc s cho v sau mi ln thao tc c s chn l nh th no. Khi bt u trn bng c 987 s l. Mi ln ta thc hin thay i, s ca nhng s l hoc l cn nguyn (khi ta ly hai s c tnh chn l khc nhau hoc hai s cng tnh chn) hoc l gim i hai s (khi ta ly hai s cng tnh l). Nh vy s cc s l cn li sau mi ln thc hin thay i lun l mt s l. Vy khi cn li mt s cui cng trn bng th n phi l s l, do n khng th l s 0. Nhn xt. Bt bin ca bi ton l s cc s l sau mi php bin i cn li l s l.
1 2 V d 10. Trn bng c cc s 96 , 96 , . . . , 96 . Mi mt ln thc hin, cho php xo i hai s 96 a, b bt k trn bng v thay bng a + b 2ab. Hi sau 95 ln thc hin php xo, s cn li trn bng l s no?

Li gii. Gi s cc s trn ang l a1 , a2 , . . . , ak . Ta cho tng ng bng ny vi tch s (2a1 1)(2a2 1) (2ak 1) Khi , sau mi ln bin i, tch trn b mt i hai tha s (2a 1), (2b 1) v c thm vo tha s 2(a + b 2ab) 1 = (2a 1)(2b 1) Do tch trn vn khng i (ch i du). V tch ban u bng 0 (do bng ban u c cha s 48 = 1 !) nn s cui cng s cng phi cho tch s bng 0, tc l 96 2 2s 1 = 0 s = 1 2 Nhn xt. Bt bin ca bi ton l tch s ca bng khng thay i. D nhin trong bi ny ta phi thit lp c bt bin ca cc s trn bng l cho tng ng vi tch s trn. Bi ton trn c to ra t ng thc 2(a + b 2ab) + 1 = (2a 1)(1 2b), nu ta c nhiu ng thc dng ny th c th ch to ra nhiu bi ton t hp dng ny. V d 11. T mt im S(a, b) trn mt phng vi 0 < a < b, Ta xy dng cc im c xc nh bi : 2xn yn xn + yn ; yn+1 = . x1 = a; y1 = b; xn+1 = 2 xn + yn Tm gii hn ca dy (xn ), (yn ).

106 Li gii. Ta d dng thy c tnh bt bin ca dy s ny l xn yn = a b, n Z+ (1)

Dng bt ng thc Cauchy ta suy ra rng xn > yn , n Z+ . Du bng khng th xy ra v xn lun khc yn (iu ny dng quy np c th d dng chng minh c) (2) Nhng ta li c x1 > x2 > > xn (iu ny cng c th d dng c c bng quy np) (3). T cc kt lun (1) v (3), ta suy ra (xn ) l mt dy gim cn (yn ) l mt dy tng (4). T (1), (2) v (4) ta suy ra lim xn = lim yn = ab. Nhn xt. Bt bin ca bi ton trn l tch ca hai dy lun l mt hng s. V d 12. Mi s trong cc s a1 , a2 , a3 , . . . , an nhn mt trong hai gi tr l 1 hoc 1. Bit rng: S = a 1 a2 a3 a 4 + a 2 a3 a 4 a5 + + an a1 a2 a 3 = 0 Hy chng minh rng n chia ht cho 4. Li gii. y l mt bi ton l thuyt s, nhng ta vn s dng bt bin gii n. Tnh bt bin ny nh sau: S s khng thay i s d khi chia cho 4 nu nh ta i du ca 4 s hng lin tip. Tht vy, nu c 2 s dng v 2 s m th s khng c chuyn g thay i, nu c 1 s khc du 3 s cn li th khi i du gi tr ca S s thay i 4 hoc 4 v iu ny khng nh hng g ti s d ca S khi chia cho 4 c, cui cng nu 4 s cng du th khi i du S s thay i mt i lng l 8 hay 8 iu ny d nhin cng khng nh hng g ti s d ca S khi chia cho 4. By gi quay li bi ton ca chng ta. Ta thc hin thut ton i du ca 4 s hng lin tip sao cho cui cng a tt c n s thnh s dng. Khi S = n v theo tnh bt bin th S chia ht cho 4 (v ban u S = 0 chia ht cho 4 ) Vy n chia ht cho 4. Ta c kt lun cho bi ton. Nhn xt. S s khng thay i s d khi chia cho 4 nu nh ta i du ca 4 s hng lin tip. V d 13. Hai ngi chi c, mi vn ngi thng c 2 im, ngi thua c 0 im, nu trn ha th mi ngi c 1 im. Hi sau mt s vn c c th xy ra trng hp mt ngi c 9 im v ngi kia c 10 im hay khng? Li gii. Tng s im ca hai ngi trong mi vn lun bng 2. Suy ra ti mi thi im th tng im ca hai ngi lun l s chn. M 9 + 10 = 19 l s l nn khng th xy ra trng hp mt ngi c 9 im v ngi kia c 10 im. Nhn xt. Bt bin ca bi ton l tng im ca hai ngi trong mi vn lun bng 2. V d 14. C 1981 tch ung tr t trn mt bn. Lc u tt c cc tch u c t nga ln. Gi s mi ln ngi ta lm cho 200 tch trong chng lt ngc li. Hi sau mt s ln nh vy, c th lm cho tt c cc tch u p xung c khng?

107 Li gii. Nu c 1981 tch, ta khng th quay p xung tt c c. ti mi thi im c x tch t nga c lm p xung v c 200 x tch p xung c lt nga ln. Do s cc tch ang p thay i i mt s l |200 2x|, v y l mt s chn. iu ny c ngha l s cc tch t p xung khng b thay i v tnh chn l. Ban u s ny bng 0, l s chn. Do khng th thay i s ny thnh 1981, l s l. Nhn xt. Bt bin ca bi ton l cc tch ang p lun thay i mt lng chn. Trong ton ta c th thay i 1981 bng mt s l bt k v 200 bng mt s chn bt k nh hn s ban u. V d 15. C 2005 ng xu, mi ng xu c hai mt, mt mt mu xanh v mt mt mu . Xp cc ng xu trn bng sao cho cc mt mu xanh nga ln. Thc hin tr chi nh sau: mi ln thc hin cho php i bn mt ca bn ng xu ty . Hi c th nhn c kt qu m tt c cc ng xu u c mt nga ln trn c khng? Li gii. Thay mi ng xu mt mu xanh nga ln trn bi s 1. Thay mi ng xu mu nga ln trn bi s (+1). Khi mi cch thc hin theo bi c th m t di dng khc nh sau: bn s bt k c xa i v thay mi s bng s i ca n. Ban u c 2005 s (1) tng ng vi 2005 ng xu c mt mu xanh nga ln trn. Ta thy rng nu thay bn s a, b, c, d bi bn s a, b, c, d th tch ca cc s mi thay vo l (a)(b)(c)(d) = abcd bng tch ca cc s ban u. Nh vy ti mi thi im th tch ca cc s khng i. Ban u c 2005 s 1 nn tch ca chng l (1), suy ra ti mi thi im tch cc s lun l (1). Vy khng th nhn c kt qu m tt c cc ng xu u c mt nga ln trn. Nhn xt. Bi ton trn c thay i d tm bt bin hn, bt bin l trong mi thao tc th tch cc ch s l khng i. V d 16. Trong dy 1, 9, 9, 9, 8, . . . , mi ch s bt u t ch s th nm bng ch s hng n v ca tng bn ch s lin trc n. Hi trong dy ny c gp cc b 1234 v 5678 khng? Li gii. Ta thay mi ch s ca dy cho bng s 0 nu n l s chn v bng s 1 nu n l s l. Ta nhn c dy s 111101111011110..., trong c sau bn ch s 1 c mt ch s 0 v c sau mi s 0 c bn ch s 1. Cc b s 1234 v 5678 ng vi cc b 4 ch s 1010 v 1001 nn khng th c mt trong dy s cho. Nhn xt. y l bi ton ti rt thch, n th hin c t duy chuyn bi ton kh thnh bi ton d c th pht hin c bt bin. R rng nguyn rt kh tng minh c li gii, tuy nhin khi chuyn ha th bt bin chnh l c sau bn ch s 1 c mt ch s 0 v c sau mi s 0 c bn ch s 1. V d 17. Mt hnh trn c chia thnh 10 hnh qut, trn mi ngi ta t 1 vin bi. Nu ta c di chuyn cc vin bi theo quy lut: mi ln ly 2 bt k mi 1 vin bi, chuyn sang lin k theo chiu ngc nhau th c th chuyn tt c cc vin bi v cng 1 hay khng?

108 Li gii.

Trc tin, ta t mu xen k cc hnh qut, nh vy s c 5 c t mu ( mu) v 5 khng c t mu ( trng). Ta c nhn xt : Nu di chuyn 1 bi mu v 1 bi trng th tng s bi 5 mu khng i. Nu di chuyn 2 mu, mi 1 bi th tng s bi 5 mu gim i 2. Nu di chuyn 2 trng, mi 1 bi th tng s bi 5 mu tng ln 2. Vy tng s bi 5 mu hoc khng i, hoc gim i 2 hoc tng ln 2. Ni cch khc, tng s bi 5 mu s khng thay i tnh chn l so vi ban u. Ban u tng s bi 5 mu l 5 vin (l s l) nn sau hu hn ln di chuyn bi theo quy lut trn th tng s bi 5 mu lun khc 0 v khc 10, do khng th chuyn tt c cc vin bi v cng 1 . Nhn xt. Bi ton c gii d dng bng cch t mu xen k cc hnh qut. Trong rt nhiu bi ton th vic t mu chnh l tng chnh gii bi ton. V bt bin trong bi ton ny l tng s bi 5 mu s khng thay i tnh chn l. V d 18. Mi s trong dy 21 , 22 , 23 , . . . , 22005 u c thay th bi tng cc ch s ca n. Tip tc lm nh vy vi cc s nhn c cho ti khi tt c cc s trong dy u c 1 ch s. Chng minh trong dy ny: s cc s 2 nhiu hn s cc s 1. Li gii. Ta thy : S t nhin A v tng cc ch s ca A lun cng s d trong php chia cho 9. Mt khc ta c : 21 chia cho 9 d 2; 22 chia cho 9 d 4; 23 chia cho 9 d 8; 24 chia cho 9 d 7; 25 chia cho 9 d 5; 26 chia cho 9 d 1; 27 chia cho 9 d 2;. . . Do 26k + r ln lt nhn cc s d trong php chia cho 9 l 2, 4, 8, 7, 5, 1 tng ng vi cc gi tr ca r l 1, 2, 3, 4, 5, 0. Dy cui cng nhn c gm 2005 s thuc tp hp {2; 4; 8; 7; 5; 1}. Ta c 2005 = 334 6 + 1 nn dy cui cng c 335 s 2 (nhiu hn s cc s khc 1 s). Vy s cc s 2 nhiu hn s cc s 1 ng 1 s. Nhn xt. Bi ton trn xut pht t tnh cht trong s hc. S t nhin A v tng cc ch s ca A lun cng s d trong php chia cho 9 v dy cc s d khi chia an cho m lun tun hon (c th khng xut pht t ch s u tin). V bt bin ca bi ton l dy s trn c chuyn thnh dy cc s d ca n 248751248751 . . . V d 19. Mt t giy b ct nh thnh 6 mnh hoc 11 mnh. Cc mnh nhn c li c th chn ct (thnh 6 mnh hoc 11 mnh nh hn),. . . C nh vy ta c th nhn c 2005 mnh ct khng ?

109 Li gii. Sau mi ln ct mt mnh giy thnh 6 mnh hoc 11 mnh th s mnh giy tng ln l 5 hoc 10. Nh vy tnh bt bin ca bi ton l s mnh giy lun tng ln mt bi s ca 5. Vy s mnh giy sau cc ln ct c dng 1 + 5k, mt khc 2005 c dng 5k nn vi cch ct nh trn, t mt t giy ban u, ta khng th ct c thnh 2005 mnh. Nhn xt. Bt bin ca bi ton l s mnh giy tng ln lun l mt bi s ca 5. V d 20. Ta xt bng 4 hng 4 ct bao gm nhng du + v du nh sau: + + + + + + + + + + + + + + +

Ngi ta c th ng thi thay i du ca tt c cc du trong mt hng bt k, mt ct bt k, hoc trong mt ng bt k m n song song vi mt trong nhng ng cho(thc t ngi ta c th thay i du 4 gc). C kh nng hay khng nhn c mt bng khng cha du no? Li gii. Ta xt bng th hai nh sau: 0 1 1 0 1 0 0 1 1 0 0 1 0 1 1 0

Bng ny c tnh cht: trong mi hng, mi ct hoc nhng ng thng song song vi ng cho c ng hai s 1 hoc khng c s 1 no. By gi ta gi nhng phn t trong bng th nht l phn t tt nu n c t vo ng v tr c s 1 nh bng th hai. Sau khi ta thay i du th ch c th xy ra mt trong ba trng hp sau: Trng hp 1. Hai du + phn t tt c chuyn thnh hai du phn t tt. Trng hp 2. Hai du phn t tt c chuyn thnh hai du phn t tt. Trng hp 3. Mt cp phn t tt gm mt du + v mt du chuyn i cho nhau. Nh vy s lng du phn t tt lun l s l. Do lun tn ti t nht mt du trn bng. V d 21. C hai ng , mt ng c n hn v ng kia c k hn. C mi pht mt my t ng li chn mt ng c s hn l chn v chuyn mt na s hn ca ng c chn sang ng kia (nu c hai ng u c s hn l chn th my s chn ngu nhin mt ng). Nu trong hai ng s hn u l l th my s ngng lm vic. Hi tn ti bao nhiu cp th t (n, k), vi n v k l cc s t nhin khng vt qu 1000, my t ng sau khong thi gian hu hn s dng.

110 Li gii. Gi s n = 2a u v k = 2b v, vi u v v l cc s l. Chng ta s chng minh rng my t ng nht thit s dng i vi cc cp v ch cc cp s (n, k) vi a = b. Nu a = b th t cp (n, k) my t ng c th nhn c cp (2a1 u, 2a1 (u + v)) hoc (2a1 (2u + v), 2a1 v). V cc s (2u + v) v (2v + u) li l l, nn my t ng lm gim s m ca 2 xung 1 n v. Qua a bc th s m ny tr nn bng 0, v my t ng s dng li. By gi gi s a < b(trng hp a > b xt tng t). Nu a b 2, th t cp (n, k) my t ng c th nhn c cp (2a (u + 2b1a v), 2b1 v) vi cc s m trong ly th ca 2 khc nhau. Nu a = b 1, th t cp (n, k) my t ng c th nhn c cp (2a(u + v), 2au) = 2a + 1 u+v , 2a u li vi 2 cc s m trong ly tha ca 2 khc nhau. D dng thy rng trong trng hp ny my t ng s lm vic mi mi khng dng. Ch cn vic m cc cp s kh d. C 500 s l khng vt qu 1000, bi vy s cp (n, k) = (2a u, 2b v) vi a = b = 0 bng 5002 ; c 250 s khng vt qu 1000 chia ht cho 2 v khng chia ht cho 4, bi vy s lng cp vi a = b = 1 bng 2502 . C tip tc nh vy, ta nhn c p s ca bi ton: 5002 + 2502 + 1252 + 632 + 312 + 162 + 82 + 42 + 22 + 12 = 333396 V d 22. Trn bng vit 10 du cng v 15 du tr. Vi 24 ln thc hin, mi ln xa i 2 du bt k ri li thm vo 1 du (cng hoc tr) cui cng trn bng ch cn li 1 du duy nht. Bit rng du c thm vo s l du tr nu trc xa i 2 du khc nhau, ngc li du c thm vo s l du cng. Hi du cn li trn bng l du g ? Li gii. Ta thy, nu xa i 2 du cng th phi thm vo 1 du cng, v vy s du tr trn bng khng thay i. Nu xa i 2 du tr th phi thm vo 1 du cng, v vy s du tr gim i 2. Nu xa i 1 du cng v 1 du tr th phi thm vo 1 du tr, v vy s du tr trn bng khng thay i. Nh vy, tnh bt bin l : sau mi ln thc hin vic xa v thm du, s du tr trn bng hoc khng thay i hoc gim i 2. Mt khc, s du tr ban u l s l nn sau mi ln thc hin th s du tr cn li trn bng bao gi cng l s l. Sau 24 ln thc hin, trn bng ch cn li 1 du duy nht m du tr khng th mt ht nn du cn li trn bng phi l du tr. V d 23. Cho mt bn c quc t 8 8. Hi rng qun m c th i nc u tin t di cng bn tri v kt thc trn cng bn phi hay khng? Vi iu kin n phi i qua tt c cc trn bn c v mi ch i qua ng mt ln Li gii. Ta t cc trn bn c xen k cc mu en trng nh bn c vua(hnh di)

111 Do s bnh ng mu nn khng mt tnh tng qut ta c th gi s rng di cng bn tri c mu trng. T cch i ca con m ta nhn thy rng sau mi nc i con m s sang mt khc mu vi m n ang ng. V th sau mt s l nc i con m s mu en, sau mt s chn nc i con m s mu trng. y l tnh bt bin ca chng ta. Tr li bi ton ta thy rng i t di cng bn tri ln trn cng bn phi cn i 63 nc i. V th trn cng bn phi s cn mang mu en (theo nh tnh bt bin). iu ny l v l. Vy qun m khng th i t di cng bn tri nn trn cng bn phi nh yu cu ca u bi c. Nhn xt. Bi ton c gii quyt nhng xung quanh bi ton ny vn cn rt nhiu iu cn phi suy ngh. Chng hn nh khi xt bn c X X vi X l mt s l th liu c mt cch i t di cng bn tri ln trn cng bn phi v tho mn cc yu cu ca bi ton hay khng? Nu c th hy ch ra mt cch i nh th? Cu hi ny kh kh. V d 24. Cho mt bng vung cha s nh hnh 4a . Ta thc hin mt thut ton T nh sau: Chn ra 2 s bt k nm hai vung cnh nhau v cng 2 s vi mt s nguyn no . Hi rng sau mt s ln thc hin thut ton T th bng hnh vung cha cc s nh hnh 4a c th thnh bng hnh vung nh hnh 4b hay khng? 1 2 3 4 5 6 7 8 9 hnh 4a 7 8 9 6 2 4 3 5 1 hnh 4b

Li gii. T mu cc ca hnh vung nh hnh di y.

t B l tng cc s cc mu en v W l tng cc s cc mu trng. Ta thy v mi ln thc hin thut ton T ta cng thm 2 s 2 cnh nhau vi mt s nguyn nn d thy rng hiu B W l khng i. Nhng vi gi thuyt ca bi ton th hnh 4a th B W = 5, cn hnh 4b th B W = 1. iu ny tri vi quy tc bt bin trn. Vy sau nhng ln thc hin thut ton T th t hnh 4a ta khng th nhn c hnh 4b. V d 25 (VMO 2006). Xt bng vung m n (m, n l cc s nguyn dng ln hn 3). Thc hin tr chi sau: mi ln t 4 vin bi vo 4 ca bng (mi 1 vin) m 4 to thnh mt trong cc hnh di y:

112 Hi sau mt s ln ta c th nhn c bng m s bi trong cc bng nhau c khng nu: a) m = 2004 v n = 2006? b) m = 2005 v n = 2006? Li gii. a) Bng cho c th chia thnh cc hnh ch nht 4 2 nn c th nhn c trng thi m s bi trong cc bng nhau. b) T mu cc ca bng nh hnh v (t cc hng 1, 3, . . . , 2005).

D thy, mi ln t bi c 2 vin c t vo cc c t mu v 2 vin c t vo cc khng t mu. Do , nu gi S(n) l s bi trong cc c t mu v T (n) l s bi trong cc khng c t mu sau ln t bi th n th S(n) T (n) l bt bin. Ta c S(n) T (n) = S(0) T (0) = 0, n 0.

Do , nu nhn c bng m s bi trong cc bng nhau th s c t mu v s khng c t mu bng nhau. iu ny khng th xy ra v m l s l. V d 26. C 2n ngi i s c mi n mt ba tic. Mi ngi i s c nhiu nht n 1 k th. Chng minh rng c th xp cc ngi i s ngi quanh mt bn trn khng ai phi ngi cnh k th ca mnh. Li gii. u tin ta xp ch ngi cho cc ngi i s theo mi cch c th. Gi H l s cp k th ngi cnh nhau. Gi (A, B) l mt cp k th, vi B ngi bn phi ca A ( hnh a). Chng ta s phi tch chng ra. Xt mt cp (A , B ) vi A l mt ngi bn ca A. Ta s t c mc ch bng cch o 2 u mt ca cung BA (hnh b). H s b gim nu (A, A ) v (B, B ) trong hnh b l 1 cp thn thin. c B B A Hnh a Hnh b A

A B

B A

113 Ta cn ch ra rng lun tn ti B nm bn phi ca A v B l bn ca B . Ta bt u t A v i khp xung quanh bn theo chiu kim ng h. Ta c t nht n ngi bn ca A ,v bn phi h s c n ch ngi. Chng khng th b chim gi bi tt c k th ca B v B ch c n 1 k th. Nh vy lun tn ti mt im A cho bn phi ca n c B , mt ngi bn ca B. Kt hp vi nguyn l bt bin m ta tm ra trn th vic lm gim H l lun thc hin c. Vy sau mt s ln nht nh thc hin thut ton nh vi cp (A, B) th ta s c mt cch xp tho mn yu cu ca bi ton.

V d 27. Cc s t nhin 0, 1, 2, 3, . . . c vit trong cc ca mt bng vung knh thc 2003 2003 theo vng xuy trn c(xoay ngc kim ng h) sao cho s 0 nm trung tm (tm ca bng). Cc dng v ct ca bng c nh s tng dn t di ln trn v t tri sang phi (bt u t s 1).

a) S 2004 nm dng no, ct no? V sao?

b) Thc hin thut ton sau: ln u tin, thay s 0 trung tm bi s 1998; mi ln tip theo, cho php ly ra 12 s trong 12 lin tip trong cng mt hng hoc trong cng mt ct hoc trung cng mt hnh ch nht 3 4 ri tng mi s ln mt n v. Hi sau mt s ln nh vy ta c th lm cho tt c cc s trong bng u l bi ca 2004 hay khng? Ti sao?

20 19 18 17 16 21 6 5 4 15 22 7 0 3 14 23 8 1 2 13 24 9 10 11 12

Li gii. a) Xt hnh vung cnh 2n + 1 c tm l cha s 0; s c vit nh di, bn tri ca hnh vung ny l (2n + 1)2 1 (v d cc s 8, 24, . . .). V (2 22 + 1)2 1 = 2024 v s 0 nm dng s 1002, ct s 1002 nn s 2024 nm hng s 1002 22 = 980 v ct s 1002 22 = 980. Vy s 2004 nm hng s 980 + 20 = 1000 v ct s 980.

a) Ta t mu cc ca bn nh hnh v.

114

Gi S(n) l tng cc s trong cc c t mu bc th n. Do mi ln thc hin thut ton (k t ln th 2) c ng 4 c t mu nn S(n + 1) = S(n) + 4, Do S(n) bt bin theo module 4. Suy ra S(n) S(1) Ta xt s d ca S(0) khi chia cho 4: Xt cc ng cho gm nhng c t mu, cc ng cho gm mt trong ba loi sau y: Loi 1: cha ton cc s chia ht cho 4 (ng cho i qua trung tm). Loi 2: cha ton cc s chia cho 4 d 2. Do tnh cht i xng ca bng nn c chn ng cho loi ny v nh vy tng cc s trn cc ng cho loi ny chia ht cho 4. Loi 3: cha ton cc s l: Trn mi ng cho loi ny c mt na s cha cc s chia cho 4 d 1 v mt na s cha cc s chia cho 4 d 1 v nh vy tng cc s trong cc ng cho loi ny cng chia ht cho 4. T c S(0) 4 (mod 4), suy ra S(1) = S(0) + 1994 2 (mod 4) hay S(n) 2 (mod 4), n 1. Vy khng th c trng thi m tt c cc s trong bng u l bi ca 2004 c. V d 28. Xc nh cc s nguyn dng m, n sao cho bng m n c th lt c bi cc qun hnh ch L di y (mod 4), n 1. n 1.

115

Li gii. Khng mt tnh tng qut ta c th gi s m n. lt c bng th m 2, n 3. Gi s ta c th lt c bng bi a qun hnh ch L, ta c m.n = 4a. V m.n 6 nn a 2. Xt a = 2, ta c bng 2 4. Bng 2 4 c th lt c bi hai qun ch hnh L nh hnh v di y:

Vi a = 3 ta c m n = 12 nn c hai bng tha mn l 2 6 v 3 4. D dng kim tra hai bng ny u khng lt c bi cc qun hnh ch L. iu khin ta d on, lt c bng bi cc qun hnh ch L th a chn. chng minh d on ny ta t mu cc ca bng nh sau: Gi s c m chn. Cc dng c th t l (tnh t trn xung di) c t mu en, c dng c th t chn c t mu trng.

Suy ra x = y v a = 2x. iu c ngha l a chn. By gi ta chng minh nu a chn, tc l m n chia ht cho 8 th c th lt c bng bi cc qun hnh ch L. Tht vy, nu m chia ht cho 2 v n chia ht cho 4 th bng c th thnh c hnh ch nht 2 4 nn lt c. Nu m l v n chia ht cho 8 th do m c th c vit di dng m = 2s + 3 nn c th chia bng cho thnh cc hnh ch nht 2 4 v 3 8. Do , nu hnh ch nht 3 8 lt c th bng cho s lt c. Hnh v di y chng t c th lt c hnh ch nht ny.

Khi , s en v s trng bng nhau v bng 2a. Mi qun hnh ch L lt vo bng chim 3 en v 1 trng hoc chim 3 trng v 1 en. Gi s lt c bi x quan hnh ch L chim 3 en v 1 trng v y qun hnh ch L chim 3 trng v 1 en. Ta c h x + y = a 3x + y = 3y + x = 2a.

116

Vy lt c bng cho bi cc qun hnh ch L th iu kin cn v l m n chia ht cho 8 v m, n 2. V d 29 (IMO 2004). Ta nh ngha vin gch hnh mc cu l hnh gm 6 vung n v nh hnh v di y, hoc hnh nhn c do lt hnh (sang tri, sang phi, ln trn, xung di) hoc hnh nhn c do xoay hnh i mt gc.

Hy xc nh tt c cc hnh ch nht m n, trong m, n l cc s nguyn dng sao cho c th lt hnh ch nht bng cc vin gch hnh mc cu. Li gii. D thy m, n {1, 2, 5}. Chia hnh ch nht m n thnh m n vung v nh s cc hng, cc ct t di ln trn, t tri sang phi. Ta gi (p, q) l nm giao ca hng th p v ct th q. Hai vin gch hnh mc cu ch c th ghp li c mt trong hai hnh di y

(H1 )

(H2 )

Do , lt c hnh ch nht m n th m.n phi chia ht cho 12. Nu t nht mt trong hai s m hoc n chia ht cho 4 th c th lt c hnh ch nht m n. Tht vy, gi s c m chia ht cho 4. Nu n chia ht cho 3 th c th chia hnh ch nht m n thnh cc hnh ch nht 4 3, do c th lt c. Nu n khng chia ht cho 3 th c th vit n di dng n = 3a + 4b vi a, b l cc s nguyn dng, do c th lt c. By gi ta chng minh mt trong hai s m, n chia ht cho 4. Gi s ngc li, khi c m v n chia ht cho 4 nhng khng chia ht cho 4. chng minh iu ny khng xy ra ta to bt bin. to bt bin ta in cc s vo cc ca hnh ch nht theo quy tc sau: Xt (p, q).

117 Nu ch mt trong hai ta p v q chia ht cho 4 th in s 1 vo . Nu ch mt trong hai ta p v q chia ht cho 4 th in s 2 vo . Vi cch in s nh vy ta thu c bt bin l tng cc s trong hnh (H1 ) v tng cc s trong hnh (H2 ) lun l s l. Do m, n chn nn tng cc s trong ton b hnh ch nht m n l mt s chn. Mun lt c hnh ch nht m n th tng s hnh (H1 ) v (H2 ) c s dng phi l s chn. Khi , m.n chia ht cho 24. iu ny khng xy ra v c m, n u khng chia ht cho 4. Kt lun. Qua cc v d trn cc bn c th thy c sc mnh ca vic tm ra cc i lng bt bin trong mi bi ton t hp. Cc i lng bt bin xut hin trong tt c cc dng ca bi ton t hp. Tm ra cng nh pht hin cc i lng bt bin chnh l con ng gii cng nh sng to cc bi ton lin quan n ni dung ny.

5. Cc bi ton luyn tp
Bi tp 1. Trn mt bng gm 4 4 vung c vit cc du cng v du tr. i du ng thi cc nm trn cng mt hng hoc trn cng mt ct hoc trn cc dc theo cc ng thng song song vi mt trong hai ng cho. Bng cch nh vy ta c th nhn c bng cha ton du cng khng ? Bi tp 2. Ti nh A1 ca mt a gic u 12 cnh A1 A2 A3 . . . A12 c vit du tr, cc nh cn li c vit du cng. Chng minh rng: bng cch i du ng thi ti 6 nh lin tip bt k vi s ln ty , ta khng th nhn c a gic m ti nh A2 vit du tr cn cc nh khc vit du cng. Bi tp 3. Cho dy s 1, 2, 3, . . . , 2006. Ta thay i v tr cc s theo nguyn tc: mi ln ly ra 4 s bt k ri t chng vo 4 v tr c nhng theo th t ngc li. Bng cch ny, ta c th sp xp dy s trn v dy s 2006, 2005, . . . , 2, 1 khng? Bi tp 4. Mi ngi sng trn tri t thc hin mt s ci bt tay nht nh vi nhng ngi khc. Chng minh rng s ngi thc hin mt s l ci bt tay l s chn. Bi tp 5. Cho cc s 1, 2, 3, . . . , n sp xp theo mt th t no . Tin hnh tro i v tr ca hai s bt k ng k nhau. Chng minh rng nu thc hin mt s l ln nh vy th khng th nhn c sp xp ban u.

Ti liu tham kho


[1] T Duy Phng, Ton tr chi: Phn loi, cng c v phng php gii, chuyn ca Hi Ton hc H Ni, 2011. [2] Nguyn Vn Mu (ch bin), Chuyn chn lc t hp v ton ri rc, NXB Gio dc, 2008. [3] Nguyn Hu in, Gii ton bng i lng bt bin, NXB Gio dc, 2004.

118 [4] Yao Zhang, Combinatorial Problems in Mathematical Competitions, East China Normal University Press, 2011. [5] Arthur Engel, Problem solving strategies, Springer, 1991.

M TS

BI TON T MU
L Tun Linh1

Khi gp nhng bi ton c bi phc tp, mt cng vic chng ta cn lm l a n tr v vi ngn ng Ton hc quen thuc, gip chng ta d dng t duy v vic trnh by li gii cng tr nn n gin hn. Mt phng php thng hay c s dng lm cng vic ny l T mu. Ta s chia i tng ang xt ra thnh nhiu i tng v t bi nhng mu khc nhau. Khng ch vy, nhiu khi T mu s thc s gip chng ta lm vn bi ra tr nn sng sa hn v i ti li gii nhanh chng, ngn gn v d hiu.

1. Tm tt l thuyt
1.1. Nguyn l Dirichlet
Dng n gin nht ca nguyn l Dirichlet l : Nguyn l 1. Nu nht n + 1 th vo n lng th tn ti mt lng c t nht hai con th. V dng tng qut ca n: Nguyn l 2. Nu nht n th vo m lng m php chia n cho m c thng l k v cn d th tn ti mt lng c t nht k + 1 con th. Chng minh. Gi s lng no cng c khng qu k th th m lng c khng qu mk con th, t hn n th (v l). V vy ta c iu cn chng minh. Nhn xt. u im ca nguyn l Dirichlet l n cho php khng nh c s tn ti ca mt i tng c tnh cht no m khng cn ch ra m hnh c th ca n. V d 1 (Bi ton 6 ngi). Chng minh rng trong 6 ngi bt k lun c th tm ra ba ngi i mt quen nhau hoc i mt khng quen nhau. Li gii. y l mt bi ton rt c in ca Ton hc v l mt phn nh trong lp rt rng cc bi ton dng Turan. gii quyt bi ton ny, ta c th pht biu li bi ton nh sau : Cho 6 im trong hai im no cng c ni vi nhau bng mt on thng v t bi ch mt trong hai mu xanh hoc . Chng minh bao gi cng tm ra c mt tam gic c ba cnh cng mu.
1

Lp 10T, trng THPT chuyn Lam Sn, Thanh Ho.

119

120 B C

Gi 6 im l A, B, C, D, E, F . T im A ta k c 5 on thng l AB, AC, AD, AE, AF . Theo nguyn l Dirichlet, tn ti ba on thng c t bi cng mt mu. Khng mt tnh tng qut gi s l cc on thng AB, AC, AD v cng t mu xanh. Nu tn ti t nht mt on thng trong ba on thng BC, CD, DB c t xanh th bi ton c chng minh (gi s BC c t xanh th tam gic ABC tho mn) Nu c ba on thng BC, CD, DB u c t th tam gic BCD tho mn. Vy bi ton c chng minh. V d 2. Mi im trn mt phng c t bi mt trong hai mu. Chng minh rng tn ti hai im cng mu c khong cch bng 1. Li gii. Chn ba im bt k trong mt phng sao cho chng to thnh mt tam gic u cnh 1. Theo nguyn l Dirichlet, v ba im c t bi hai mu nn tn ti hai im cng mu v chnh l hai im cn tm. V d 3. Mi im trn mt phng c t bi mt trong hai mu. Chng minh tn ti mt hnh ch nht c 4 nh c t bi cng mt mu. Hy pht biu v chng minh mnh tng qut. Li gii. Xt cc giao im ca ba ng thng ngang v 9 ng thng ng nh hnh v. S cch t mu ba giao im trn cng mt ng thng ng l 2 2 2 = 8 cch. Do c 9 ng thng ng nn tn ti hai ng thng c cch t mu ba giao im ging ht nhau. Gi s hai ng thng l hai ng thng cha cc giao im l A1 , A2 , A3 v B1 , B2 , B3 nh hnh v. A1 A2 A3 B1 B2 B3

Ba im A1 , A2 , A3 c t bi hai mu nn tn ti hai im cng mu. Gi s l A1 v A3 . Nh vy hnh ch nht A1 A3 B3 B1 c 4 nh c t bi cng mt mu. Mnh tng qut: Mi im trn mt phng c t bi mt trong n mu (n l s nguyn dng). Chng minh tn ti mt hnh ch nht c 4 nh c t bi cng mt mu. Chng minh mnh tng qut tng t nh vi trng hp n = 2. Ta s xt giao im ca ca ba ng thng ngang v n3 + 1 ng thng ng.

121

1.2. Nguyn l cc hn
Nguyn l 3 (Nguyn l cc hn). Trong mt tp hp hu hn (khc rng) cc s thc, lun tn ti s nh nht v s ln nht. Nh nguyn l cc hn ta c th xt cc phn t m mt i lng no c gi tr nh nht hoc ln nht nh khong cch nh nht (ln nht) trong cc khong cch gia hai im hay gia mt im vi mt ng thng, xt im pha bn tri nht hay bn phi nht trong cc im nm trn mt on thng, im c ni vi nhiu im nht,. . . Ta cng n vi cc v d di y: V d 4. Cho mt s hu hn im, trong c mt s im mu en, cn li l im trng. Bit rng: (i) Mi im en u c ni vi t nht mt im trng. (ii) Khng c im trng no c ni vi tt c cc im en. Chng minh rng tn ti mt nhm 4 im gm 2 im trng, 2 im en m mi im c ni vi ng mt im khc mu ca nhm . Li gii. Gi A l im trng c ni vi nhiu im en nht. Do iu kin (ii) nn tn ti im en khng c ni vi A, gi im l B. Do iu kin (i) m B phi c ni vi 1 im trng khc A, gi im l C. Trong tt c cc im en c ni vi A tn ti mt im khng c ni vi C, gi l D (v nu khng tn ti th im trng C c ni vi nhiu im en hn A, tri vi cch chn im A). Nh vy nhm 4 im A, B, C, D l nhm cn tm. V d 5. Cho tp hp X gm hu hn cc im trn mt phng. Mi cp im bt k c ni vi nhau bi mt on thng v t bng mt trong hai mu xanh hoc . Bit rng nu hai im A v B thuc tp hp X l u mt ca cng mt s on thng xanh th khng tn ti im C thuc tp hp X sao cho AC v BC cng t xanh. Chng minh rng tn ti mt im m n ch l u mt ca ng mt on thng xanh. Li gii. V s im thuc X l hu hn nn c th tm c mt im A l u mt ca nhiu on thng t mu xanh nht. Gi s AB1 , AB2 , . . . , ABk t mu xanh v Bi l u mt ca ni on thng xanh. T iu kin bi ton ra ta suy ra n1 , n2 , . . . , nk i mt khc nhau. Mt khc cc s n1 , n2 , . . . , nk nhn cc gi tr nguyn trong khong t mt n k nn chc chn tn ti ni = 1. Ta c iu cn chng minh.

1.3. Tnh cht bt bin


Trong cc bi ton t hp, ngoi cc bi ton s dng nguyn l Dirichlet v nguyn l cc hn, ta cn gp cc bi ton s dng tnh cht bt bin ca cc i tng khi chng thay i. Mc d cc i tng ny thay i nhng vn c nhng tnh cht khng b thay i trong sut qu

122 trnh bin i v c gi l tnh cht bt bin. S dng tnh cht bt bin ta c th loi b nhng trng hp, nhng kh nng khng th xy ra. Mt bt bin n gin nht v thng gp l tnh chn l ca s, tc l ta s xt s d khi chia cho 2. Ngoi ra ta cng c th xt s d cho cc s bt k khc. thit lp cc bt bin i khi ta cng s dng phng php t mu, tc l chia cc i tng ang xt ra lm nhiu nhm, cc phn t ca mi nhm c t bi cng mt mu. V d 6. Cho 13 on thng thng ng, u trn t , u di t xanh. Mi lt ngi ta i mu hai u ca 4 on thng: thnh xanh, xanh thnh . C cch no sau mt s lt i mu, u trn ca 13 on thng c t xanh hay khng? Li gii. Nhn thy rng sau mi ln i mu th s u trn mu ca 13 on thng lun thay i mt s chn, m ban u s u trn mu l s l (13) nn khng th c kh nng khng c u trn no mu sau mt s ln i, tc l khng xy ra trng hp c 13 u trn ca 13 on thng u c mu xanh. V d 7. Mt hnh trn c chia thnh 6 hnh qut, trong mi hnh qut ta t mt vin bi. Thc hin tr chi nh sau: Mi ln chuyn mt vin bi mt hnh qut no sang hnh qut k vi n. Hi sau 20 ln chuyn bi ta c th nhn c trng thi m c 6 vin bi u cng mt hnh qut hay khng? Li gii. T mu cc hnh qut nh hnh v. Gi Sn l tng s vin bi trong cc hnh qut mu en sau ln chuyn bi th n.

Ta thy Sn thay i sau mi ln chuyn bi, tuy nhin Sn n + 1 (mod 2) vi mi n 0. Suy ra S20 l s l. Nu sau 20 ln chuyn bi m ta c th nhn c kh nng 6 vin bi cng thuc mt hnh qut th Sn = 0 hoc Sn = 6. Vy sau 20 ln chuyn bi ta khng th nhn c kt qu m c 6 vin bi u thuc mt hnh qut. V d 8. Cho mt bn c quc t. c php sn li cc ca mt ng nm ngang hoc ng thng ng no thnh mu khc. Hi bng cch c th nhn c mt bn c ch gm duy nht mt en hay khng ? Li gii. Khi t li mt ng nm ngang hoc mt ng thng ng c k en v 8 k trng ta c 8 k en v k trng. Do s en thay i i (8 k) k = 8 2k, tc l thay i mt s chn . Bi v tnh chn l ca s en khng thay i, m ban u c mt s chn cc en (32 ) nn ta khng th nhn c kh nng bn c ch c duy nht mt en.

123

2. Mt s dng ton thng gp v t mu


2.1. Bi ton lin quan n th
V d 1 trnh by trn l mt bi ton m li gii ca n c trnh by theo ngn ng th. Di y l mt s v d khc v cc bi ton dng ny. V d 9. Cho 17 im trn mt phng sao cho hai im no cng c ni vi nhau bi ng mt on thng v t bi ch mt trong ba mu xanh; hoc vng. Chng minh rng tn ti mt tam gic c ba cnh cng mu. Li gii. Theo nguyn l Dirichlet, t mi im ta k c 16 on thng v t bi ba mu, do tn ti 6 on thng k t im v t cng mt mu. Gi 6 on thng l M A, M B, M C, M D, M E, M F v chng cng c t vng. Nu tn ti t nht mt on thng ni hai trong 6 im A, B, C, D, E, F t mu vng th bi ton c chng minh (v d on AB t vng th tam gic M AB tho mn) Nu khng tn ti on thng no ni hai trong 6 im y t mu vng, tc l 6 im A, B, C, D, E, F c ni vi nhau bi cc on thng t bng mt trong hai mu xanh hoc . p dng kt qu v d 1, ta c iu cn chng minh. Tm li ta lun tm c mt tam gic c ba cnh cng mu. V d 10. Cho 6 im trn mt phng trong ba im no cng l nh ca mt tam gic c di ba cnh khc nhau. Chng minh tn ti mt on thng ni hai trong 6 im y sao cho n va l cnh nh nht ca tam gic ny, va l cnh ln nht ca tam gic khc. Li gii. Vi mi tam gic c to thnh t ba trong 6 im, ta t cnh ln nht. Nh vy bt c tam gic no cng phi c cnh . Lm thao tc trn vi tt c cc tam gic. Cui cng nhng on thng cha c t mu, ta t mu xanh. Theo v d 1, tn ti mt tam gic ABC c ba cnh cng mu. V tam gic no cng c cnh nn tam gic ny c ba cnh cng mu . Do vy, gi s BC l cnh nh nht ca tam gic ny th BC chnh l on thng cn tm bi n l cnh nh nht ca tam gic ABC v l cnh ln nht ca mt tam gic khc nn mi c t .

2.2. Bi ton t mu bng vung


i vi cc bi ton t mu lin quan n bng vung, ngoi cch t mu mt s ca bng mt cch hp l, ta cng c th nh s hoc nh to cc i n li gii ca bi ton. Di y l mt s v d: V d 11. Hi c th lt mt bn c quc t b mt hai hai gc i din (nh hnh v) bng cc qun c min 1 2 c hay khng?

124

Li gii. T mu phn cn li ca bn c nh hnh di.

Nhn thy mi qun min khi lt vo bng lun chim mt en v mt trng. Nu lt c phn cn li ca bng bi cc qun min th s en v s trng ca bng phi bng nhau. Nhng trong hnh v ta li c 32 en v ch c 30 trng. Nh vy khng th lt c cc qun min tha mn yu cu bi ton. V d 12. Cho mt hnh ch nht 3 7 c chia thnh 21 vung con. Chng minh rng tn ti mt hnh ch nht to bi cc vung con m 4 vung con 4 gc ca hnh ch nht y c t bi cng mt mu. Li gii. Nu tn ti mt ct c ba vung c t cng mu th bi ton c chng minh v khi chc chn tn ti mt ct khc c hai c t cng mu vi ba .T tm c hnh ch nht cn tm. Nu ct no cng ch c t bi ng hai mu. Ta thy s cch t mu ba vung mi ct l 6, do theo nguyn l Dirichlet tn ti hai ct c cch t mu ging nhau. V t hai ct ny ta tm ra c hnh ch nht tho mn yu cu bi ton. V d 13. C th nh s cc ca mt bng vung 4 4 bi cc s t nhin t mt n 16 (mi s vit mt ln) sao cho tng 4 s mi phn ca bng vung c dng nh hnh ch T di y (c th xoay v mi pha) u chia ht cho 4 hay khng?

Li gii. Ta s chng minh rng khng c cch nh s no tho mn yu cu bi ton. Gi s tn ti cch nh s tho mn yu cu bi ton. Ta xt mt phn ca bng vung nh hnh di y.

125 . . Ta c: (a + b + n + d) . 4 v (a + b + n + c) . 4. Suy ra c d (mod 4) . . Tng t a b c d (mod 4) Nh vy nu mt s a chia cho 4 d m th v tr cho vi n cng chia 4 d m. Ta t mu bng vung 4 4 bi cc mu en trng xen k. Gi s s a v tr en, th th cc s v tr en khc (tr hai en gc) u chia 4 d m, suy ra c 6 en cha s c cng s d trong php chia cho 4.
m m m m m

Trong bng khng th c 6 s c cng s d khi chia cho 4 v trong cc s t 1 n 16 ch c 4 s chia 4 d 0, 4 s chia 4 d mt, 4 s chia 4 d hai, 4 s chia 4 d ba. Vy khng th nh s cc ca bng vung tho mn yu cu bi ton.

2.3. Cc bi ton khc


V d 14. Ngi ta t mt s cnh ca mt a gic li v t xanh cc cnh cn li. Bit rng tng di cc cnh nh hn na chu vi a gic, v khng c 2 cnh k nhau no c t cng mu xanh. Hi a gic c th l a gic ngoi tip mt ng trn c hay khng? Li gii. Gi s BC l cnh xanh, AB, CD l cc cnh k vi cnh BC. Theo gi thit, AB v CD l cc cnh . Gi s a gic ngoi tip c ng trn (O). Gi M, N, P l cc tip im ca ng trn trn AB, BC, CD. Ta c BC = BN + N C = BM + CP . Do tng di cc cnh xanh nh hn hoc bng tng di cc cnh (du nh hn xy ra khi tn ti hai cnh k nhau. Du bng xy ra khi khng tn ti hai cnh k nhau). T suy ra tng di cc cnh xanh khng ln hn na chu vi a gic, tc l tng di cc cnh ln hn hoc bng na chu vi a gic, tri vi gi thit. Vy a gic cho khng th ngoi tip mt ng trn c. V d 15. Cho mt ng trn. Ta t mu xanh mt s cung ca ng trn, tng di cc cung mu xanh ca ng trn nh hn na chu vi ng trn. Chng minh rng tn ti mt ng knh ca ng trn m hai u khng b t mu. Li gii. T cc cung i xng vi cc cung xanh qua O. Do tng di cc cung xanh nh hn na chu vi ng trn nn tng di cc cung xanh v cung nh hn chu vi ng trn. Suy ra tn ti mt im A khng c t mu xanh hay . im B i xng vi im qua O cng khng c t mu v v th, ng knh AB l ng knh cn tm. V d 16. C 13 vung xp thnh mt hng ngang, mi c t bi mt trong hai mu xanh hoc . Bit rng s m bn phi l cng mu bng s m bn phi l khc mu. u tin bn tri l . Hi cui cng bn phi l mu g? Cng hi nh trn nu s vung l 15 .

126 Li gii. Ta gi mt ln i t mt sang lin k bn phi n l mt bc chuyn. T s 1 n s 13 c 12 bc chuyn. Theo gi thit, s m bn phi l cng mu bng s m bn phi l khc mu, nn s bc chuyn i mu bng s bc chuyn khng i mu. Do c 6 bc chuyn i mu v 6 bc chuyn khng i mu. Nu t a n b c chn bc i mu th b cng mu vi a, nu c l bc i mu th b khc mu vi a. Do vy nu c 13 th t mt n 13 c 6 bc chuyn i mu, do 13 c cng mu vi mt.Tng t, nu c 15 th t mt n 15 c 7 bc chuyn i mu, do 15 c mu xanh.

3. Cc bi ton tng hp
Bi ton 1. Mi im trn mt phng c t bi mt trong hai mu. Chng minh rng tn ti mt tam gic vung c di cnh huyn bng mt v mt gc nhn bng 60 v ba nh ca tam gic ny c cng mu. Li gii. Gi s ta t bng hai mu en v trng. Theo nh v d 2 th tn ti mt tam gic u ABC c ba cnh bng 1 v c hai trong ba nh ca n c cng mu. Gi AXY BZT l mt lc gic u nhn AB lm ng cho ln nht. Nu mt trong cc nh X, Y, Z, T (chng hn X) c mu trng th ba nh ca tam gic ABX (c AXB = 90 , XAB = 60 ) s c ba nh cng mu trng nh yu cu bi. Ngc li, hin nhin ta thy tam gic XY T (c Y XT = 90 , XY T = 60 ) c ba nh cng mu en. Vy ta c iu cn chng minh. Bi ton 2. Mi im trn mt phng c t bi mt trong hai mu. Chng minh tn ti mt tam gic m cc nh v trng tm ca n c t cng mt mu. Li gii. Ly 7 im bt k trn mt phng sao cho khng c b ba im no thng hng. Theo nguyn l Dirichlet tn ti ba im cng mu. Gi s l ba im A, B, C v chng c t xanh. A

A G B B C C

Xt trng tm G ca tam gic ABC. Nu G cng c t xanh th tam gic ABC tho mn yu cu bi ton. Nu G c t . Trn tia GA ta ly im A sao cho GA = 4GA; B , C c nh ngha tng

127 t. Khi ta c A, B, C, G tng ng l trng tm cc tam gic A BC, AB C, ABC , A B C . Nu tn ti mt trong ba im A , B , C c t xanh, gi s A t xanh th khi tam gic A BC c ba nh v trng tm cng c t xanh tho mn yu cu bi ton. Nu c ba im A , B , C cng c t th tam gic A B C c ba nh v trng tm G cng t tha mn yu cu bi ton. Vy bi ton c chng minh. Bi ton 3. Mi im trn mt phng c t bi mt trong ba mu. Chng minh rng tn ti hai im cng mu c khong cch bng mt. Li gii. Xt hnh thoi ABCD vi cc tam gic ABD, BCD l cc tam gic u. Theo nguyn l Dirichlet tn ti hai im cng mu. Nu hai im y cng l nh ca mt trong hai tam gic u ABD hoc BCD th l hai im cn tm. Nu ngc li th A v C phi cng mu. Xt tp hp cc hnh thoi nh trn vi A c nh. Nu khng c hnh thoi no tho mn tn ti t nht mt cnh ca mt trong hai tam gic u ABD hoc BCD c hai u cng mu th C phi cng mu vi A. Khi tt c cc im nm trn ng trn (A, 3) c t bi cng mt mu. Do hai im bt k trn ng trn ny c khong cch bng mt s tho mn yu cu bi ton. Bi ton 4. Mi im trn mt phng c t bi mt trong by mu. Hi c phi lun tn ti hai im cng mu c khong cch bng mt hay khng? Li gii. Ta s a ra mt v d chng t rng mt phng c t bng 7 mu nhng khng c hai im cng mu bt k c khong cch bng 1. Ta chia mt phng thnh cc lc gic u bng nhau vi cc cnh bng a v t chng nh cch hnh di y (cc im thuc hai hay ba lc gic c th t bng mt mu bt k trong s cc mu t cc lc gic ). 2 A 1 5

7 6

3 4 C

7 6

2 B 1 5

3 4

Khi khong cch ln nht gia cc im cng mu nm cng mt lc gic khng qu 2a, cn khong cch gia cc im cng mu cc lc gic khc nhau khng nh hn di on thng AB. Ta c AB 2 = AC 2 + BC 2 = 4a2 + 3a2 = 7a2 > (2a)2 . 1 1 Do nu ta chn a sao cho 2a < 1 < 7a, hay 7 < a < 2 th khong cch gia cc im cng mu khng th bng mt. Vy khng phi lun tm c hai im cng mu c khong cch bng 1. Bi ton 5. Mt phng c k c t bng 10 mu sao cho cc cnh nhau (c cnh chung) c t bi cc mu khc nhau, ng thi tt c 10 mu u c s dng. Hai mu c gi l cnh nhau nu u chng c dng t hai cnh nhau. Hi s nh nht cc cp mu cnh nhau c th bng bao nhiu?

128 Li gii. Xt mt tuyn ng i qua cc ca tt c 10 mu v khng i qua mt nh no ca cc .Trn mt t giy khc ta v 10 im c t bng cc mu cho, mi ln khi ta i t mu ny sang mu khc, ta s ni bng mt on thng cc im ca cc mu . Cui cng ta s c mt hnh gm n nh (trong trng hp ny n = 10) c ni bng nhng on thng, ng thi theo cc on thng c th i t mt nh bt k sang mt nh bt k khc. Bng phng php quy np theo n, ta c th chng minh c hnh nh th c khng t hn n 1 on thng. V mi on thng tng ng vi mt cp cc mu cnh nhau, nn s cc cp cc mu cnh nhau khng nh hn 9. Ta ly v d mt cch sn vi 9 cp mu cnh nhau c th lp c nh sau: u tin t mu th nht vo cc th t nh bn c. Sau ta sn cc cha c t mu bng cc mu cn li theo mt th t bt k sao cho tt c cc mu u c s dng. Khi trong s hai cnh nhu bt k lun c mt c t bng mu th nht. Bi ton 6. Cho 9 im trn mt phng, trong bt c ba im no cng to thnh mt tam gic m cnh c t mu xanh hoc , nhng lun c cnh . Chng minh tn ti mt t gic c cc cnh v ng cho c t cng mu . Li gii. Nu tn ti mt im A no sao cho t im xut pht t nht 4 on thng mu xanh (gi s l AB, AC, AD, AE) th bi ton c chng minh (4 im B, C, D, E) tho mn. C B D

Nu im no trong 9 im cng ch l u mt ca nhiu nht ba on thng xanh, ta thy rng khng th xy ra trng hp c 9 im u l u mt ca ng ba on thng xanh bi khi y s on thng xanh l 93 khng l s nguyn. 2 Nh vy tn ti t nht mt im sao cho n l u mt ca nhiu nht hai on thng xanh, ng ngha vi n l u mt ca t nht 6 on thng . Gi s im l A v 6 on thng l AB, AC, AD, AE, AF, AG. Theo v d 1, trong 6 im B, C, D, E, F, G tn ti mt tam gic c ba cnh cng mu (gi s l tam gic BCD). Theo bi ra th tam gic no cng c cnh mu nn tam gic BCD c ba cnh cng mu Do 4 im A, B, C, D tho mn yu cu bi ton. Bi ton 7. Trn mt phng cho 18 im, sao cho khng c ba im no trong chng thng hng. Ni tng cp im v t mu cho mi on thng thu c bng mt trong hai mu xanh hoc . Chng minh rng tm c mt t gic m cc nh ca n nm trong tp im cho sao cho cnh v ng cho ca n cng mu.

129 Li gii. Mt im M s c ni vi 17 im cn li to thnh 17 on thng, theo nguyn l Dirichlet tn ti 9 on cng mu, gi s l mu xanh. Xt 9 im khc M cc u mt ca cc on thng xanh k t M . Nu tn ti tam gic ABC c ba cnh cng xanh th 4 im M, A, B, C tho mn yu cu bi. Nu khng tn ti tam gic no c ba cnh cng xanh, tc l tam gic no c nh l ba trong 9 im y cng c ba cnh Theo bi ton 6, lun tn ti mt t gic c 4 cnh v hai ng cho cng mu . Bi ton 8. Cho nm im A, B, C, D, E trn mt phng sao cho khng c ba im no thng hng. Ni tt c cc cp hai im trong nm im trn bi mt on thng v t chng bi mt trong ba mu xanh, hoc vng. Chng minh tn ti mt ng gp khc khp kn c bn cnh c t bi khng qu hai mu. Li gii. Xt 4 on thng AB, AC, AD, AE. Theo nguyn l Dirichlet tn ti t nht hai on thng cng mu. Gi s hai on AB, AC c t bi cng mt mu xanh. B C

A E

Xt hai on thng DB, DC. Nu tn ti mt on c t xanh th ng gp khc ABDCA c 4 cnh c t bi khng qu hai mu. Xt hai on thng EB, EC. Nu tn ti mt on c t xanh th ng gp khc ABECA c 4 cnh c t bi khng qu hai mu. Nu c 4 on thng DB, DC, EB, EC u khng c t xanh th ng gp khc BDCEB c 4 cnh c t bi khng qu hai mu. Vy ta c iu cn chng minh. Bi ton 9. Cho nm im A, B, C, D, E trn mt phng sao cho khng c ba im no thng hng. Ni tt c cc cp hai im trong nm im trn bi mt on thng v t chng bi mt trong hai mu xanh hoc mt cch tu . Chng minh rng lun tn ti mt ng gp khc khp kn c t bi cng mt mu. Li gii. Nu trong 5 im tn ti mt l u mt ca t nht ba on thng cng mu (gi s l im A v cc on thng AB, AC, AD cng c t xanh). Khi nu mt trong cc on thng BC, BD, CD c t xanh th ta nhn c mt ng gp khc c ba cnh cng mu (v d BC c t xanh th ng gp khc ABC c ba cnh cng mu xanh) Nu c ba on thng BC, BD, CD u c t th ng gp khc BCD tho mn yu cu bi ton.

130 B C A D

Xt trng hp t mi im trong 5 im ch xut pht ng hai on thng xanh, hai on thng . B C A E Xt im A. Ta gi s AB, AC t xanh; AD, AE t . Khng mt tnh tng qut, gi s CD t xanh. T C xut pht ng hai on thng xanh (l CA, CD) nn CB, CE phi t . T E xut pht ng hai on thng (l EA, EC) nn EB, ED phi t xanh. Do ng gp khc ABEDCA c 5 cnh cng t xanh tho mn yu cu bi ton. T cc trng hp xt trn, ta suy ra iu cn chng minh. Bi ton 10. Mt a din li trong khng gian c tt c cc mt u l hnh tam gic. Vi mi cnh AB ca a din ta v mt mi tn c chiu t A n B hoc t B n A sao cho mi nh bt k u c t nht mt mi tn i vo v mt mi tn i ra. Chng minh rng tn ti mt mt ABC ca a din c cc mi tn trn cc cnh c nh du cng chiu. Li gii. bi ton ny chng ta s a ra mt khi nim mi l gc cng mu. Ta nh ngha gc ABC c gi l gc cng mu khi v ch khi AB v AC c nh du mi tn theo cch c mt mi tn i vo B v mt mi tn i ra t B. D

Gi s tn ti mt cch v mi tn m khng tn ti mt no ca a din c cc mi tn trn cc cnh c nh du cng chiu. Gi D, M ; C ln lt l s nh, s mt v s cnh ca a din. Theo h thc Euler ta c D+M C =2 (1) Theo gi s phn chng ta thy rng do khng tn ti mt no ca a din c cc mi tn trn cc cnh c nh du cng chiu, m cc mt ca a din u l hnh tam gic nn

131 mi mt ca a din ch c ng mt gc cng mu. Do c tng cng M gc cng mu. Mt khc xt mt nh bt k ca a din. Gi s ti nh c x mi tn i ra v y mi tn i vo v x y. Theo bi ra th x 1 v y 1. S gc cng mu ti nh ny l xy. Ta li c mt iu na l xy (x + 1)(y 1) = x y + 1 0. Do ti mt nh bc n (tc l c n cnh c u mt l nh ny) th c t nht n 1 gc cng mu. Do s gc cng mu khng b hn 2C D. T y ta suy ra bt ng thc M 2C D (2)

T (1) v (2) suy ra D + m 2C hay 2 C. iu ny khng th xy ra i vi a din li nn bt ng thc sai, dn ti gi s sai v ta c iu cn chng minh. Bi ton 11 (IMO 1992). Trong khng gian cho 9 im sao cho khng c bt c h 4 im no ng phng. C hai im th c ni vi nhau bng mt on thng v c t bi ch mt trong hai mu xanh hoc , hoc khng t g c. Tm gi tr nh nht ca n sao cho c ng n on thng c t mu th tp hp cc cnh c t phi cha mt tam gic c ba cnh cng mu. Li gii. Vi h gm 5 im bt k v c 10 cnh u c t mu xanh hoc th kt qu bi ton khng ng. Cch t mu nh di y s ch ra iu (on thng c t bng nt t tng trng cho on thng ni hai im m khng t mu)

Vi h gm 6 im bt k m c 15 cnh u c t mu xanh hoc th s lun tn ti mt tam gic m c ba cnh cng mu. ( chng minh v d 1) Xt n h 9 im. Nu ch c ba cnh khng c t mu th s cnh c t mu l 363 = 33, do lun tn ti mt h 6 im trong 9 im y m tt c 15 cnh u c t mu. Theo nh chng minh trn th h 6 im y s cha mt tam gic c ba cnh cng mu. Nh vy vi n = 33 th s lun m bo c c mt tam gic c ba cnh cng mu. Vi n = 32. Ta s ch ra mt trng hp t mu m khng tn ti tam gic c ba cnh cng mu. Xut pht t 5 im A, B, C, D, E, ta thm im P ri ni vi B, C, D, E v t mu cc cnh P B, P C, P D, P E sao cho chng tng ng cng mu vi AB, AC, AD, AE. Khi ta c h 6 im, 14 cnh c mu nhng khng c tam gic no c ba cnh cng mu.

132 B

C P

Ta thm im Q ri ni vi P, A, C, D, E v t mu cc cnh QP, QA, QC, QD, QE sao cho chng tng ng cng mu vi BP, BA, BC, BD, BE. Khi ta c h 7 im, 19 cnh c mu nhng khng c tam gic no c ba cnh cng mu. Sau ta thm im R ri ni vi P, Q, A, B, D, E v t mu cc cnh RP, RQ, RA, RB, RD, RE sao cho chng tng ng cng mu vi CP, CQ, CA, CB, CD, CE. Khi ta c h 8 im, 25 cnh c mu nhng khng c tam gic no c ba cnh cng mu. Cui cng ta thm im S ri ni vi P, Q, R, A, B, C, E v t mu cc cnh SP, SQ, SR, SA, SB, SC, SE sao cho chng tng ng cng mu vi DP, DQ, DR, DA, DB, DC, DE. Khi ta c h 9 im, 32 cnh c mu nhng khng c tam gic no c ba cnh cng mu.

Vy n = 33 l s nh nht tho mn yu cu bi ton.

Bi ton 12. Cho n im (n 3) trong khng c ba im no thng hng. Hai im bt k c ni vi nhau bng mt on thng, mi on thng c t bi mt mu xanh, hoc vng. Bit rng: c t nht mt on thng mu xanh, mt on thng mu , mt on thng mu vng, khng c im no m cc on thng xut pht t c c ba mu v khng c tam gic no to bi cc on thng ni c ba cnh cng mu. (a) Chng minh rng khng tn ti ba on thng cng mu xut pht t cng mt im. (b) Hy cho bit c nhiu nht bao nhiu im tho mn yu cu bi ton. Li gii. (a) Gi s tn ti im A m t c ba on thng AB, AC, AD cng c t xanh. V khng c tam gic no c ba cnh cng mu nn tam gic BCD c ba cnh c t bi ng hai mu v vng. Do chc chn tn ti mt im trong ba im B, C, D m t xut pht ba on thng c c ba mu (v d tam gic BCD c cnh BC, BD t v CD t vng, nh th th t im C ta c CA t xanh; CB t v CD t vng), iu ny tri vi

133 gi thit khng c im no m cc on thng xut pht t c c ba mu, suy ra gi s sai v ta c iu cn chng minh. (b) T bi ra kt hp vi chng minh cu (a), ta c cc on thng xut pht t mt im c khng qu hai mu v c khng qu hai on thng cng mu. Nh vy t mt im xut pht khng qu 2 2 = 4 on thng, tc l n 5. Vi n = 5. Gi s 5 im A, B, C, D, E tho mn yu cu bi ton. Theo lp lun trn, khng mt tnh tng qut, ta c th gi s t im A xut pht hai on thng xanh AB, AC v hai on thng AD, AE. Do khng c tam gic no c ba cnh cng mu nn BC t hoc vng. Nu BC t , suy ra BD v BE khng th t vng, v cng khng th cng t xanh hoc cng t , nh vy c mt on t , mt on t xanh. Vai tr D, E nh nhau nn ta c th gi s BD t v BE t xanh. Chng minh tng t nh th ta c cc on CD, CE, DE ch c th t bi hai mu xanh hoc , nh vy tc l khng c on thng no t vng (Tri vi gi thit c t nht mt on thng mu vng) Nu BC t vng, suy ra BD v BE khng th t , v cng khng th cng t xanh hoc cng t vng, nh vy c mt on t xanh, mt on t vng. Vai tr D, E nh nhau nn ta c th gi s BD t vng v BE t xanh. Xt im D. Ta c DA t v DB t vng, do DC phi t hoc vng. Nhng v BD v BC t vng nn DC phi t DE t vng. Xt im E ta c EA t , ED t vng, EB t xanh (mu thun vi kt qu cu (a)) Tm li n khng th bng 5. Vi n = 4, ta ch ra mt h im tho mn yu cu bi ton nh hnh v di y B A

D Vy s im ln nht tho mn yu cu bi ton l 4.

Bi ton 13. Cho hnh chp c y l a gic 9 cnh. 27 ng cho ca mt y v 9 cnh bn ca hnh chp c t bi hai mu xanh, . Chng minh bao gi cng tm ra c mt tam gic c ba cnh cng mu. Li gii. Theo nguyn l Dirichlet tn ti 5 cnh bn ca hnh chp c t cng mu. Gi s l mu .

134 Xt ng gic A1 A2 A3 A4 A5 vi A1 , A2 , A3 , A4 , A5 c 5 nh l nh ca a gic 9 nh A1 A2 . . . A9 v tng ng l 5 u mt khc S ca 5 on thng (im A no ca ng gic c ch s ln hn th tng ng trng vi nh A ca a gic 9 nh c ch s ln hn) Nu tn ti mt on thng ni hai trong 5 im A1 , A2 , A3 , A4 , A5 t mu th bi ton c chng minh (gi s A1 A3 t th tam gic SA1 A3 c 3 cnh ) Nu iu ny khng xy ra, tc l tt c cc on thng ni hai trong 5 im A1 , A2 , A3 , A4 , A5 phi c t mu xanh. Ta thy rng tt c cc ng cho ca ng gic A1 A2 A3 A4 A5 u c t mu v lun tn ti t nht mt cnh ca ng gic c t mu (bi v khng xy ra trng hp c 5 cnh ca ng gic A1 A2 A3 A4 A5 u l cnh ca a gic 9 nh A1 A2 . . . A9 ). Khi tam gic c ba nh m hai trong s l u mt ca cnh ng gic A1 A2 A3 A4 A5 c t mu, nh cn li l nh i din vi cnh , l tam gic cn tm. Bi ton c chng minh. Bi ton 14 (IMO 1979). Cho hnh lng tr c y trn v y di l hai ng gic A1 A2 A3 A4 A5 v B1 B2 B3 B4 B5 . Mi cnh ca hai ng gic cng nh cc on thng Ai Bj (i, j = 1, 5) u c t bi ch mt trong hai mu xanh hoc . Bit rng bt k tam gic no c ba nh l nh ca lng tr m c ba cnh u c t mu th lun tn ti hai cnh c mu khc nhau. Chng minh tt c 10 cnh ca hai ng gic ny ( y trn v y di) c cng mt mu. Li gii. Trc tin ta chng minh rng tt c 5 cnh ca ng gic A1 A2 A3 A4 A5 u c cng mu. Tht vy, gi s iu ngc li xy ra, tc l tn ti hai cnh k nhau ca ng gic A1 A2 A3 A4 A5 khc mu. Gi s cnh A1 A2 t xanh v A1 A5 t . Xt 5 on thng A1 Bi vi 1, 5. Theo nguyn l Dirichlet tn ti ba trong 5 on thng y cng mu, khng mt tnh tng qut c th gi s ba on thng ny mu xanh, v ba cnh l A1 Bk , A1 Bm , A1 Bn . Lc ny ta xt cc tam gic A1 A2 Bk , A1 A2 Bm , A1 A2 Bn , ta c ba cnh A2 Bk , A2 Bm , A2 Bn cng mu . Mt khc hai trong ba nh Bk , Bm , Bn l hai nh k nhau ca ng gic B1 B2 B3 B4 B5 nn chng c t xanh hoc , gi s hai nh l Bk , Bm . T y ta thy: Nu Bk Bm t xanh th tam gic A1 Bk Bm c ba cnh t xanh. Nu Bk Bm t th tam gic A2 Bk Bm c ba cnh t . T y suy ra iu mu thun nn gi s sai. Nh vy 5 cnh ca ng gic A1 A2 A3 A4 A5 c cng mu. Hon ton tng t ta chng minh c 5 cnh ca ng gic B1 B2 B3 B4 B5 t cng mu. By gi ch cn chng minh mu cc cnh ca hai ng gic ging nhau th bi ton kt thc. Gi s ngc li, chng hn 5 cnh ca ng gic A1 A2 A3 A4 A5 mu xanh, cn ca ng gic B1 B2 B3 B4 B5 mu . Chng minh tng t nh trn ta c ba trong 5 cnh A1 Bi vi i = 1, 5 c cng mu v hai trong ba nh Bi l hai nh k nhau, t ta c hai nh ny to vi A1 thnh tam gic c ba cnh (mu thun). Nh vy ba trong 5 cnh A1 Bi vi i = 1, 5 c cng mu xanh. Lp lun tng t ta c ba trong 5 cnh A2 Bi vi i = 1, 5 c cng mu xanh.

135 Trong s ba cnh A1 Bi (tp hp mt) v ba cnh A2 Bi (tp hp hai) va ch ra trn, chc chn c Bk no cng tp mt v tp hai. Khi tam gic A1 A2 Bk c ba cnh mu xanh (mu thun), do gi s sai. Tm li ta c iu cn chng minh. Bi ton 15. T mu cc nh ca a gic u 12 cnh bng hai mu khc nhau. Hy tm tt c cc cch t mu sao cho khng c a gic u no c cc nh cng mu. Li gii. Ta gi mt a gic c cc nh cng mu l a gic cng mu. Trong a gic u 12 cnh A1 A2 . . . A12 c 4 tam gic u (A1 A5 A9 , A2 A6 A10 , A3 A7 A11 , A4 A8 A12 ), 3 hnh vung (A1 A4 A7 A10 , A2 A5 A8 A11 , A3 A6 A9 A12 ) v hai lc gic u. Nu cc nh ca lc gic u cng mu th s c hai tam gic u c cc nh cng mu. Do vy ta ch cn quan tm n s cch t mu cc nh ca cc tam gic u v cc hnh vung lit k trn. Vi mi tam gic u, s cch t mu khng c ba nh cng mu l 23 2 = 6 cch, do c tt c 64 = 1296 cch t mu sao cho khng c tam gic u cng mu. Trong 1296 cch t ny c nhng cch t khng tho mn yu cu bi ton v tn ti hnh vung cng mu. By gi ta s m s cch t mu sao cho tn ti t nht mt hnh vung cng mu. c mt hnh vung cng mu ta c 2 34 = 162 cch v chng hn nu mt hnh vung c cng mu , ng ngha vi vic mi tam gic u chc chn c mt nh mu th hai nh cn li ca mi tam gic khng cng mu ch c th nhn mt trong ba trng thi mu l : xanh - , - xanh, xanh - xanh. c hai hnh vung cng mu (cc nh ca hai hnh vung ny c th c cng mu hoc 4 nh hnh vung ny khc mu 4 nh hnh vung kia) ta c s cc cch t l: 2 + 2 24 = 34 cch. c ba hnh vung cng mu (lu ch cn xt trng hp mi hnh vung c 4 nh cng mu nhng c 12 nh ca ba hnh vung khng cng mu), c tt c 6 cch. Do vy s cc cch t mu khng c a gic u no cng mu l 1296 (3 162 (3 34 6)) = 906 (cch) Bi ton 16. Cho lc gic u ABCDEF , trong nh A c t , cc nh cn li c t xanh. i mu cc nh ca lc gic theo quy tc mi ln i mu ng thi ba nh lin tip (xanh thnh , thnh xanh). Hi sau mt s hu hn ln i mu nh vy c th t c kt qu l nh B t cn cc nh cn li t xanh hay khng? Li gii. Cu tr li l khng. Tht vy. Xt hai cp nh i xng nhau qua tm O ca lc gic l (A, D) v (C, F ). Khi ta i mu ba nh lin tip ca lc gic th khng bao gi xy ra trng hp c hai im trong mt cp ni trn u i mu.

136 A F E O B C D

Ban u im A t , im D t xanh nn mun chng cng mu xanh th cn mt s l ln i mu. Trong khi , ban u im C t xanh, im F t xanh nn mun chng vn cng mu xanh th cn mt s chn ln i mu. Hai iu trn mu thun nhau nn khng xy ra trng hp B t , cc nh cn li t xanh sau mt s hu hn ln i mu. Nhn xt : Bi ton s dng phng php phn chng bng cch ch ra sau mt s m ln i mu, nu xt mt cp nh ny th m l s chn, xt mt cp nh khc th m l s l. i vi bi ton tng qut hn cho 2n gic u (n N; n 3) th ta cng c li gii tng t nh trn. Bng phng php tng t, ta gii quyt c hai bi ton sau : Bi ton 17. Cho lc gic u ABCDEF , trong nh A v D c t , cc nh cn li c t xanh. i mu cc nh ca lc gic theo quy tc mi ln i mu ng thi ba nh ca mt tam gic cn c nh l nh lc gic (xanh thnh ; thnh xanh). Hi sau mt s hu hn ln i mu nh vy c th t c kt qu l nh B t cn cc nh cn li t xanh hay khng? Bi ton 18. C 41 hc sinh ca hai lp Ton v Vn xp thnh mt hng dc. Bit rng s hc sinh m ngi ng lin trn l bn cng lp bng s hc sinh m ngi ng lin trn l bn khc lp. Ngi ng u l hc sinh lp Ton. Hi ngi ng cui cng l hc sinh lp no? Nu c 43 hc sinh th ngi ng cui l hc sinh lp no? Tng qut bi ton : Nu c 4k + 1 hc sinh th hc sinh ng u v hc sinh ng cui cng lp. Nu c 4k + 3 hc sinh th hc sinh ng u v hc sinh ng cui khc lp. Bi ton 19. Cho hai a hnh trn bng nhau, mi a c chia thnh 12 hnh qut bng nhau v t mu 5 hnh qut mt cch tu . t a th hai ln trn a th nht sao cho cc hnh qut hai a trng nhau. Ln lt quay a th hai quanh tm ca n mt gc 30 theo chiu kim ng h, ta c tt c 12 v tr. Chng minh rng trong 12 v tr , s v tr c t ba cp hnh qut trng nhau tr ln khng qu 8. Li gii. Mi hnh qut c t mu a th nht s ln lt trng vi 5 hnh qut a th hai. Nh vy s cp hnh qut trng nhau trong 12 v tr l 5 5 = 25.

137

Gi a l s v tr c t ba cp hnh qut trng nhau tr ln v n l s cp hnh qut trng nhau trong a v tr Ta c 3a n v n 25. Suy ra 3a 25 hay a 8. Bi ton 20. Cho hai a hnh trn bng nhau, mi a c chia thnh 2000 hnh qut bng nhau v t mu 200 hnh qut mt cch tu . t a th hai ln trn a th nht sao cho cc hnh qut hai a trng nhau. Ln lt quay a th hai quanh tm ca n mt gc 360 theo chiu kim ng h, ta c tt c 2000 v tr. Chng minh rng trong 2000 v tr , c 2000 t nht 96 v tr c khng qu 20 cp hnh qut trng nhau. Li gii. Gi m l s v tr c khng qu 20 cp hnh qut trng nhau tr ln v n l s v tr c t 21 cp hnh qut trng nhau tr ln. Khi m + n = 2000. Tng t bi ton trn, ta c 21n 40000. V n l s t nhin nn n 1904. Do m 2000 1904 = 96. Bi ton 21. Cho mt lc gic li. Chng minh c th chia lc gic thnh cc tam gic con v t hai mu en trng sao cho hoc c cnh chung (khi chng c t bi hai mu khc nhau), hoc c chung nh, hoc khng c im chung. Ngoi ra mi cnh ca lc gic l cnh ca mt tam gic mu en. Nu thay lc gic li bng a gic c 8 cnh, 10 cnh th bi ton cn ng hay khng? Li gii. Hnh v di y s ch ra mt cch t tho mn yu cu bi ton khi a gic c 6 cnh.

Ta s chng minh nu a gic c k cnh m k khng l bi ca ba th bi ton khng cn ng. Tht vy. Gi s tam gic trng l n. Ta c s cnh tam gic trng l 3n. Mi cnh ca mt tam gic trng s l cnh ca duy nht mt tam gic en, mt khc ta c mi cnh ca a gic l cnh ca mt tam gic en, do s tam gic en l 3n+k . 3 V k khng l bi ca 3 nn 3n+k khng l s nguyn (v l) 3 T y ta suy ra nu a gic c 8 hay 10 cnh th bi ton khng cn ng. Bi ton 22. Trong mt phng cho bt gic li A1 A2 . . . A8 m khng c ba ng cho no ca n ng quy ti mt im (khc nh a gic). Ta gi mi giao im ca hai ng cho ca bt gic l mt nt.

138 Xt cc t gic li m mi t gic u c c 4 nh l nh ca bt gic. Ta gi mi t gic nh vy l t gic con. Hy tm s nguyn dng n nh nht c tnh cht: C th t mu n nt sao cho vi mi i, k {1; 2; 3; 4; 5; 6; 7; 8} v i khc k, nu k hiu S(i, k) l s t gic con nhn Ai , Ak lm nh v ng thi c giao im hai ng cho l mt nt c t mu th tt c cc gi tr S(i, k) u bng nhau. Li gii. Gi s c mt phng n t mu tha mn yu cu bi ton. t S(i, j) = k vi k l s nguyn dng. Ta thy rng mt im c t mu chung cho 6 cp im phn bit (Ai , Aj ) m Ai , Aj l nh ca t gic c giao im ca cc ng cho c t mu . 2 S cp (Ai , Aj ) l C8 = 28 nn s cp im c t mu l n = 28k = 14k . 6 3 4 Do n nguyn dng v n C8 = 70 nn k chia ht cho ba, do n 14. Ta s ch ra mt phng n t mu vi n = 14 tng ng vi k = 3 tho mn yu cu bi ton. Trn hnh di y cc im t mu c nh du chm. A3 A2 A1 A5 A8 A7 Vy n nh nht bng 14. A6 A4

Mt cu hi na c t ra l : Vi nhng gi tr no ca n ta c th t n trong s 70 giao im ca cc ng cho ca cc t gic li con c c tho mn iu kin bi ton? T cng thc n = 14k suy ra n l bi ca 14 v k {3; 6; 9; 12; 15}. Hn na ta thy rng nu 3 c mt phng n t mu n im tho mn yu cu bi ton th cng c mt phng n t mu (70 n) im tho mn yu cu bi ton bng cch i li cch gi im t mu thnh cc im khng t mu v ngc li. Nh vy ng vi mt cch t mu n = 14 (k = 3) s c cch t mu n = 56 (k = 12) im. Vi n = 28 (k = 6) ta ch ra cch t mu tho mn yu cu bi ton nh hnh di y : A3 A2 A1 A5 A4

A8 A7 A6

139 Nh vy th vi n = 70 28 = 42 (k = 9) s c phng n t mu tng ng tho mn. Cui cng hin nhin vi n = 70 (k = 15) c phng n t mu tt c 70 im tho mn. Do vi mi s nguyn dng n, n 70, n l bi ca 14 th u c phng n t mu tho mn yu cu bi ton. Bi ton 23. Cho mt bn c hnh vung kch thc 6 6. C th dng 9 mnh g hnh ch nht kch thc 1 4 ghp kn bn c c hay khng? Li gii. (i) Cch 1. T mu bn c nh hnh v.

Khi t ln bn c mi mnh g che lp mt en v ba trng, do 9 mnh g che lp 9 en. Nhng trn bn c c 10 en nn khng th dng 9 mnh g 1 4 lp kn bn c. (ii) Cch 2. T mu bn c nh hnh di.

Khi t ln bn c, mi mnh g 1 4 che lp hai en v hai trng. Do 9 mnh g che lp 18 en. Nhng bn c c 20 en nn khng th dng 9 mnh g 1 4 che lp bng c. Bi ton 24. Trn giy k vung c th t mu 8 sao cho mi c t u c mt s l cc bn cnh c t hay khng? (hai cnh nhau l hai c mt cnh chung). Cng hi nh trn nu s l 9 Li gii. Nu s l 8, ta t mu nh hnh di y

Nh vy nu s l 8 th ta c cch t tho mn. Ta s chng minh rng nu s phi t l 9 th khng tn ti cch t tha mn yu cu bi ton. Gi s c 9 c t trong gm x c mt bn cnh c t mu, y c ba bn cnh c t mu. Ta c x + y = 9. Gi m l s cnh chung ca cc c t, do mi cnh l chung ca hai nn m = x+3y . 2 Ta c m = x+y + y. Suy ra x + y chn. iu ny mu thun vi x + y = 9. 2 Vy khng th t mu 9 tho mn bi ton.

140 Bi ton 25. Cho n ng thng trn mt phng. Chng chia mt phng y thnh cc min. Chng minh rng c th t mu cc min y bng hai mu trng, en sao cho hai min k nhau c t bi hai mu khc nhau. Li gii. Ta chng minh bi ton bng quy np ton hc. Ta thy vi n = 1, mt phng c chia thnh hai min v hin nhin ta c cch t mu tho mn yu cu bi ton. Gi s bi ton ng n n = k. K thm ng thng th k + 1, ng thng ny chia mt phng thnh hai na mt phng I v II. i vi cc phn na mt phng I, ta gi nguyn nh c. Cn i vi cc phn na mt phng II, ta i mu en thnh trng, trng thnh en. Mt phng c chia ra thnh cc min m hai min k nhau c mu khc nhau. Vy theo nguyn l quy np, ta c iu cn chng minh. Nhn xt : Bi ton vn ng nu ta thay n ng thng bng n ng bt k, trong c c ng trn. Li gii hon ton tng t.

C C TR V B T NG TH C R I R C
Nguyn Hin Trang1

Ton ri rc l loi ton t i hi tnh ton m i hi s thng minh v t duy logic. Cc tr ri rc l mt b phn ca ton ri rc, yu cu tm gi tr ln nht, nh nht ca mt i lng ri rc. Phng php gii dng ton ny l d on gi tr ln nht, nh nht ca biu thc f (x) t ti gi tr no ca x no v chng minh c d on . Trong phng php ny ta thng s dng kin thc ca s hc, t hp, ton ri rc, vic d on thng c tin hnh vo mt s tnh ton ban u.

1. Cc tr trong dy s nguyn
Hn ch cc bin trn tp R+ thnh tp N ca mt s bt ng thc ta s c bt ng thc dng ny. Nhng bt ng thc ny nu nhn c bi ton ban u s kh d dng trong vic chng minh. Ta xt mt s v d sau : V d 1. Cho a, b l cc s nguyn dng. Chng minh rng b a + 1+a 1+b a+b 1 + (aa bb ) a+b
1

Li gii. Nhn thy aa bb = a a a b b b (a tha s a, b tha s b). a 1 1 1 = + + + a+1 a+1 a+1 a+1
a s hng

1 1 1 b = + + + b+1 b+1 b+1 b+1


b s hng

Nh vy ta ngh n mt bt ng thc vi n bin, p dng cho a ln a v b ln b. B . Vi xi 1, i = 1, n ta c:


n

i=1

1 1 + xi

n
n
1 n

(1)

1+
i=1

x1 1:

Chng minh. D chng minh c bt ng thc sau vi a, b 1 1 + 2 1+a 1 + b2


1

2 1 + ab

K39, THPT chuyn Phan Bi Chu, Ngh An.

141

142 Do vi a, b 1 th

1 1 + 1+a 1+b

1+

ab

Theo bt ng thc Jensen th (1) ng vi n = 2 nn (1) ng vi n nguyn ln hn 2. Tr li bi ton. p dng b trn cho : x1 = x2 = = xa = a xa+1 = xa+2 = = xa+b = b Ta c iu cn chng minh. Nhn xt. Vi b trn ta c xy dng c rt nhiu bt ng thc dng nh trn. Bi ton 1. Vi a, b, c nguyn dng, ta c b c a + + 1+a 1+b c+1 a+b+c 1 + (aa bb cc ) a+b+c
1

Bi ton 2. Cho a, b, c, m, n, p nguyn dng. Chng minh ma nb pc + + 1+a 1+b 1+c Ta c th xy dng bi ton 1 bng cch sau : aa S dng nhn xt xx x, x 1 suy ra 1+aa ma + nb + pc 1 + (aa bb cc ) a+b+c
a , 1+a
1

ta c: a+b+c 1 + (aa bb cc ) a+b+c


n i=1
1

aa bb cc + + 1 + aa 1 + b b 1 + c c

Xt bt ng thc sau : Cho a1 , a2 , . . . , an > 0 tha mn: minh rng


n
k

ai = 1, v k nguyn dng. Chng

1 (nk 1)ai + 1

1
n i=1

(2) ai = 1 m bt ng

i=1

Chng minh. Gi s tn ti cc s thc dng a1 , a2 , . . . , an tha mn thc trn khng ng, tc l tn ti p > 1 tha mn:
n
k

1 (nk 1)ai + 1 p
k

i=1

1 p

Tng ng vi

i=1

t k

p (nk 1)ai +1

= xi , ta c
n

(nk 1)ai + 1

=1

(n 1) =

i=1

pk 1 xk i

>
i=1

1 1 , xi > 0, xi

xi = 1
i=1

143 Mt khc vi mi x1 , x2 , . . . , xn > 0 tha mn


n n i=1

xi = 1 th nk 1
n

i=1

1 1 xk i

Tht vy:
n n

i=1

1 1 xk i

i=1

k1 1 + xi + x2 + + xi i

n i=1

(x1 + + xi1 + xi+1 + + xn )

(x1 x2 xn )k (x1 + + xi1 + xi+1 + + xn ) (n 1)n x1 x2 xn

Ta c

i=1

Ta cn chng minh
n

i=1

1 + x1 + +
1 nG

k1 xi

nk 1 n1

(x1 x2 xn )k1

t G = Ta c

x1 x2 xn , suy ra
n

1. n n (x1 x2 xn )i 1 + xj + +
k1 xj

i=1

xi j 1 + xj + + xi j
n

n
n

(i = 0, n)

i=1

Suy ra

i=1

k1 1 + xj + + xj

(1 + G + + GK1 )n

M
n

i=1

1 + xj + +

k1 xj

k1

+ + n + 1

(G )

n k1

nk 1 n1

(x1 x2 xn )k1

Ta c iu cn chng minh. Nh vy iu gi s sai. Vy (2) ng. T (2), chn n = a + b, ta c a1 = a 2 = = aa = a aa b b = b aa b b 1 aa1 bb = 1 aa bb1

aa+1 = aa+2 = = aa+b = Chn tip k = ab ta c bt ng thc sau

V d 2. Cho a, b l cc s nguyn dng. Chng minh rng a


ab a1 +1 ab

(ab)ab 1 + aa1 bb

ba

b
ab

b1 +1 ab

(ab)ab 1 + aa bb1

ab

144 Xt bt ng thc sau : Cho hai b s thc dng x1 , x2 , . . . , xn v a1 , a2 , . . . , an tha mn


n n

ai =
i=1 i=1

xi . Khi ta c
n n

xai i
i=1 i=1

aa i i

(3)

Chng minh. Bt ng thc cn chng minh tng ng vi


n n

ai ln xi
i=1 i=1

ai ln ai

Hm s f (x) = ln x l hm li nn f (xi ) Suy ra


n

1 f (ai )(x ai ) + f (ai ) = (x ai ) + f (ai ) a

ai f (xi ) Tng ng vi

xi ai + ai + ai f (ai ) =
n n

ai f (ai )
i=1

ai ln xi
i=1 i=1

ai ln ai

Do (3) ng. p dng bt ng thc (3) vi x1 = x2 = = xa = a2 a + b2 + c 2 a2 b + b2 + c 2 c = 2 a + b2 + c 2

xa+1 = xa+2 = = xa+b = xa+b+1 = xa+b+2 = = xa+b+c Chn a1 = a2 = = an =


1 , a+b+c

ta c bt ng thc:

V d 3. Cho a, b, c l cc s nguyn dng. Chng minh rng a bc


a b c

a2 + b 2 + c 2 a+b+c 0 th
n

a+b+c

Xt bt ng thc sau : Cho x1 , x2 , . . . , xn


n n

1 n

xi
i=1 i=1

1 xi

n(n 1)
n

xi
i=1
1 n

+
i=1

1+
i=1

xi

xi 1 + xi

(4)

145 Chng minh. Nu t S =


n i=1 n n
1 n

xi th (4) tng ng vi dy cc bt ng thc sau :

i=1

S xi 1 + xi

n(n 1)
n

xi
i=1
1 n

1+
i=1

xi
n
1 n

i=1

S xi +1 1 + xi

n(n 1)
n

xi
i=1
1 n

+n

1+
i=1

xi
n
1 n

i=1

S xi +1 1 + xi

n2 1+

xi
i=1 n

+n
1 n

xi
i=1

p dng bt ng thc AM-GM v (1), ta c


n

i=1

S xi +1 1 + xi

(1 + S)
i=1

1 1 + xi
1 n

1 + n n =
2 n i=1 n

xi
i=1
1 n

n
n
1 n

1+
i=1

xi

xi xi
i=1

+n
1 n

1+ Do (4) ng. p dng (4) cho:

x1 = x2 = = xa = a xa+1 = xa+2 = = xa+b = b xa+b+1 = xa+b+2 = = xa+b+c = c Ta c bi ton sau: V d 4. Cho a, b, c l cc s nguyn dng. Chng minh rng: a2 b c a + + 1+a 1+b 1+c (a + b + c)(a + b + c 1) aa bb cc 1 + (aa bb cc ) a+b+c
1 1 a+b+c

a2 1+a

V d 5. Cho hai s nguyn dng x, y tha mn x + y = 100. Tm GTNN, GTLN ca (a) A = xy;

146 (b) B = x! + y!; (c) C = x!y!; (d) D=x2 + y 2 f (x) g(x). y.

Li gii. Khng mt tnh tng qut gi s x t d = y x (d 0).

1. Ta chng minh nu d > 1 th A cha t GTLN. Tht vy. Xt cp (x + 1; y 1) ta c (x + 1)(y 1) = xy + y x + 1 > xy. Do A ch t GTLN khi d = 0 hoc d = 1. M x + y = 100 nn A t GTLN khi x = y = 50. Vy max A = 502 x = y = 50. 2. Ta chng minh nu 1 < x y th A cha t GTNN. Tht vy. Chn b (x 1; y + 1), ta c (x 1)(y + 1) = xy + x y 1 < xy. C tip tc nh vy, ta suy ra min A = 99 khi x = 1, y = 99. Tng t vi B, C, D ta c cc kt qu sau: (a) max A = 502 x = y = 50; min A = 99 x = 1, y = 99; (b) min B = 2.50! x = y = 50; max B = 1 + 99! x = 1, y = 99; (c) min C = (50!)2 x = y = 50; max B = 99! x = 1, y = 99; (d) min D = 2.502 x = y = 50; max B = 1 + 992 x = 1, y = 99. Ta c th m rng bi ton trn nh sau : V d 6. Cho n s nguyn dng x1 , x2 , . . . , xn tha mn (a) P = x1 x2 xn ; (b) Q = x2 + x2 + + x2 ; n 2 1 (c) R = x1 ! + x2 ! + + xn !; (d) S = x1 !x2 ! xn !. Li gii. Khng mt tnh tng qut gi s x1 x2 xn t d = xn x1 Sau lm tng t v d 5, ta c cc kt qu sau Nu t k = nq + r, r < n th (a) min P = k n + 1; max P = q nr .(q + 1)r ; (b) min Q = (n r).q 2 + r.(q + 1)2 ; max Q = (n 1) + (k n + 1)2 ; (c) min R = (n r)!.q! + r.(q + 1)!; max R = (n 1)! + (k n 1)!;
n i=1

xi = k. Tm GTLN, GTNN ca

147 (d) min S = (q!)nr . [(q + 1)!]r ; max S = (k n 1)!. Bi ton trn cng c th pht biu di dng sau : V d ly k = 2011; n = 10. 1. Hy vit 2011 thnh tng ca mt s s nguyn dng sao cho tch cc s t GTLN. 2. Cho 10 s nguyn dng tha mn Tm y0 = max {y | y A}.
10 i=1

ai = 2011. t A = {x | x = BCN N (a1 , . . . ; a10 )}.

3. Cho mt s nguyn dng c 2011 ch s c vit t cc ch s 0; 1; . . . ; 9. Ch s i c vit ai ln (i = 0, 9). Tm cc b s ai tn ti nhiu nht cc s c 2011 ch s tha mn trn.

2. Mt s bi ton bt ng thc t hp
2.1. S dng mt s tnh cht ca nh x
Cho A v B l hai tp hu hn. 1. Nu c mt n nh f : A B th |A| |B|;

2. Nu c mt song nh f : A B th |A| = |B|; 3. Nu c mt ton nh f : A B th |A| |B|. Trong mt s bi ton m, nh l trn thng c p dng bng cch : Mun m s phn t ca tp hp A, ta thit lp mt song nh t A n B m B l mt tp hp bit s phn t. Trong bi ton bt ng thc t hp, tng trn thng c p dng nh sau : Mun chng minh |A| |B|, ta tm mt nh x i t A vo B (hoc B A). Sau chng minh nh x l mt n nh (hoc ton nh) nhng khng phi l song nh. V d 7 (IMO 1989). Mt hon v (x1 , x2 , . . . , x2n ) ca tp hp (1, 2, . . . , 2n) (n l s nguyn dng) c gi l c tnh cht P nu |xi xi+1 | = 1 vi t nht mt i {1, 2, . . . , 2n 1}. Chng minh rng mi n, s hon v c tnh cht P ln hn s hon v khng c tnh cht P . Li gii. (i) Cch 1. t A l tp tt c hon v ca {1, 2, . . . , 2n}; B l tp tt c hon v ca {1, 2, . . . , 2n} c tnh cht P ; C l tp tt c hon v ca {1, 2, . . . , 2n} khng c tnh cht P. Ta chia cc s 1, 2, . . . , 2n thnh n cp sau : (1; n + 1), (2; n + 2); . . . ; (n; 2n). Gi s (x1 ; x2 ; . . . ; xk1 , xk ; xk+1 , . . . ; x2n ) l mt hon v thuc C. Gi s xk l s cng cp vi x2n , suy ra k 2n 2. Ta xy dng nh x f : C B nh sau X = (x1 , x2 , . . . , xk1 , xk , xk+1 , . . . , x2n ) Y = (x1 , x2 , . . . , xk1 , xk , x2n , . . . , xk+1 ) D thy rng f l n nh v f khng phi l ton nh. Tht vy, xt y0 = (x1 , x2 , . . . , xn1 , xn+1 , xn+2 , . . . , n, 2n).

148 Khi khng tn ti x0 f (x0 ) = y0 . Do f khng phi l ton nh. T ta suy iu cn chng minh. Nhn xt. Ta c |C| |B| = m |C| + |B| = |A| = (2n)! nn |B| > |C| < n(2n 1)!. T dn n cch chng minh th 2 sau :
(2n)! 2

= n(2n 1)! v

(ii) Cch 2. Gi Ak l tp tt c hon v tha mn k v k + n ng cnh nhau, A l tp hp tt c cc hon v c tnh cht P . Suy ra A = Ak . Theo nguyn l bao hm loi tr, ta c |A| = Ta c |Ak | = (2n 1)!, |Ak Ah | = 4(2n 1)! Suy ra
k k

|Ak |

k<h

|Ak Ah | +

k<h<m

|Ak Ah Am |

|Ak |

k<h

|Ak Ah |

|Ak | = 2n(2n 1)!,

k<h

|Ak Ah | =

n(n 1) 4(2n 2)! 2

V vy |A| 2n(2n 1)! (2n)! n(n 1) .4(2n 2)! = 2n2 (2n 2)! > 2 2

T ta c iu cn chng minh.

V d 8 (Romania TST 2002). Vi mi s nguyn dng, ta gi f (n) l s cch chn cc du +, trong biu thc: En = 1 2 n sao cho En = 0. Chng minh rng (a) f (n) = 0 khi n 1, 2 (mod 4). (b) Khi n 0, 3 (mod 4) ta c ( 2)
2
n

f (n)

2n 2[ 2 ]+1 .
n

Li gii. (a)Gi s tn ti mt cch t du +, vi n 1, 2 (mod 4) En = 0. Khi 1 + 2 + + n 0 (mod 2), suy ra n(n+1) 0 (mod 2). M iu ny xy ra khi v ch 2 khi n 0, 3 (mod 4).Ta c iu cn chng minh. (b) Ta chng minh f (n) 2n1 . Tht vy, chia tt c biu thc thnh 2n1 cp theo dng: (1 + 2a2 + 3a3 + + nan ; 1 + 2a2 + 3a3 + + nan ) vi ai {1; 1}, i = 1, 2n. Nu f (n) > 2n1 th theo nguyn l Dirichlet tn ti 2 biu thc cng nm trong mt cp nh trn, hiu ca chng bng 2. Do chng khng th ng thi bng 0 c (mu thun). n Do f (n) 2n1 = 2n 2n1 2n 2 2 +1 ( 2) n Ta chng minh f (n) . 2 Xt biu thc En , gi An l tp hp cc s xut hin trong En vi du + trc. Nu En = 0 th tng cc phn t ca An bng n(n+1) 4 T En ta s xy dng En+4 Ta nh ngha An+4 nh sau Nu 1 An th chn An+4 = An \ {1} {n + 2; n + 4};

149 Nu 1 An th chn An+4 = An {1; n + 1; n + 3}; / Nu 2 An th chn An+4 = An \ {2} {n + 3; n + 4}; Nu 2 An th chn An+4 = An {2; n + 1; n + 2}. / Nh vy vi mi cch chn An ta c th xy dng t nht 4 tp An+4. Gi s cho trc tp An+4 . Ta thy tp An+4. c xy dng t tp An v thm ng mt cp trong tp {n + 1; n + 2; n + 3; n + 4} v An cha ng mt cp nh th. Do ta c th ch ra 6 trng hp khi xy dng An+4 . t xc nh c An l duy nht. Do nh x t An n An+4 l n nh. Mt khc vi mi An ta xc nh c duy nht mt En , vi mi En cng xc nh c duy nht mt An . Do f (n + 4) 4f (n). Ta c f (3) = f (4) = 2. Suy ra 4k+3 2 f (4k + 3) = f ((4k 1) + 4) 4f (4k 1) 42 f (4k 5) 4k f (3) = 2 ( 2) V vy f (n) . 2 Vy ta c iu cn chng minh.

V d 9. Cho tp hp A = {1; 2; . . . ; 2n}. Mt tp con C ca A c gi l mt tp tt nu trong tp s cc s chn t hn s cc s l. Mt tp con B ca A c gi l mt tp cn nu trong tp s s chn bng s s l. t s tp tt l T , s tp cn l C. Chng minh rng T + C < 22n1 Li gii. K hiu X = {2; 4; . . . ; 2n} l tp tt c cc s chn ca A; Y = {1; 3; . . . ; 2n 1} l tp tt c cc s l ca A. Gi D l h tt c cc tp cn ca A; E l h tt c tp con n phn t ca A. Tnh s tp cn: Gi s B l mt tp cn ca A. B1 , B2 ln lt l tp cc s chn v l ca B, khi |B1 | = |B2 |. Xt nh x f :DE Ta c B B1 (Y \ B2 )

Suy ra f (B) E. D thy f l n nh. Ta chng minh f l ton nh. Tht vy, gi s M l mt tp con c n phn t ca A. Gi M1 , M2 ln lt l tp cc s chn, cc s l ca M . t B1 = M1 , B2 = Y \ M2 , B = B1 B2 . Suy ra |B1 | = |M1 |, |B2 | = |Y \ M2 | = |Y | |M2 | = n |M2 | = |M1 |. Do B l mt tp cn hay f l ton nh. n V vy f l song nh. T suy ra s tp cn l C2n .

|f (B)| = |B1 (Y \ B2 )| = |B1 | + |Y | |B2 | = |Y | = n

150 Tnh s tp tt: Gi s C l mt tp tt ca A. Gi C1 , C2 ln lt l tp s chn v l ca C th |C| < |C| . Gi F l h tt c tp tt ca A. Ta thit lp nh x sau g:EF C C1 (Y \ C2 ) Suy ra |g(C)| < n. n1 0 1 n Ta c g l mt n nh nn |F | < C2n + C2n + . . . + C2n = 22n1 C2n . T cc ng thc v bt ng thc trn, ta c iu cn chng minh. Nhn xt. Ta c
n C2n =
n (n + 1)(n + 2) 2n (2n)! = > 2 2 (n!)2 n!

T ta c bi ton sau: V d 10. Cho tp A = {1; 2; . . . ; 2n}. Tp con B cu A c gi l tp tt nu trong cc phn n t ca B s s chn nh hn s s l. Chng minh rng s tp tt ca A nh hn 22n1 2 2 . V d 11. Chng minh rng s cc cch biu din mt s nguyn dng n thnh tng ca cc s nguyn dng m khng c s no chia ht cho p (p l mt s cho trc) xut hin qu mt ln khng ln hn s cch biu din n thnh tng cc s nguyn dng m khng c s no chia ht cho p2 . Li gii. Xt b {a1 , a2 , . . . , an } sao cho trong b khng c phn t no chia ht cho p xut hin qu mt ln v
m

ai = n

. Biu din tt c cc s chia ht cho p di dng pr t (r 1; t . .p). r1 rt Gi s {a1 , a2 , . . . , an } = b1 , b2 , . . . , bk , p t1 ; . . . ; p tt Gi M l tp tt c cc cch biu din n thnh tng ca cc s nguyn dng m khng c s no chia ht cho p (p l mt s cho trc) xut hin qu mt ln v N l tp tt c cc s biu din n thnh tng cc s nguyn dng m khng c s no chia ht cho p2 . Xt nh x sau : f :M N {a1 , a2 , . . . , an } b1 , b2 , . . . , bk , pt1 , pt1 , . . . , pt1 , pt2 , pt2 , . . . , pt2 , . . . , ptt , ptt , . . . , ptt r r 1 r 1
p
1 1

i=1

D thy f l mt n nh nn ta c iu cn chng minh.

V d 12. Cho tp X gm n s thc phn bit. Gi s a1 , a2 , . . . , ak l cc s tha mn u = v X th ai sao cho (u ai ) (v ai ) 0. Chng minh rng k log3 n.

151 Li gii. Vi mi u X ta lp b {u1 , u2 , . . . , uk } 0 nu ui = 1 nu 2 nu tng ng tha mn u < ai u = ai u > ai 0 m

Php lp ny l mt n nh. Tht vy, nu tn ti ai tha mn (u ai ) (v ai ) ui = vi , i = 1; k th u, v < ai hoc u = v = ai hoc u, v > ai (khng tha mn). Suy ra c ti a 3k b m |X| = n. Do n 3k hay k log3 n.

2.2. S dng nguyn l Dirichlet


Nguyn l Dirichlet c pht biu ln u tin bi G. Lejeune Dirichlet (1805-1859), nhton hc gc Php nh sau Nguyn l 1 (Nguyn l Dirichlet). Nu nht m con th vo n ci chung (m, n N ) th lun tn ti t nht mt chung c m1 + 1 con th. n Nguyn l 2 (Nguyn l Dirichlet i ngu). Cho tp hu hn S = v S1 , S2 , . . . , Sn l cc tp con Si , i = 1, n ca S sao cho |S1 | + |S2 | + + |Sn | > k|S|. Khi tn ti mt phn t x thuc S sao cho x l phn t chung ca k + 1 tp Si , i = 1, n. V d 13 (Czech 1998). Cho X l mt tp hp gm 14 s nguyn dng phn bit.Chng minh rng c mt s nguyn dng k < 8 v c hai tp con k phn t {a1 , a2 , . . . , ak } v {b1 , b2 , . . . , bk } ri nhau ca X sao cho 1 1 1 + + + a1 a2 ak 1 1 1 + + + b1 b2 bk < 1 1000

Li gii. C 3432 tp con 7 phn t ca X. Tng nghch o cc phn t ca mi tp con 7 phn t ca X khng vt qu 1 + 1 + + 1 < 2 7 2, 6. 1 2599 1 2 Xt 2600 khong 0, 1000 , 1000 , 1000 , . . . , 2600 , 2600 . 1000 Theo nguyn l Dirichlet tn ti hai tp con 7 phn t ca X c tng nghch o cc phn t thuc cng mt khong trong cc khong trn. B cc phn t ging nhau ca hai tp trn ta thu c hai tp con tha mn bi ton. V d 14. Mt tp hp M l hp ca cc on thng nm trong on [0; 1]. Bit rng khong cch gia hai im bt k ca M khc 0, 1. Chng minh rng tng di ca nhng on to nn M khng vt qu 0, 5. Li gii. Ga s tng di cc on trong M ln hn 0, 5. 1 2 9 1 Chia on [0; 1] thnh 10 phn 0, 10 , 10 , 10 , . . . , 10 , 1 . i K hiu Mi l phn ca M nm trong on 10 , i+1 , (i = 0, 9) v Di l tng cc on thng 10 to ra Mi . 1 2 9 1 Bng cc php tnh tin thch hp ta chuyn cc on 10 , 10 ; . . . ; 10 , 1 v on 0; 10 . K hiu Mi l nh ca Mi .

152 Ta c D = D0 + D1 + + D9 > 0, 5 = 5.0, 1 Theo nguyn l Dirichlet i ngu c t nht 6 tp hp trong M0 , M1 , M2 , . . . , M9 c im chung. 1 Tc l mt s no trong 0; 10 l kt qu ca 6 im khc nhau x1 , x2 , . . . , x6 ca M tr i nhng s tng ng c dng k1 , k2 , . . . , k6 vi ki l mt s no trong {0; 1; 2; . . . ; 9} v 10 10 10 i = 1, 6 Theo nguyn l Dirichlet c t nht hai trong cc s k1 , k2 , . . . , k6 l hai s t nhin lin tip. k 1 Ga s k2 = k1 + 1, suy ra x2 x1 = k210 1 = 10 (mu thun vi gi thit). Ta c iu cn chng minh. Nhn xt. Nguyn l Dirichlet cn c ng dng rt quan trng nh sau : Xt on [m; n] v k s thc x1 , x2 , . . . , xk (k > n m) th lun tn ti hai s xi , xj (i, j = 1, k) tha mn |xi xj | = 1. V d 15. Chng minh rng vi mi s nguyn dng n > 1, t n + 2 phn t bt k ca tp {1; 2; . . . ; 3n} c th chn ra hai s c hiu ln hn n v nh hn 2n. Li gii. t A = {1; 2; . . . ; 3n}. Xt B A v |B| = n + 2. Khi thm vo hai s cng mt lng th hiu ca chng khng thay i nn ta c th thm vo cc phn t ca B cng mt lng sao cho phn t ln nht sau khi thm l 3n. Nu tn ti phn t x {n+1, n+2, . . . , 2n1} sau khi thm vo B th, ta c n < 3nx < 2n (tha mn bi ton). Gi s khng tn ti phn t x {n+1, n+2, . . . , 2n1} sau khi thm vo B. Xt C = B\{3n}, khi C l mt tp con n + 1 phn t ca tp {1; 2; . . . ; 3n 1}. Phn hoch tp trn thnh cc tp sau : T1 = {1; 2n}, T2 = {2; 2n + 1}, . . . , Tn = {n; 3n 1}. Theo nhn xt trn ta suy ra tn ti t nht mt tp Ti cha hai phn t ca B, hai phn t ny tha mn bi ton. Nhn xt. Ta c th m rng bi ton trn nh sau : Vi mi s nguyn dng n > 1, k, i N th trong kn + 2 phn t ca tp {1; 2; . . . ; (2k + i)n} lun chn c hai phn t c hiu ln hn kn v nh hn (2k + i)n.

2.3. Phng php ghp cp


V d 16 (Lin X 1965). Trong cuc hi tho c 40 cuc hp, mi cuc hp c 10 thnh vin. Cho bit hai thnh vin bt k ch cng d hp vi nhau ti a mt ln. Chng minh rng cuc hi tho c nhiu hn 60 thnh vin. Li gii. Gi s cuc hi tho c n thnh vin. 2 Do c Cn cch chn hai thnh vin. 2 C C10 cch chn hai thnh vin t mt cuc hp. Do hai thnh vin ch cng d hp vi nhau 2 2 ti a mt ln nn Cn 40 C10 , tng ng vi n(n 1) 3600 hay n > 60. Ta c iu cn chng minh. Nhn xt. Nu gi thit i thnh hai thnh vin bt k cng i d hp vi nhau t nht mt ln th cuc hi tho c nhiu nht 60 thnh vin. Tng qut bi ton ln vi cuc hp c m

153 thnh vin th


2 Cn 2 k Cm

n(n 1) 2

km(m 1) n> 2

km(m 1)

V d 17. Cho 5 s, mi s c 100 ch s 1 hoc 2. Bit trong hai s bt k c r hng bng nhau v trong mi hng c c ch s 1 v 2. Chng minh rng 40 r 60. Li gii. Gi T l s cp m mi cp c hai ch s thuc hai s c tnh cht cng hng v bng nhau.
2 2 C hai s bt k c r hng bng nhau m c C5 cch chn hai s. Suy ra T = rC5 .

Trong mi hng (gm 5 ch s) c c ch s 1 v 2 nn c hai trng hp sau :


2 C mt ch s 1 (hoc 2) v 4 ch s 2 (hoc 1) nn c C4 cp thuc T . 2 2 C hai ch s 1 (hoc 2) v 3 ch s 2 (hoc 1) nn c C2 + C3 cp thuc T .

C tt c 100 hng nn
2 2 100 C3 + C2

T = 10r

2 100C4

T suy ra 40

60.

V d 18. Mt hi ngh ton hc s dng 4 ngn ng chnh. Bit hai i biu bt k lun c mt ngn ng m h u bit. Chng minh rng c mt ngn ng c bit n bi nhiu hn 60% i biu. Li gii. Gi s c tt c n i biu v 4 ngn ng I, II, III, IV. Gi A, B, C, D ln lt l tp cc i biu bit ngn ng I, II, III, IV. Nu tn ti mt ngi ch bit duy nht mt ngn ng th n 1 ngi cn li cng phi bit ngn ng . Do ngn ng c bit bi 100% i biu.
2 2 2 2 2 Nu tt c i biu u bit c t nht hai th ting th C|A| + C|B| + C|C| + C|D| 2Cn . Gi s A l tp tha mn |A| = max {|A| , |B| , |C| , |D|}. 2 2 Suy ra Cn C|A| hay tng ng vi 2|A|(|A| 1) n.(n 1). 2 3 6 Gi s |A| < 3 n th 2|A|(|A| 1) < 6 n 5 n 1 = 18 n2 5 n < n2 n (mu thun) 5 5 25 Do A 3 n. 5

Vy trong mi trng hp, ta c iu cn chng minh. V d 19 (IMO 1988). Cho n, k l cc s nguyn dng, n mt phng tha mn (i) Khng c ba im no thng hng. (ii) Vi mi im P ca h u khng c t hn k im trong h cch u P . Chng minh rng k 1 + 2n 2

k v S l tp hp n im trong

154 Li gii. Gi s k > 1 + 2n. 2 2 Ly im P thuc S th tn ti t nht k im trong S cch u P . Suy ra tn ti t nht Ck cp im (A, B) m P A = P B. 2 C t nht nCk cp im m trn ng trung trc ca on thng m hai u mt l hai im c t nht mt im thuc S. Ta c
2 nCk = n

1 k(k 1) > n 2 2

1 + 2n 2

2n

1 2

=n n

1 8

2 > n(n 1) = 2Cn

2 2 Li c Cn l s cp im khng th t ca S, 2Cn l s cp im c th t ca S. Theo nguyn l Dirichlet tn ti mt cp im A, B v ba im P1 , P2 , P3 tha mn APi = BPi i = 1; 3 Suy ra P1 , P2 , P3 thng hng (v l). Vy ta c iu cn chng minh.

Nhn xt. C th m rng bi ton trn trong khng gian nh sau : V d 20. Cho n, k l cc s nguyn dng, n tha mn (i) Khng c 65 im no thng hng. (ii) Mi im P ca h u khng c t hn k im trong h cch u P . Chng minh rng k 1 + 4 3 (n 1)(n 2) Li gii. Gi A l tp cc b im (M, N, P ) tha mn M, N, P l ba im ca h v tn ti t nht mt im ca h cch u ba im (im ny c gi l tm ca h). Vi mi im P ca h c t nht k im thuc h cch u P . Suy ra s b thuc A 2 nhn P lm tm ca b ln hn hoc bng Cn . 3 C n im c tnh nn s cp khng nh hn n Ck .
3 C n im nn c Cn b. Theo (i), mi b c khng qu 64 ln tnh. S cp c tnh 3 khng ln hn 64Cn .

k v S l tp hp n im trong khng gian

T suy ra
3 n Ck 3 64 Cn

Tng ng vi nk(k 1)(k 2) k(k 1)(k 2) 64n(n 1)(n 2) 64(n 1)(n 2)

(k 1)3 < 64(n 1)(n 2) k < 1 + 4 3 (n 1)(n 2) Bt ng thc cui cho chng ta iu cn chng minh. Nhn xt. C th tng qut gi thit (i) thnh khng c qu m im no thng hng. Khi k < 1 + 3 m(n 1)(n 2).

155 V d 21 (IMO 1998). Trong mt cuc thi c m th sinh v n gim kho, n l s nguyn dng l v n > 2. Mi gim kho nh gi th sinh theo hai mc , trt. Gi k l mt s sao cho ly hai gim kho ty th hai ngi c chung nh gi k th sinh. Chng minh rng k m n1 2n

Li gii. Gi N l b ba (hai gim kho, mt th sinh) tha mn hai gim kho trong nhm c cng nh gi ( hoc trt) cho th sinh trong b . 2 C Cn cch chn b hai gim kho. M k l s nguyn dng tha mn nu ly hai gim kho bt k th hai gim kho ny nh gi trng nhiu nht k th sinh nn N
2 kCn

(5)

Xt th sinh A bt k. Gi s c x gim kho nh gi A u.


2 2 C Cx cp gim kho nh gi A u, Cnx cp gim kho nh gi A trt. 2 2 C tt c Cnx + Cx cp gim kho c cng nh gi i vi A.

Ta c:

2 2 Cnx + Cx =

x(x 1) + (n x)(n x 1) 2 2 2 n n n = x + 2 4 2 (n 1)2 1 n2 n = 4 2 4 4 cp gim kho c cng nh gi i vi A. Suy ra N m (n 1)2 4


k m n1 . 2n

(n1)2 4

N nn c t nht

(n1)2 4

(6)

T (1) v (2) ta c k

n(n1) 2

(n1)2 4

Suy ra

2.4 Mt s bi ton khc


V d 22. Cho n l s t nhin ln hn 1. Gi S l tp hp gm n phn t v Ai , i = 1, m l m tp con i mt khc nhau ca S tha mn |A | 2; i i, j, k : Nu |Ai Aj | = , |Aj Ak | = , |Ak Ai | = th |Ai Aj Ak | = . 2n1 1.

Chng minh rng m

Li gii. Ta chng minh quy np theo n. Vi n = 2, 3 th kt lun bi ton ng. Vi n 4. Gi s bi ton ng vi n 1. Ta cn chng minh vi n th bi ton ng ng. Tht vy, gi T l h gm cc tp Ai . Xt hai trng hp sau :

156 Trng hp 1. Tn ti tp A m A v S A u thuc T . Chia T thnh 3 h gm: T1 l h cc tp con ca A, T2 l h cc tp con ca S \ A, T3 = T \ (T1 T2 ). Ta c nhn xt sau: Vi mi tp B T3 th |B A| > 0, |B (S \ A)| > 0. Do vi mi B, C T3 th |B C| = 0. Tht vy, nu |B C| = 0 th |A B C| > 0 v | (S \ A) B C| > 0 (theo nhn xt trn), v l. Ta c |T3 | min{|A|, |S \ A|}. Suy ra |T | = |T1 | + |T2 | + |T3 | 2|A| 1 + 2n|A|+1 + min{|A|, n |A|} 2n1 1

Trng hp 2. Vi A T th S \ A T . / n n1 V S c 2 tp con nn |T | 2 . Gi s |T | 2n1 . Suy ra |T | = 2n1 . Gi s X = {1; 2; . . . ; n 1} T . Chia T thnh 2 h nh sau: T4 l h cc tp con ca X v T5 l h cc tp cha phn t n. Ta c nhn xt sau : Bi , Bj T5 th |Bi Bj | 1. Tht vy, nu |Bi Bj | < 1 th |Bi Bj X| = 0 m |Bi Bj | > 0; |Bi X| > 0; |X Bj | > 0 (Mu thun) Xt T5 = {B \ {n}|B T5 }, mi tp T5 l 1 tp con ca {1; 2; . . . ; n 1} v mi tp C, D con ca T5 th |C D| 1 |T5 | = |T5 | 2n2 Theo gi thit quy np th |T4 | 2n2 1 |T | = |T4 | + |T5 | 2n2 1 + 2n2 = 2n1 1 (Mu thun vi gt). Kt hp c hai trng hp ta c iu cn chng minh. V d 23. Cho hm f xc nh trn cc tp con ca tp hu hn S (S = ). Chng minh rng nu f (S \ A) = f (A) v max {f (A) ; f (B)} f (A B) vi mi tp con A, B ca S th f nhn khng qu |S| gi tr phn bit. Li gii. Ta s chng minh bng quy np theo s phn t ca S. Vi |S| = 1 v |S| = 2 s cc cp gm hai tp con b nhau ca S tng ng bng 1 v 2. Kt lun bi ton ng. Vi |S| > 2. Gi s bi ton ng cho s phn t b hn. Gi m l phn t ln nht ca Imf . Trc ht ta chng minh S c mt tp con c 1 phn t X = {x} tha mn f (x) = m. Tht vy, xt tp tt c tp con nhn gi tr m qua nh x f . V f (S \ A) = f (A) nn trong cc tp con ny c t nht mt tp con khc rng. Gi X l tp con b nht tha mn f (X) = m. Nu X c t nht 2 phn t

157 th X c th vit thnh hp ca hai tp phn bit : X = X1 X2 . Suy ra max {f (X1 ) ; f (X2 )} f (X1 X2 ) = f (X) = m M m l phn t ln nht trong tp gi tr ca f nn trong hai phn t f (X1 ) ; f (X2 ) c t nht mt phn t bng m (mu thun). Do S c mt tp con c 1 phn t X = {x} tha mn f (X) = m.

t R = S\X. Xt hm g : R f (R) nh sau : Vi A R th

g (A) = min {f (A) ; f (A X)} = min {f (A) ; f (R \ A)} Vi A R th m = f (X) = f (S \ X) = f (A (R \ A)) = max {f (A) ; f (A X)} max {f (A) ; f (R \ A)} = max {f (A) ; f (S \ (R \ A))}

Ta chng minh bng quy np gi thit cng ng vi hm g, tc l g (R \ A) = g (A) v g (A B) max {g (A) ; g (B)} Tht vy, g (R \ A) = g (A) ng theo cch xc nh hm g. Nu g(A) = m hoc g(B) = m th g (A B) max {g (A) ; g (B)} (theo nh ngha s m). Nu g(A), g(B) < m th mt trong hai gi tr f (A) ; f (A X) < m v mt trong hai gi tr f (B) ; f (B X) < m. Nu f (A) < m t C = A, nu f (A X) < m t C = A X. Nu f (B) < m t D = B, nu f (B X) < m t D = B X. Suy ra f (C) = g (A) ; f (D) = g (B) v tp C D hoc bng A B hoc bng A B X. T ta c max {g (A) ; g (B)} = max {f (C) ; f (D)} = g (A B) Vy g tha mn gi thit. f (C D) min {f (A B) ; f (A B X)}

Theo gi thit quy np th g ch c th nhn |S| 1 gi tr khc nhau. Vy A R, mt trong hai gi tr f (A) v f (R \ A)c mt gi tr bng m v gi tr cn li bng g(A). Do hm f ch c th nhn nhiu hn hm g mt gi tr. T ta c iu cn chng minh. V d 24 (THTT 395). Cho X l tp con ca {1; 2; . . . ; 2011} tha mn (i) |X| = 62; (ii) Vi x X u a, b X {0; 2011} (a; b = x) tha mn x =
a+b . 2

158 Chng minh rng c hai phn t x, y ca X tha mn |x y| 11, x+y X. / 2

Li gii. Gi s X l tp c 62 phn t c sp th t x1 x2 x62 . Ta s chng minh bi ton ny bng phn chng : Gi s khng tn ti hai phn t x, y ca / X tha mn |x y| 11, x+y X. 2 Ta chng minh cc phn t ca X cng tnh chn l. Tht vy : X {1; 2; . . . ; 2011}. Suy ra xi+1 xi + 1, i = 1, 61. Do x62 xk + 11, k = 1, 51. Suy ra xk (k = 1, 51) cng tnh chn l vi x62 , nu khng th xk +x62 X (mu thun) / 2 Tng t ta c xl (l = 12, 62) cng tnh chn l vi x1 . V vy xi i = 1, 62 cng tnh chn l. T gi thit suy ra a; b X {0; 2011} tha mn x62 = a+b . 2 Do x62 > x1 nn xp (p = 1; 2; . . . ; 61) tha mn x62 = xp +2011 hay xp l. 2 +0 Tng t do xi > x1 (i = 1; 62) nn xq (q = 2; 3; . . . ; 61) tha mn x1 = xq2 chn (mu thun). Vy ta c iu cn chng minh.

3. Mt s bi ton tm cc tr t hp
Cc bi ton ny thng c chng minh theo cc bc sau : chng minh min k = hay max k = ta s chng minh k (hoc k ) v ch xy ra mt kh nng c du bng hoc l ch ra k = tha mn ri chng minh k (hoc k ) th khng ng. Ci kh nht ca nhng bi ton dng ny chnh l tm c (thng l khng t nhin). Thc t ton t hp c rt nhiu tng gii nn khng c iu g tht s hu hiu ngoi kinh nghim v cm gic c c nh s luyn tp mt cch thng minh ca chng ta. Cc bi ton ny thng ri vo mt trong hai dng sau : Dng 1 : Tm s nguyn dng n ln nht (b nht) tn ti tp hp gm n phn t tha mn iu kin (*). Dng 2 : Tm s nguyn dng n ln nht (b nht) mi tp hp gm n phn t tha mn iu kin (*). Cch gii hai dng ton ny c trnh by trn. Nhn xt : Trong nhng bi ton c ni dung ti u m cc phn t c quan h chia ht hoc nguyn t cng nhau, ta thng quan tm n nhng iu sau : Xt theo modulo 2 hoc ly tha ca 2. Nu vic phn tch trn khng thnh cng th chng ta phi quan tm n cc s nguyn t v phn tch chnh tc ca cc s t nhin. V d 25. Tm s nguyn dng k nh nht sao cho ta c th t mu cc s nguyn dng t 1 n 2011 bng k mu khc nhau khng c hai s no cng mu l bi ca nhau.

159 Li gii. Vi bi ny ta s dng ly tha ca 2. V cc s 20 ; 21 ; 22 ; . . . ; 210 khng th c hai s no cng t mt mu nn k 11. i Vi k = 11 ta c cch t mu nh sau : t Ai = 2i , 2 + 1; . . . ; 2i+1 1 i = 0; 9 v A10 = {210 , 210 + 1; . . . ; 2011}. T mu cc s thuc Ai bi mu th i + 1. Cch t mu ny tha mn bi ton. Vy kmin = 11. Nhn xt : Ta c th thay 2011 bi mt s nguyn n dng bt k, khi kmin = [log2 n] + 1. V d 26. Cho S l tp hp gm 2011 s nguyn dng v chn ra n tp con ca S sao cho tng cc phn t trong n tp con i mt nguyn t cng nhau. Tm gi tr ln nht c th c ca n. Li gii. p s l 22010 + 1. Trc ht ta chng minh n 22010 + 1. Trong 22011 tp con ca S th nhiu nht trong mt na trong s c tng cc phn t l s l. Tht vy, nu tt cc phn t ca S u l s chn th tt c cc tp con ca S u c tng cc phn t u chn nn khng c tp con no trong S c tng cc phn t l s l. Nu tn ti mt phn t x ca S l s l th ta c th chia 22011 tp con ca S thnh 22010 cp (T ; T {x}) vi T l tp con ca S m x T . Trong mi cp c mt / tp c tng cc phn t ca n l l, tp cn li c tng cc phn t l chn. Do c ti a 22010 cc tp con ca S c tng cc phn t l s l. V vy nu n > 22010 + 1 th c t nht hai tp c tng cc phn t u l s chn (tri vi gi thit). Vyn 22010 + 1. Ta s xy dng mt v d cho n = 22010 + 1. t k = (22010 )! v tp S = {k + 1; 2k + 1; 4k + 1; . . . ; 22009 k + 1}. Xt 22010 tp con ca S v thm mt tp {k}. Cc tp con ca S cha phn t 1 th c tng cc phn t c dng ak +1 vi 0 a 22010 v hai tp khc nhau th c tng cc phn t khc nhau. Ta c gcd(k; ak + 1) = 1. Nu gcd(ak + 1; bk + 1) = p (a = b) th p|k(a b). Suy ra p|a b hoc p|k, m k > |a b| > 0 nn p|k hay p = 1. Do tng cc phn t ca cc tp trn nguyn t cng nhau. Vy max n = 22010 + 1.

Nhn xt : Cng ging nh bi ton trn ta c th thay 2011 bi mt s nguyn dng k bt k, cch gii hon ton tng t, khi max n = 2k1 + 1. V d 27 (Ba Lan 2001, Trung Quc 2001). Cho tp X = {1; 2; . . . ; 2001}. Tm s n b nht sao cho nu A l tp con gm n phn t ca X th ta lun c th tm c mt phn t l ly tha ca 2 hoc hai phn t c tng l ly tha ca 2. Li gii. p s l 999. Gi s A l mt tp con ca X khng cha bt k mt ly tha no ca 2 v khng c 2 phn t no c tng l ly tha ca 2. Do A khng cha cc phn t 1; 2; 22 ; . . . ; 210 v t nht mt phn t trong 992 cp sau y cng b loi:

160 17 cp (47; 2001); (48; 2000); . . . ; (63; 1985) 63 cp (65; 1983); (66; 1982); . . . ; (127; 1921) 127 cp (129; 1919); (130; 1918); . . . ; (255; 1793) 255 cp (257; 1981); (258; 1980); . . . ; (511; 1537) 511 cp (513; 1535); (514; 1534); . . . ; (1023; 1025) (Cc cp ny c tng l 211 ) 14 cp (18; 46); (19; 45); . . . ; (31; 33) (Cc cp ny c tng l 26 ) 1 cp (15; 17) (Cp ny c tng l 25 ) 4 cp (3; 13); . . . ; (7; 19) (Cc cp ny c tng l 24 ) Vy A ch cha nhiu nht 998 phn t. Xt tp c 998 phn t sau: 977 phn t: 1025; 1026; . . . ; 2001 (nhm 1) 14 phn t: 33; 34; . . . ; 46 (nhm 2) 7 phn t: 9, 10, 11, 12, 13, 14, 17 (nhm 3) D thy tp trn khng c phn t no l ly tha ca 2. Tng 2 phn bt k trong nhm 1 nm gia 2 1025 = 2050 v 2 2001 = 4002 Tng 2 phn bt k trong nhm 2 nm gia 2 33 = 66 v 2 46 = 92 Tng 2 phn bt k trong nhm 3 nm gia 2 9 = 18 v 2 17 = 34 Tng 2 phn bt k trong nhm 1 v nhm 2 nm gia 33 + 1025 = 1058 v 46 + 2001 = 2047 Tng 2 phn bt k trong nhm 2 v nhm 3 nm gia 33 + 9 = 42 v 46 + 17 = 63 Tng 2 phn bt k trong nhm 1v nhm 3 nm gia 1025 + 9 = 1034 v 2001 + 17 = 2018 Vy trong cc tp con ca S khng c tp no cha hai phn t c tng l mt ly tha ca 2. Do min n = 999. V d 28 (IMO 1991). Xt tp S = {1; 2; . . . ; 280}. Tm s nguyn dng n nh nht sao cho n s bt k ly ra t S c 5 s nguyn t cng nhau. Li gii. p s l 217. Xt tp hp A = {k S | k |A| = chia ht cho 2,3,5 hoc 7}. Ta c

280 280 280 280 280 280 280 280 + + + 2 3 5 7 6 10 14 15 280 280 280 280 280 280 280 + + + + 21 35 30 42 70 105 210

= 216 v trong 5 phn t bt k ca A lun c hai s c c chung ln hn 1. Do n > 216. Xt tp con T S; |T | = 217. Ta s chng minh T cha 5 phn t nguyn t vi nhau tng i mt. t B1 = A \ {2; 3; 5; 7} B2 = 112 , 11 13; 11 17; 11 19; 11 23; 13 17; 13 19 P = S \ (B1 B2 )

161 Khi ta c |P | = |S| |B1 | |B2 | = 60 v P l hp ca 1 v tt c cc s nguyn t trong S. Nu |T P | 5 th ta c iu cn chng minh. Nu |T P | < 5, ta c |T (S \ P )| 217 4 = 213. Do tp hp cc s nguyn t trong S c nhiu nht 280 213 60 = 7 s khng thuc T . Xt cc tp hp M1 = {2 23; 3 19; 5 17; 7 13; 11 17} M2 = {2 29; 3 23; 5 19; 7 17; 11 13} M3 = {2 31; 3 29; 5 23; 7 19; 11 17} M4 = {2 37; 3 31; 5 23; 7 19; 11 17} M5 = {2 41; 3 37; 5 29; 7 29; 11 23} M6 = {2 43; 3 41; 5 37; 7 31; 13 17} M7 = {2 47; 3 43; 5 41; 7 37; 13 19} M8 = {22 ; 32 ; 52 ; 72 ; 132 } Ta c Mi S \ P v cc phn t ca Mi nguyn t cng nhau. Tn ti j Mj T . Vy min n = 217.

V d 29. Tm s nguyn dng k nh nht sao cho mi tp con k phn t ca tp {1; 2; . . . ; 50} u c cha 2 phn t phn bit a v b sao cho a2 + b2 l s chnh phng. Phn tch. Gi (a, b) l mt cp Pythagore nu a2 + b2 l s chnh phng. Biu din s trong tp {1; 2; . . . ; 50} trn mt graph, cc s trong mt cp Pythagore th c ni vi nhau bng mt on thng, cc s khng cng nm trong mt cp Pythagore no th khng c ni vi nhau. u tin ta xy dng tp B gm t phn t nht sao cho trong tp C = S \ B khng c b Pythagore no. Mun vy trn graph B phi l cc nh sao cho khi xa cc nh ny v tt c cc nh c ni t n th graph khng cn cnh no na. Ta chn cc im thuc B nh sau: (3;4); (33;44) l 2 on thng ring r nn ta chn B {3; 33} (hoc B {3; 44} hocB {20; 28} hoc B {4; 44} u c) v xa cc on i. Cc on thng c mt u l nh n th nh kia (khng n) phi thuc B nn B {8; 12; 24; 36; 40; 48}. Xa chng v cc cnh ni vi chng. Cp (16; 30) ring l nn ta chn B {16} (hoc B {30}). Cui cng B {20, 28}. Lc ny graph b xa ht. Vy ta c B = {3; 33; 12; 24; 8; 36; 40; 48; 16; 20; 28}. Tip theo ta s ch ra 11 b Pythagore ri nhau, trong graph cc cp ny chnh l cc on thng ri nhau. Mun chn 11 cp ta phi chn c nhiu on nht. Trc ht ta chn 2 cp (3; 4) v (33; 44). Tip n ta chn cc cp sau (5; 12); (7; 24); (6; 8); (27; 36); (40; 42); (48; 14); (16; 30) v chn hai trong 4 cp (42; 58); (28; 21); (21; 20); (20; 15). Nh vy 11 cp Pythagore ri nhau c xc nh. Li gii. Trc ht ta lit k tt c cc cp Pythagore: (3; 4); (6; 8); (9; 12); (12; 16); (15; 20); (18; 24); (21; 28); (24; 32); (27; 36); (30; 40); (33; 44); (36; 48); (8; 15); (16; 30); (24; 45); (12; 35); (5; 12);

162 (10; 24); (15; 36); (20; 48); (20; 21); (40; 42); (28; 45); (7; 24); (14; 48); (9; 40) Xt B = {3; 8; 12; 16; 20; 24; 28; 33; 36; 40; 48}. D thy mi cp Pythagore u c t nht mt phn t thuc tp B nn C = S \ B l tp khng cha bt k 1 cp Pythagore no. Vy k > |C| = 39. Xt 11 cp s Pythagore ri nhau sau: (3; 4); (33; 44); (5; 12); (7; 24); (6; 8); (27; 36); (40; 42); (48; 14); (16; 30); (45; 28); (21; 20).V mi tp con cha k phn t ca S phi cha t nht mt cp Pythagore trn nn min k = 40.Vy min k = 40. Nhn xt : T bi ton ny ta ngh n mt bi ton khc c tng tng t nhng vic tm tp B v cc b ri nhau s kh khn hn. V d 30. Tm s nguyn dng k nh nht sao cho trong k phn t ty ca tp {1; 2; . . . ; 50} lun cha 3 s l di 3 cnh ca mt tam gic vung. Mt b 3 s thuc tp {1; 2; . . . ; 50} c gi l b 3 s Pythagore nu 3 s l di 3 cnh tam gic vung. C 15 b 3 s Pythagore sau: {3; 4; 5}, {6; 8; 10}, {9; 12; 15}, {12; 16; 20}, {15; 20; 25}, {18; 24; 30}, {21; 28; 35}, {24; 32; 40}, {27; 36; 45}, {8; 15; 17} {16; 30; 34}, {5; 12; 13}, {10; 24; 26}, {15; 36; 39}, {30; 40; 50} Xt tp B = {5; 8; 9; 16; 20; 24; 35; 36; 50}. D thy mi b 3 s Pythagore u c t nht mt s thuc tp B nn C = A \ B khng cha b 3 s Pythagore no. Do k > |C| = 41. Xt 9 b 3 s Pythagore ri nhau sau: {3; 4; 5}, {6; 8; 10}, {12; 35; 37}, {27; 36; 45}, {9; 40; 41} {20; 21; 29}, {14; 48; 50}, {16; 30; 34}, {7; 24; 25} Bt c tp con 42 phn t no ca A cng phi cha t nht mt b 3 s Pythagore trong 9 b trn. Vy min k = 42. V d 31. Cho X = {1; 2; . . . ; 15}. M l tp con ca X sao cho tch ca 3 phn t khc nhau ca M u khng l s chnh phng. Tm max |M |. Li gii. Gi b 3 s thuc X c tch l s chnh phng l mt b tt. Ta chng minh vi mi tp con T ca X m |T | = 11 th trong T lun tn ti mt b tt. Tht vy: Cc b tt gm: Loi 1 (cha 2): (2; 1; 8); (2; 3; 6). . . Loi 2 (cha 3, khng cha 2) Loi 3 (cha 8, khng cha 2, 3) Loi 4 (cha 15, khng cha 2, 3, 8) Loi 5 (cc b cn li) Trng hp xu nht T khng cha b tt no l 4 s khng thuc T phi tham gia vo nhiu b tt nht. trng hp xu nht xy ra, ta chn 4 s khng thuc T l 2; 3; 4; 15 nhng vn cn mt

163 tp loi 5. Do |M | < 11. Khi |M | = 10 ta c tp sau : M = {1; 5; 6; 7; 9; 10; 11; 13; 14; 15}. Vy max |M | = 10. V d 32 (THTT, Hello IMO 2007). Mt lp c 15 hc sinh nam v 15 hc sinh n. Vo ngy 8/3 c mt s nam sinh gi in n chc mng mt s n sinh. Bit rng ta c th phn chia tt c hc sinh ca lp mt cch duy nht thnh 15 cp sao cho mi cp u gm mt n sinh v mt nam sinh m nam sinh gi in thoi chc mng. Hi c ti a bao nhiu cuc in thoi c gi? Li gii. Gi s 15 bn nam l a1 , a2 , . . . , a15 v 15 bn n l b1 , b2 , . . . , b15 , cch ghp cp duy nht l (ai ; bi ), (1 i 15). Vi mi cuc gi (ai ; bj ), (i = j; 1 i, j) ta cho ng vi mt tp con gm hai phn t {i; j}. D thy ng vi mi tp con hai phn t {i; j} ny c hai cuc gi (ai ; bj ) v (aj ; bi ) th ta c cch ghp cp th 2 {(ak ; bk ) | k = i, k = j} {(ai ; bj ); (aj ; bi )} (mu thun). 2 Do s cuc gi khng vt qu 15 + C15 = 120. Cch gi tho mn iu kin bi ton l ngi ai ch gi cho ng i ngi b1 , b2 , . . . , bi . Vy c ti a 120 cuc gi.

Ti liu tham kho


[1] Nguyn Vn Nho, Tuyn chn cc bi ton t nhng cuc thi thi ti mt s nc ng u - 2 tp, NXB Gio dc, 2005. [2] V Dng Thy, Nguyn Vn Nho, 40 nm Olympic ton hc quc t, NXB Gio dc, 2003. [3] Nguyn Vn Nho, Tuyn chn cc bi ton t mt s cuc thi ti Trung Quc, NXB Gio dc, 2002. [4] Nguyn Vn Mu (ch bin), Chuyn chn lc T hp v ri rc, NXB Gio dc, 2008. [5] Phan Huy Khi, Cc bi ton hnh hc t hp, NXB Gio dc, 2006. [6] Mt s IMO Shortlist, VMO, Vit Nam TST. [7] Tp ch Ton hc v Tui tr. [8] Tp san T hc ton ca hc sinh chuyn ton trng THPT chuyn Phan Bi Chu, Ngh An cc kha K29, K32, K33. [9] Ti liu tp hun gio vin chuyn ton nm 2011.

164

M TS

BI TON T H P I N HNH V BN C
Nguyn Vit Dng1

T hp bn c bao gm cc bi ton hay v kh. Bi vit ny thng qua mt s bi ton in hnh nhm gip cho bn c c nhn tng quan v phng hng gii quyt khi gp cc bi ton bn c dng ny. Bi ton 1. (a) Tm s ln nht cc qun xe c th t trn bn c n n sao cho khng c hai qun no tn cng ln nhau? V c bao nhiu cch t tha mn vi s lng qun xe ? (b) Tm s nh nht cc qun xe c th sp xp trn mt bn c n n sao cho bt k vung no trn bn c u b khng ch bi t nht mt qun xe trong s chng? C bao nhiu cch xp tha mn vi s lng qun xe ? Li gii. (a) Do khng c qun xe no trn bn c tn cng qun khc nn iu kin cn v l khng c hai qun no nm trn cng hng hoc cng ct. Do tng s cc qun xe khng vt qu n. Ta ch ra cch t n qun tha mn iu kin bi ton l t n qun xe trn ng cho chnh ca bn c. Chng ta s xc nh s cch sp xp n qun xe tha mn iu kin bi ton. Trc ht chng ta gi qun xe ct u tin l qun xe u tin, qun xe ct th 2 l qun xe th 2,. . . v qun xe ct th n l qun xe th n. Qun xe u tin c th di chuyn trong n . Vi bt k cch t qun xe u tin hng no th cn li n 1 hng cc qun xe cn li c th di chuyn. Hay qun xe th 2 c th di chuyn trong n 1 hng. V cc qun xe cn li c kh nng di chuyn trong n 2 hng. . . Cui cng ch c duy nht mt hng cho qun xe th n c th t. Theo quy tc nhn, s cch t n qun xe tha mn bi ra l n(n 1)(n 2) 1 = n! cch t khc nhau. (b) R rng khng th dng t hn n qun xe t ln bn c n n tha mn bi ra v tri li dng t hn n qun th s c mt ct v mt hng khng cha qun xe no, khi giao nhau ca ct v hng khng b kim sot bi bt k qun xe no. Mt khc, r rng c th t n qun xe trn bn c tha mn bi ton nn n l s nh nht tha mn bi ra. Nu n qun xe trn bn c n n kim sot mi vung trn bn c th c hoc mt qun xe trn mi ct hoc mt qun xe trn mi hng. Ngc li nu c mt qun xe trn mi hng hoc mt qun xe trn mi ct th chng s kim sot c bn c. S cc cch sp xp n qun xe trn mi ct l nn (mi qun c th t trn n hng). Tng t c nn cch sp xp cc qun
1

Lp 12A1, THPT chuyn HKHTN - HQGHN.

165

166 xe mi hng cha mt qun xe. Mt khc hai cch xp trn b trng nhau n! cch (xem cu a) nn p s l 2nn n!. Bi ton 2. (a) Tm s ln nht cc qun tng c th sp xp trn bn c 8 8 sao cho khng c qun no tn cng qun khc? Gii bi ton vi bn c n n? (b) Tm s nh nht cc qun tng c th sp xp trn mt bn c 8 8 sao cho bt k vung no trn bn c u b khng ch bi t nht mt qun tng trong s chng? Gii bi ton vi bn c n n? Li gii. (a) Xt cc ng cho i t gc di bn tri ln gc trn bn phi ( gn hn ta gi cc ng cho ny l cc ng cho dng). C tt c 15 ng cho nh vy (hnh minh ha). Nu khng c hai qun tng no tn cng ln nhau th khng th c hn mt qun cng nm trn mt ng cho. Do tng s qun tng khng th ln hn 15. Nhng khng th t tt c 15 qun trn bn c v qun u tin v qun cui cng cng nm trn mt hng. Do , nhiu nht 14 qun tng c th xp trn bn c tha mn. Ta ch ra cch xp tha mn trong trng hp 14 qun tng nh hnh sau

Tng qut cho bn c n n, vi cch lp lun tng t ta nhn c s ln nht cc qun tng c th t l 2n 2. (b) Chng ta s chng t rng cn t nht 8 qun tng sp xp trn bn c 8 8 tha mn. lm iu ny chng ta t mu xen k cc bn c v chng t phi c t nht 4 qun tng trn mi mu. Quy c qun tng chim gi en l qun tng en v qun tng chim gi trng l qun tng trng. Nu cc hnh vung en c quay mt gc 45 theo chiu ngc chiu kim ng h nh hnh sau

167

thun tin, ta thay i thnh hnh sau

Khi cc qun tng c th di chuyn ngang v dc nh cc qun xe. By gi chng ta thy rng cn t nht 4 qun xe t vo cc hnh vung trong hnh trn v n cha hnh vung 4 4 (in m). Do chng ta cn t nht 4 qun tng en, v tng t cn t nht 4 qun tng trng. Mt khc, chng ta ch ra cch xp tha mn cho 8 qun tng nh trong hnh sau

Gii bi ton vi bn c n n. Tng qut hn, vi n = 2k th chng ta c th chng minh rng cn t nht n qun tng t vo bn c n n. Bng cch quay mt gc 45 cc vung mu en v chuyn cc qun tng thnh cc qun xe, khi s c mt vung k k cha trong hnh mi. T cn t nht k qun tng en, v tng t cn t nht k qun tng trng. Tng s qun

168 tng t nht l k + k = n. Mt khc n qun tng xp trn bn c tha mn bi ra v chng ta c th t trn cng mt ct. Khi mi vung b khng ch bi t nht mt qun tng. Trong trng hp n l, v tr cc qun c c t khc mt cht v s en khc vi s trng. Tuy nhin ta vn c th gii tng t nh trong trng hp bn c 8 8. Mt cch t tha mn l t cc qun c trn cng mt ct ca bn c. Tng t nh trn, quay bn c mt gc 45 , ta thu c hai hnh tng ng vi cc mu en v cc mu trng. Trong hnh tng ng vi cc en cha hnh vung 4 4 v hnh tng ng vi cc trng cha hnh vung 5 5. Do chng ta cn t nht 4 + 5 = 9 qun tng t ln bn c. Vi s qun ny c cch xp tha mn nh trn. Tng qut, nu n = 2k + 1 l s l bt k th vi cch chng minh tng t th c tng s k + (k + 1) = n qun tng cn dng, bi v c th bin i cc trng thnh hnh cha bng (k + 1) (k + 1) (do trng nm gc nn s nhiu hn en) v hnh cha bng k k en. Ta c th ch ra trng hp nh vy bng cch t cc qun tng vo ng cho chnh gia. Bi ton 3. Chng minh rng vi n chn, cc s sau l cc s chnh phng (a) S cc cch sp xp khc nhau cc qun tng trn bn c n n khng c qun tng no tn cng qun khc v s ln nht cc qun tng c th c s dng. (b) S cc cch sp xp khc nhau cc qun tng trn bn c n n mi vung u b kim sot bi t nht mt qun tng v s nh nht cc qun tng c s dng. Li gii. (a) Bi v cc qun tng ch kim sot hoc cc mu trng hoc cc mu en. Bi ton v vic xy dng cch xp ln nht cc qun tng khng c hai qun no tn cng ln nhau c th phn hoch bn c thnh hai phn khng ph thuc vo nhau; xy dng cch xp nhiu nht cc qun tng trng (en) khng c hai qun no tn cng ln nhau. Theo gi thit th n chn nn cc en trn bn c c th chuyn thnh cc trng bi php quay 90 . Do s cc qun tng trng trong cch xp ln nht bng s qun tng en trong cch xp ln nht v chng tng ng nhau v cch xp. Bi v tng s qun tng l 2n 2 (theo bi 2) nn mi mu c n 1 qun. Chng ta nhn c s cch xp tha mn ca 2n 2 qun tng trn bng bi s cch xp tha mn ca n 1 qun tng en v cch xp tha mn ca n 1 qun tng trng. Theo quy tc nhn suy ra tng s cch xp tha mn l bnh phng s cch xp ca n 1 qun tng mi mu. (b) Li gii tng t phn a. Bi ton 4. Chng minh rng trong cch sp xp cc qun tng tha mn gi thit bi 3a, tt c cc qun tng u nm cc hng ngoi cng v cc ct ngoi cng ca bn c. Li gii. Trc ht theo bi 3a ta c cch sp xp 2n 2 qun tng trn bng n n m khng c 2 qun no tn cng ln nhau. Trn mi vung ca bng ta vit s cc qun tng kim sot n. Mt vung b kim sot bi mt qun tng c nh du bi s 1 (quy c mi qun tng kim sot chnh n). Khng c hnh vung no c nh du bi 0, bi v nu c hnh vung nh th chng ta c

169 th t mt qun tng mi vo n m khng tn cng qun khc, iu ny mu thun vi tnh ln nht ca cch xp cho. Cc vung gc c nh du bi 1 bi v ch c mt ng cho i qua nn ch c mt qun tng kim sot n. Cc khng gc c nh du bi hoc 1 hoc 2, bi v mi ch c hai ng cho i qua v c nhiu nht mt qun tng trn mi ng cho ny. Trn 4 gc ca bn c, t nht hai khng c t qun tng no v nu c nhiu hn hai c t th s c hai qun tng cng nm trn mt ng cho v chng tn cng ln nhau. Do c t nht 2n vung c nh du bi 1 (2n vung ny gm cc c t tng v cc gc khng c t tng). t S l tng tt c cc s c vit trn bn c. Bi v c t nht 2n phn t cng thm vo S 1 n v v n2 2n phn t cng thm vo S t nht 2 n v nn ta c S 2n + 2(n2 2n) = n(2n 2) (1) By gi gi s rng trong cch xp cho c B qun tng trn bin v I qun tng pha bn trong ca bng (do B + I = 2n 2). Mt qun tng trn bin lun kim sot chnh xc n vung (bao gm c chnh n). V d, nu mt qun tng nm trn mt vung trn hng u tin hoc hng cui cng, n s kim sot chnh xc mt vung trn mt ct. Mt khc, mt qun tng trn mt vung nm pha trong s kim sot t nht n + 1 vung. Nhng khi mt qun tng kim sot mt vung th n thm mt n v vo s c vit trn vung . Khi , chng ta c: S nB + (n + 1)I = n(B + I) + I = n(2n 2) + I

Kt hp vi (1) suy ra n(2n 2) + I n(2n 2). Suy ra I 0. M ta lun c I 0. Vy I = 0 chng t tt c cc qun tng bin.

Bi ton 5. C bao nhiu cch xp 8 con xe ln bn c vua sao cho khng c con xe no nm trn ng cho chnh (ng cho ni gc trn bn tri v gc di bn phi) v khng c con no n con no? Li gii. 2 K hiu un l s cch xp cc qun xe tha bi ng vi bn c n n. Gi con xe hng th i l Xi . Xt bn c n n.Khng gim tng qut, c th gi s X1 nm B1 (sau cng s nhn n 1ln). Nu X2 nm A2 : Ta c th xa i (khng quan tm n) hng 1, 2 v ct A, B. Phn cn li ca bng c s cch xp qun xe l un2 . Nu X2 nm cc cn li (tc l khc A2, B2), ta c th xa i hng 1 v ct B, sau tnh tin ct A li to thnh mt bn c mi vi yu cu xp cc qun xe vn nh c. Vy s cch xp l un1 . R rng, vi cch gii quyt bi ton bng tnh tin bn c nh vy th d X1 nm u ta cng ch c hai trng hp ca cc qun xe nh trn, v kt qu cng tng t. Do c th
2

Ca bn Nguyn Hng,Trng THPT Nguyn Thng Hin, TP.HCM.

170 gi s nh trn. Vy un = (n 1)un1 + (n 1)un2 vi u1 = 0, u2 = 1. Ta cn tnh u8 . Ta tnh c u3 = 2, u4 = 3(2 + 1) = 9, u5 = 4(u4 + u3 ) = 44, u6 = 5(u5 + u4 ) = 265, u7 = 6(u6 + u5 ) = 1854, u8 = 7(u7 + u6 ) = 14833. hiu r hn, sau y l mt s bi tp cc bn t luyn. Bi tp 1. Xc nh s cc cch sp xp cc qun tng v s nh nht cc qun tng c s dng bt k vung no u b kim sot bi t nht mt con tng (a) Trn bn c 8 8. (b) Trn bn c 9 9. (c) Trn bn c 10 10. (d) Trn bn c n n. Bi tp 2. Tm s ln nht cc qun vua c th sp xp khng c hai qun no tn cng ln nhau (a) Trn bn c 8 8. (b) Trn bn c n n. Bi tp 3. Tm s nh nht cc qun vua c th sp xp bt k vung no u b kim sot bi t nht mt qun vua (a) Trn bn c 8 8. (b) Trn bn c n n. Bi tp 4. Tm s ln nht cc qun hu c th sp xp khng c hai qun no tn cng ln nhau (a) Trn bn c 8 8. (b) Trn bn c n n. Bi tp 5. (a) Tm s ln nht cc qun m c th sp xp trn bn c 8 8 khng c hai qun no tn cng ln nhau. (b) Xc nh s cc cch sp xp cc qun m trn bn c 8 8 khng c hai qun no tn cng ln nhau v s ln nht cc qun m c th c s dng..

171

Ti liu tham kho


[1] Tp ch Ton hc v Tui tr. [2] Cc din n Ton hc : Din n Art of Problem Solving (Mathlinks) http://www.artofproblemsolving.com/Forum/index.php Din n MathScope http://forum.mathscope.org/index.php Din n Ton hc VMF http://diendantoanhoc.net/forum

172

STIRLING LO I HAI
Hong Minh Qun1

Cc k hiu s dng trong chuyn


1. [x]0 = 1, [x]n = x(x 1)(x 2) (x n + 1) vi n = 1, 2, 3, . . . 2. [x]0 = 1, [x]n = x(x + 1)(x + 2) (x + n + 1) vi n = 1, 2, 3, . . .

1. nh ngha
Trc khi n nh ngha chng ta bt u vi v d sau: V d 1. Hy ch ra rng c 7 cch phn hoch tp hp {1, 2, 3, 4} gm 4 phn t thnh hai tp con khc rng. Li gii. Chng ta s ch ra c 7 cch phn hoch tha mn bi: {1} {2, 3, 4}, {2} {1, 3, 4}, {3} {1, 2, 4}, {4} {1, 2, 3}, {1, 2} {3, 4}, {1, 3} {2, 4}, {1, 4} {2, 3}. nh ngha 1. S phn hoch tp hp n phn t thnh k khi khng rng (partittions in k blocks) gi l s Stirling loi II, k hiu n . k Ni cch khc, s Stirling loi 2 l s cch phn phi n qu bng phn bit vo k hp ging nhau m khng c hp no rng. iu kh khn i vi chng ta l i tm cng thc tng qut cho s Stirling loi 2. Bi vit sau s ch ra mt s cch chng minh v cc vn lin quan n s Stirling loi 2. Mnh 1. S ton nh t tp n phn t vo tp k phn t bng k!
n k

Chng minh. Ly cc tp hp X = {x1 , x2 , . . . , xk } v Y = {y1 , y2 , . . . , yk }. Ta bit rng mi cch phn hoch mt tp hp tng ng 1-1 vi mt quan h tng ng trn tp hp . C k! ton nh 1-1 nh vy v theo nh ngha c n cch phn chia tp X. k Do ta c s ton nh t tp n phn t vo tp k phn t bng k! n . k

2. Cng thc tnh s Stirling


nh l 1. S Stirling loi 2 c tnh theo cng thc sau : n k
1

1 k!

(1)ki
i=0

k n i i

Gio vin trng THPT Ngc To, H Ni.

173

174 Chng minh. (i) Cch 1. Gi N v K l hai tp hp vi |N | = n, |K| = k. H l tp con ca K v f (H) l s nh x t N ti K \ H. Thay i = k j v p dng nguyn l b tr, ta c: k! n k = #{f : N K | f l ton nh} =
HK k

(1)|H| f (H) (1)j


j=0 k

k (k j)n j k (i)n i

=
i=0

(1)ki

(ii) Cch 2. p dng hm sinh ly tha cho s Stirling loi 2:


+

Gk (x) =
n=k

n xn (ex 1)k = , k = 0, 1, 2, . . . k n! k!
k

Ta c

n=k

1 n xn = k! k n! 1 = k! =

(1)ki
i=0 k

k ix e i k i
+

(1) 1 k!
k

ki

in
n=0

i=0 +

xn n! xn n!

(1)ki
i=0

n=0

k n i i

Mt s gi tr ca s Stirling loi 2
1 1 1 1 3 1 1 7 6 1 = 1 15 25 10 1 . . . . . . . . . . . . . . . . . . . . . . . .

n k

3. Mt s bi ton v s Stirling loi 2


Bi ton 1. Chng minh rng n k =k n1 n1 + k k1

175 Li gii. K hiu [n] l tp gm n phn t. Vi mi cch phn hoch [n] thnh k khi ta u thu c t mt phn hoch [n 1] thnh k khi bng cch b sung phn t n vo mt trong k khi (iu ny c k cch thc hin) hoc thu c t mt phn hoch [n 1] thnh k 1 khi bng cch b sung mt khi mi {n}. T ta c n = k n1 + n1 . k1 k k Bi ton 2. Chng minh rng
n

k=1

n x(x 1)(x 2) (x k + 1) = xn k

Li gii. xn = #{f : N X}
n

=
k=0 n

#{N X : |f (N )| = k} x n k! k k n x(x 1)(x 2) (x k + 1) k

=
k=0 n

=
k=1

Chng hn vi n = 3 ta c 3 3 3 x(x 1)(x 2) = 1 x + 3(x2 x) + 1(x3 3x2 + 2x) x(x 1) + x+ 3 2 1 = x3 Bi ton 3. Chng minh rng vi n 2, ta c n 2 = 2n1 1

Li gii. tng : y l mt ng thc cha bin n. i vi nhng bi ton cha mt bin n chng ta thng chng minh bng phng php quy np. C th bi ny ta lm nh sau: Vi n = 2 ta c: 2 = 1, 21 1 = 1 vy 2 = 21 1 (ng) 2 2 Gi s ng thc ng vi n = k 2, tc l k = 2k1 1, ta chng minh ng thc ng 2 vi n = k + 1 , tht vy: k+1 2 = k k +2 1 2 (theo bi ton 1)

= 1 + 2(2k1 1) Vy bi ton c chng minh.

= 1 + 2k 2 = 2(k+1)1 1

Bi ton 4. Chng minh ng thc v hm sinh ly tha cho s Stirling loi 2 :


+

fk (u) =
n=k

n un (eu 1)k = , k = 0, 1, 2, . . . k n! k!

176 Li gii. T tnh cht tn = xy dng c hm sinh


n k=0 n k

[t]k , n = 0, 1, 2..., p dng cng thc nh thc Newton ta


n

f (t, u) = Ta c f (t, u) = Mt khc ta c

n=0 k=0

un n = etu [t]k n! k
k=0

k=0 n=k

n un [t]k = k n!

fk (u)[t]k

(1)

f (t, u) = [1 + (e 1)] = T (1) v (2) ta suy ra

k=0

t (eu 1)k = k

k=0

(eu 1)k [t]k k!

(2)

fk (u) =
n=k

n un (eu 1)k = k n! k!

Bi ton 5. Chng minh rng s Stirling loi 2 vi k = 0, 1, 2, . . . , n = 0, 1, 2, . . . c biu din di dng sau: n n r n+1 = r k k+1 r=k Li gii. T cng thc tn =
n k=0 n n k

[t]k , n = 0, 1, 2, . . . () v t

n+1 0

= 0 ta c

k=0

n+1 [t + 1]k+1 = (t + 1)n+1 k+1

Mt khc li c [t + 1]k+1 = (t + 1)[t]k Do ta c n


k=0

n+1 [t]k = (t + 1)n k+1

p dng cng thc nh thc Newton v t () ta c:


n

k=0

n+1 [t]k = (t + 1)n = k+1 =

r=0 n

n r
n

k=0

r [t]k k r k [t]k

k=0

r=k

n r

Vy n+1 k+1 =

r=k

n r

r k

You might also like